X



トップページ数学
1002コメント527KB
面白い問題おしえて〜な 26問目
レス数が1000を超えています。これ以上書き込みはできません。
0001132人目の素数さん
垢版 |
2018/02/19(月) 00:21:10.33ID:uzLAXv/z
過去ログ
http://www3.tokai.or.jp/meta/gokudo-/omoshi-log/
まとめwiki
http://www6.atwiki.jp/omoshiro2ch/

1 http://cheese.2ch.net/test/read.cgi/math/970737952/
2 http://natto.2ch.net/test/read.cgi/math/1004839697/
3 http://science.2ch.net/test/read.cgi/math/1026218280/
4 http://science.2ch.net/test/read.cgi/math/1044116042/
5 http://science.2ch.net/test/read.cgi/math/1049561373/
6 http://science.2ch.net/test/read.cgi/math/1057551605/
7 http://science2.2ch.net/test/read.cgi/math/1064941085/
8 http://science3.2ch.net/test/read.cgi/math/1074751156/
9 http://science3.2ch.net/test/read.cgi/math/1093676103/
10 http://science4.2ch.net/test/read.cgi/math/1117474512/
11 http://science4.2ch.net/test/read.cgi/math/1134352879/
12 http://science6.2ch.net/test/read.cgi/math/1157580000/
13 http://science6.2ch.net/test/read.cgi/math/1183680000/
14 http://science6.2ch.net/test/read.cgi/math/1209732803/
15 http://science6.2ch.net/test/read.cgi/math/1231110000/
16 http://science6.2ch.net/test/read.cgi/math/1254690000/
17 http://kamome.2ch.net/test/read.cgi/math/1284253640/
18 http://kamome.2ch.net/test/read.cgi/math/1307923546/
19 http://uni.2ch.net/test/read.cgi/math/1320246777/
20 http://wc2014.2ch.net/test/read.cgi/math/1356149858/
21 http://wc2014.2ch.net/test/read.cgi/math/1432255115/
22 http://rio2016.2ch.net/test/read.cgi/math/1464521266/
23 http://rio2016.2ch.net/test/read.cgi/math/1497416499/
24 http://rio2016.2ch.net/test/read.cgi/math/1502016223/
25 http://rio2016.5ch.net/test/read.cgi/math/1502032053/
0002132人目の素数さん
垢版 |
2018/02/19(月) 00:29:56.44ID:uzLAXv/z
保守がてら

自分のIDに含まれている数字で10を作れ。
最初に成功した者が勝者である。
四則演算のみ使ってよい。
数字の分離、結合、並びの変更は認める。

ID:43ab2017 → 20/4+1+7-3=10
0003132人目の素数さん
垢版 |
2018/02/19(月) 01:03:59.02ID:d7VVg8qs
削除依頼を出しました
0004132人目の素数さん
垢版 |
2018/02/19(月) 01:21:31.07ID:/8jC6j7+
〔問題980〕
平面上にn個の異なる点を配置する。
どの2点間の距離も、必ず或る2つの実数値のどちらかを取るようにする。
n=3の時は、二等辺三角形をなすように配置する例がある。

1) n=4の時、点の配置をすべて求めよ。
2) n=5の時、点の配置は正五角形に限ると言えるか。
3) n=6の時、条件を満たす点は位置は存在しないことを示せ。

[前スレ.980]

1)は、円周上の4点(2種)、円周上の3点と円の中心(4種)が判明している。>>983
0005132人目の素数さん
垢版 |
2018/02/19(月) 01:33:40.61ID:/8jC6j7+
〔問題970〕
a,b の最大公約数を gcd(a,b)と略記する。
gcd(a,b)= 1 をみたす a,b∈Z と 任意の n∈Z(n≠0)に対して、
gcd(ax+b,n)= 1 をみたす x∈Z が存在することを示せ。

[前スレ.970-971]
0006132人目の素数さん
垢版 |
2018/02/19(月) 01:35:31.90ID:3Kr+w6c6
975 132人目の素数さん 2018/02/18(日) 05:13:40.49 ID:3SEy6oaf
(1) 0<θ<πで、2-2cosθと(sinθ)^2の大小を比較せよ。
(2) πを下から抑えるとき、円に内接する正多角形の周長と面積のどちらを用いるのがよいか。
(3) πを上から抑えるとき、円に外接する正多角形の周長と面積のどちらを用いるのがよいか。

(参考)
一般の平面図形で、与えられた周長で面積が最大となるものは円である(等周問題)。
従って、ある平面図形の周長が2πより短いとき、(周長の半分の数値)>(面積の数値)である。


976 132人目の素数さん sage 2018/02/18(日) 06:31:15.01 ID:3BoN6Yxt
>>975
(1)
(2-2cosθ)-(sinθ)^2=(1-cosθ)^2
0<θ<πで、cosθ<1なので2-2cosθ>(sinθ)^2
0007132人目の素数さん
垢版 |
2018/02/19(月) 02:32:16.03ID:Qxq3n/kp
974
問.点Oを中心とする半径1の円と、点Aがあり、OA=2とする。
円周上に点P,Q,Rをとり、Oから直線APへの距離は1/2とする。
また、AP上P側に点XをAX=5となるように取り、
さらに、AQ上、AR上それぞれQ,R側に点Y,Zを取ると、
四角形PQYX,及び四角形QRZYはそれぞれ円に内接するという。
∠XYZ=120°の時、XZの長さを求めよ。
0008132人目の素数さん
垢版 |
2018/02/19(月) 02:47:23.03ID:uzLAXv/z
解答は手に入れたが、貼る前にもう一度考えてみて

[23-937]
[24-30]
地球上の2地点A,B間を飛行機で移動する。このとき、飛行機がA,Bの両方より北側(高緯度側)を通ることがあるためのA,Bの位置関係を答えよ。
例えば、東京(N35.7°,E139.7°)と
・ロンドン(N51.5°,W0.1°)
・メキシコシティー(N19.4°,W99.1°)
・リオデジャネイロ(S22.9°,W43.2°)
はこの位置関係にあるが、
・大阪(N34.7°,E135.5°)
・ヨハネスブルク(S26.2°,E28.0°)
・ブエノスアイレス(S34.6°,W58.4°)
はこの位置関係にない(東京より北は通らない)
地球は球と見なせるとし、飛行機は2地点間を最短距離で(大圏航路で)移動する。
また、球面上の2点を最短距離で結ぶ線は、球面をその中心を通る平面で切った円(大円)の弧になることが知られている。

[24-437]
3^m+4^n=5^kを満たす非負整数の組(m,n,k)をすべて求めよ。
0009132人目の素数さん
垢版 |
2018/02/19(月) 08:52:47.71ID:Qj8lOgw5
【カーマイケル数】
gcd(a,n)=1 をみたす任意のa∈Zに対して、a^(n-1)≡1 (mod n) をみたすn∈Nをカーマイケル数という。
以下を証明せよ。

(1) カーマイケル数は奇数である。

(2) n∈Nに対して、nがカーマイケル数であるための必要十分条件は、(i)かつ(ii)である。
 (i) nは合成数で、平方因子をもたない
 (ii) nの任意の素因数pに対して、n-1はp-1で割り切れる

(3) カーマイケル数は、3個以上の異なる素数の積である。

(4) 6k+1、12k+1、18k+1がすべて素数のとき、n = (6k+1)(12k+1)(18k+1) はカーマイケル数である。
0010132人目の素数さん
垢版 |
2018/02/19(月) 08:59:11.10ID:VF4EpRLf
>>9
n=2
0011132人目の素数さん
垢版 |
2018/02/19(月) 12:33:56.24ID:Qj8lOgw5
素数p、整数a, bに対して、a≡b (mod p) ならば、a^p≡b^p (mod p^2) であることを示せ。

これは簡単杉か…
0012132人目の素数さん
垢版 |
2018/02/19(月) 23:51:59.50ID:VF4EpRLf
>>11
a-b=np
a=b+np
a^p=(b+np)^p=b^p+pb^(p-1)np+...+(np)^p
a^p-b^p=p^2{b^(p-1)n+...+n^pp^(p-2)}
0013132人目の素数さん
垢版 |
2018/02/20(火) 01:44:46.11ID:sbEXHfX1
>>9
(4)
Chernick のカーマイケル数と云うらしい。 (A033502)
k=1, 1729,
k=6, 294409,
k=35, 56052361,
k=45, 118901521,
k=51, 172947529,
k=55, 216821881,
k=56, 228842209,
k=100, 1299963601,
k=121, 2301745249,

なお、逆は成り立たないようです。

・3因子のうち2つだけが素数であるカーマイケル数 (A242980)
k=5, 172081, (4), 18k+1=91=7*13,
k=11, 1773289, (4), 12k+1=133=7*19,
k=15, 4463641, (4), 6k+1=91=7*13,
k=33, 47006785, (5), 18k+1=595=5*7*17,
k=61, 295643089, (4), 18k+1=1099=7*157,
k=85, 798770161, (4), 6k+1=511=7*73,
k=96, 1150270849, (5), 18k+1=1729=7*13*19,
k=115, 1976295241, (4), 18k+1=2071=19*109,

・3因子のうち1つだけが素数であるカーマイケル数 (A242981)
k=22, 13992265, (5), 6k+1=133=7*199, 12k+1=265=5*53,
k=105, 1504651681, (5), 12k+1=1261=13*97, 18k+1=1891=31*61,
0014132人目の素数さん
垢版 |
2018/02/20(火) 02:21:17.57ID:sbEXHfX1
>>11 >>12
pは素数でなくても成り立ちそう… (ただしp≧2)

a^p - b^p = (a-b) {a^(p-1) + a^(p-2) b + …… + a b^(p-2) + b^(p-1)}
ここで
a-b = np,
a = b+np,
{……}≡ b^(p-1) ×(p項) ≡ 0,  (mod p)

>>13 の訂正
k=22, n=13992265, (5つの素数の積), 6k+1=133=7*19, 12k+1=…
0015132人目の素数さん
垢版 |
2018/02/20(火) 09:36:02.27ID:X8ym3XpP
>>9
(2)の条件に奇数が抜けていたので修正します。
内容が被ってしまうので、問題自体を書き直します。スマソ。

--------------------------------------------------
【問題:カーマイケル数】
gcd(a,n)=1 をみたす任意のa∈Zに対して、a^(n-1)≡1 (mod n) をみたす n∈N をカーマイケル数という。
以下を証明せよ。

合成数 n∈N に対して、nがカーマイケル数であるための必要十分条件は、(i)かつ(ii)かつ(iii)である。
 (i) nは奇数である。
 (ii) nは平方因子をもたない。
 (iii) nの任意の素因数pに対して、n-1はp-1で割り切れる。

(1) 合成数 n∈N がカーマイケル数ならば、(i)を示せ。
(2) 合成数 n∈N がカーマイケル数ならば、(ii)を示せ。
(3) 合成数 n∈N がカーマイケル数ならば、(iii)を示せ。
(4) 合成数 n∈N が(i),(ii),(iii)をみたすならば、n はカーマイケル数である。
(5) カーマイケル数は、3個以上の異なる素数の積である。
(6) 6k+1、12k+1、18k+1がすべて素数ならば、n = (6k+1)(12k+1)(18k+1) はカーマイケル数である。
--------------------------------------------------

(1)(4)(5)(6)は、易。
(2)は、私の用意した解答は間違っていますた。(フェルマーの小定理を誤用)
(3)は、(1)(2)と原始根と中国剰余定理を使って証明した。
0017132人目の素数さん
垢版 |
2018/02/20(火) 10:18:17.64ID:X8ym3XpP
>>11を少し変えてみる。

素数p、整数a, bに対して、a^p≡b^p (mod p) ならば、a^p≡b^p (mod p^2) であることを示せ。
0018132人目の素数さん
垢版 |
2018/02/20(火) 10:45:51.52ID:nXzkbh+j
>>17
a=a^p=b^p=b (p)
0019132人目の素数さん
垢版 |
2018/02/21(水) 02:41:30.51ID:xFlMdI8t
>>17

pは素数だから、フェルマーの小定理( >>18)により
a^p ≡ a (mod p)
b^p ≡ b (mod p)
∴ a ≡ b (mod p)

p≧2 だから、>>12 >>14 により、
a^p ≡ b^p (mod pp)
0020132人目の素数さん
垢版 |
2018/02/22(木) 02:44:52.37ID:/AvHZMpx
一辺が整数の正方形で、3頂点からの距離が整数になる点を内部(周含まず)に持つものを挙げよ。
なお、4頂点すべてからの距離が整数になる場合は未解決である。
0022132人目の素数さん
垢版 |
2018/02/22(木) 21:40:02.05ID:QrRPfkyE
>>21
A(0,0),B(52,0),C,D,P(24,45)
これが最小の正方形?

条件を満たす正方形ABCDと内部の点Pの例は無限にある

A(0,0),B(700,0),C,D,P(396,297)
A(0,0),B(3364,0),C,D,P(945,900)
A(0,0),B(10952,0),C,D,P(1155,396)
A(0,0),B(12675,0),C,D,P(8288,4884)


もちろんPとy=x,y=-x+1について対称な点も条件を満たす(各正方形に計4つ)。

解説や類例(英語)
http://mathworld.wolfram.com/RationalDistanceProblem.html
0023132人目の素数さん
垢版 |
2018/02/22(木) 22:30:00.29ID:QyvGUXHk
>>22
一辺52は最小
一辺1000未満の解も他にいくつか
A(0,0),B(195,0),C,D,P(48,140)
A(0,0),B(740,0),C,D,P(168,315)
A(0,0),B(867,0),C,D,P(416,780)
A(0,0),B(996,0),C,D,P(520,765)
4頂点すべてからの距離が整数になる場合があるとしたら、一辺は12の倍数になるはず
0024132人目の素数さん
垢版 |
2018/02/24(土) 08:07:10.34ID:AT99r3l3
n,k,iを自然数とし、自然数b1,...,bnがあり、b1+b2+...+bn =k とする。
この時、次の条件を満たす自然数の数列a1,...anが存在することを示せ。
・ai>ai+1
・ai≧bi
・a1+...+an=k^2
0025132人目の素数さん
垢版 |
2018/02/24(土) 08:32:12.28ID:6AjWBEuj
k=2,b1=b2=1
ai=?
0027132人目の素数さん
垢版 |
2018/02/24(土) 09:13:38.88ID:IV3aQ+/t
>>22
一辺が 12675 のとき、互いに対称でない解が複数ある
A(0,0),B(12675,0),C,D,P(8288,4884)
A(0,0),B(12675,0),C,D,P(9100,3120)
A(0,0),B(12675,0),C,D,P(9588,784)
2番目は既約でなく一辺が 195 の解と本質的に同じ
3番目は既約
0028132人目の素数さん
垢版 |
2018/02/24(土) 13:00:01.29ID:lgGVwMHM
k+k^2-kn+n(n-1)/2.
k-1.
k-2.
...
k-n+1.
0029132人目の素数さん
垢版 |
2018/02/24(土) 16:56:19.85ID:AT99r3l3
>>28
正解!感想は?
0030132人目の素数さん
垢版 |
2018/02/24(土) 21:20:28.33ID:h/Dh65p2
>>24
これいつかのIMOよな
0031132人目の素数さん
垢版 |
2018/02/25(日) 06:01:36.41ID:gGaVEUAO
〔掛谷の定理〕
正係数のn次多項式を
F(x)= a_0・x^n + a_1・x^{n-1} + …… + a_{n-1}・x + a_n,
とし、
F(x)= 0 の解をαとする。
 a_0 > a_1 > a_2 > …… > a_{n-1}> a_n > 0 ならば |α|< 1.
 0 < a_0 < a_1 < a_2 < …… < a_{n-1}< a_n ならば |α|> 1.
0033132人目の素数さん
垢版 |
2018/03/01(木) 20:09:04.83ID:Ti0zUc6I
>>32
3^2+5^2+7^2=83
0034132人目の素数さん
垢版 |
2018/03/01(木) 23:52:59.95ID:d3ZXiDwQ
では、相異なる3つの 「5以上の素数」 の平方和は、素数にならないことを示せ。
0035132人目の素数さん
垢版 |
2018/03/02(金) 00:58:19.40ID:aJjdMvYA
>>34
5以上の素数は、その平方がいずれも3を法として1と合同であり、3つの和が常に3の倍数となるから
「相異なる」の条件は付けなくても問題は成立する
0036132人目の素数さん
垢版 |
2018/03/02(金) 01:34:29.12ID:Ub9nfASN
こう書けばよかったんですね、さんくす。
「3と異なる3つの素数の平方和は、素数でない」
0039132人目の素数さん
垢版 |
2018/03/03(土) 16:58:06.28ID:MMc1xkls
x^5 +2x^2 + 3x + 4 ≡ 0 (mod 5) の解は、x=0,1,2,3,4を代入して x≡1,2 (mod 5) を分かるけど、
これを無理やり因数分解できないものか?

  x^5 +2x^2 + 3x + 4
≡x^5 +2x^2 - 2x - 1
=(x^5-1) + 2x(x-1)
≡(x-1)(x^4 +x^3 +x^2 + 3x + 1) (mod 5)

ここまでいったが、x^4 +x^3 +x^2 + 3x + 1 が x≡2 (mod 5) だけを解にもつから (x-2)をくくりだしたいんだけど。
00416
垢版 |
2018/03/03(土) 23:54:29.55ID:hj2seOgl
>>6の正解

θ=2π/nとする。


(2)
単位円に内接する正n角形の面積は(n/2)sinθ、半周長は(n/2)(√(2-2cosθ))
(1)より
(n/2)sinθ<(n/2)(√(2-2cosθ))
よって、πを下から抑えるときは周長を用いた方がよい。

ちなみに
(n/2)(√(2-2cosθ))=((2n)/2)sin(2π/(2n))
より
(単位円に内接する正2n角形の面積)=(単位円に内接する正n角形の半周長)
である。
面積で抑えようとするのは効率が悪い。


(3)
単位円に外接する正n角形の一辺の長さをaとおけば、面積も半周長もna/2であり等しい。
よって、πを上から抑えるときはどちらを用いても変わりはない。

実際に計算すると
na/2=n√((1-cosθ)/(1+cosθ))=n(1-cosθ)/(sinθ)=n(sinθ)/(1+cosθ)
00426
垢版 |
2018/03/03(土) 23:55:21.58ID:hj2seOgl
【参考】
単位円について
(内接正n角形の面積) < (内接正n角形の半周長) < π < (外接正n角形の面積,半周長)
の一覧


n=3
(3/4)√3 < (3/2)√3 < π < 3√3

n=4
2 < 2√2 < π < 4

n=5
(5/8)(√2)√(5+√5) < (5/4)(√2)√(5-√5) < π < 5√(5-2√5)

n=6
(3/2)√3 < 3 < π < 2√3

n=8
2√2 < 4√(2-√2) < π < 8(-1+√2)

n=10
(5/4)(√2)√(5-√5) < (5/2)(-1+√5) < π < 2(√5)√(5-2√5)

n=12
3 < 3(√2)(-1+√3) < π < 12(2-√3)

n=16
4√(2-√2) < 8√(2-√(2+√2)) < π < 8(√2)(√(2-√2))(2-√(2-√2))

n=20
(5/2)(-1+√5) < (5/2)(√2)(1+(√5)-(√2)√(5-√5)) < π < 5(1+√5)(4-(√2)√(5+√5))

n=24
3(√2)(-1+√3) < 6(√2)√(4-(√2)-√6) < π < 12(1+√3)(-1-(√3)+2√2)


(内接正16角形の半周長)のみ3重根号を用いている。

n=24の不等式を用いてπを評価すると
3.1326… < π < 3.1596…
である。
0043132人目の素数さん
垢版 |
2018/03/04(日) 00:56:38.21ID:S6IbgXV0
ところで>>41,42では
(1/2)(単位円に内接する正n角形の周長) < (1/2)(単位円の円周2π)
を用いている。
「円に内接する(凸)多角形の周長は円周より小さい」
は、各辺(弦)が弧より短いので自明である。

では
「円に包含される凸多角形の周長は円周より小さい」
更には
「凸多角形Aに包含される凸多角形Bの周長はAの周長より小さい」
は、初等数学(できれば初等幾何)で証明可能か?



凸多角形Bは凸多角形Aに包含される
⇒任意のxy座標系において[B(x,y)のxの変域]⊆[A(x,y)のxの変域]かつ[B(x,y)のyの変域]⊆[A(x,y)のyの変域]
0044132人目の素数さん
垢版 |
2018/03/05(月) 03:27:18.13ID:G91DujrK
出題!
a,bは0<a<bなる実数として
平面図形
{(x,y)∈R^2|a≦x^2+y^2≦b}
を考える。
まあ要するにバウムクーヘンを上から見た形ですね。

これを4つの直線で切って、いくつかの部分に分ける。
最大何個に分けられるでしょう?
0045132人目の素数さん
垢版 |
2018/03/05(月) 04:49:24.03ID:/FRv3F/K
>>20

A(0,0) B(L,0) C(L,L) D(0,L) P(x,y)
とおく。
 {L-x,y} 組、{L-y,x} 組はピタゴラス数だから h,m,n により
 L-x-y = ±h{(nn-mm) -2mn},
と書ける。
 その因数は、7,17,47,217,257 などで、1の位の数字は7である。(なぜか?)

(nn-mm) - 2mn = ±(L-x-y)/h
は、下記のように ペル方程式 ff -2gg = ±1 の形になる。
その解は
 f_k = {(1+√2)^k + (1-√2)^k} /2,
 g_k = {(1+√2)^k - (1-√2)^k} /(2√2),
(+のときはk:偶数とし、−のときはk:奇数とする。)

・(nn-mm) -2mn = 7 のとき
 (nn-mm) -2mn = (n-m)^2 -2mm = {(5m-3n)^2 -2(4m-n)^2}/7 = {(m-3n)^2 -2(2m+n)^2}/7,

・2mn -(nn-mm) = 7 のとき
 2mn -(nn-mm) = (m+n)^2 -2nn = {(5m-n)^2 -2(3m-2n)^2}/7 = {(3m+n)^2 -2(m-2n)^2}/7,

・(nn-mm) -2mn = 17 のとき
 (nn-mm) -2mn = (n-m)^2 -2mm = {(9m-5n)^2 -2(7m-2n)^2}/17 = {(m-5n)^2 -2(3m+2n)^2}/17,

・2mn -(nn-mm) = 17 のとき
 2mn -(nn-mm) = (m+n)^2 -2nn = {(5m+n)^2 -2(2m-3n)^2}/17 = {(7m-n)^2 -2(4m-3n)^2}/17,

・(nn-mm) -2mn = 47 のとき
 (nn-mm) -2mn = (n-m)^2 -2mm = {(9m-7n)^2 -2(8m-n)^2}/47 = {(5m-7n)^2 -2(6m+n)^2}/47,

・2mn -(nn-mm) = 47 のとき
 2mn -(nn-mm) = (m+n)^2 -2nn = {(7m+5n)^2 -2(m-6n)^2}/47 = {(17m-5n)^2 -2(11m-6n)^2}/47,

∴ ff -2gg = ±1 の形になる。
0046132人目の素数さん
垢版 |
2018/03/05(月) 05:24:46.33ID:/FRv3F/K
>>44

x = ±d と y = ±d で 「井の字」
 但し、d = min{ (1/2)√(a+b),√a }
だと 12個か。
もっと多いんだろうな。
0047132人目の素数さん
垢版 |
2018/03/05(月) 09:15:36.94ID:bwF+lzst
新手(荒手とも)の技で導いたので一般的(?)な解放が見たいです(*´ 艸`)

マクローリンです
ついでにゼータも分解すると(1)の類題だと分かる形になります
因みにRamanijanの発見した恒等式を使用しました


https://i.imgur.com/EkGvNIH.jpg
https://i.imgur.com/bSvsQUN.jpg
0048132人目の素数さん
垢版 |
2018/03/05(月) 10:41:19.19ID:MUI2m42J
>>45
(L,x,y)=(6240,711,3080),(26180,3285,24528)という反例もあるけど、7や17の倍数になる例は確かに多いと思います
何か法則が…?
0049132人目の素数さん
垢版 |
2018/03/05(月) 11:53:26.30ID:MUI2m42J
>>45
>f_k = {(1+√2)^k + (1-√2)^k} /2,
これは1,3,7,17,41,99,239,577,1393,…となると思うんですが、
その中でも特に7と17だけ際立って多く登場するのはなぜでしょうね?
0050イナ ◆/7jUdUKiSM
垢版 |
2018/03/05(月) 12:30:00.14ID:ctb2v+HR
┏┳┓>>44>>46
 ̄┣━━◎ ̄ ̄ ̄/\
_◎______/\/|
 ̄ ∩∩ ̄ ̄ ̄ ̄\/ |
⊂(_-) )`⌒ つ ̄/ |
 ̄|、_`υ___/| |
]|‖ ̄ ̄ ̄ ̄‖ | /
_|‖ □ □ ‖ |/
 ̄`‖____‖/
_   ̄ ̄ ̄ ̄ ̄おもしろい。CDに細い紐4本、井桁に置いてずらしていくと、ちょうど頂角36°底角72°の二等辺三角形ができるときが星形で、それまでのあいだだろうなって見当はつく。かぞえるとバウムクーヘンの切れ端は13個あるね。
0051132人目の素数さん
垢版 |
2018/03/05(月) 14:00:02.90ID:/FRv3F/K
>>47

Σ[n=-∞,∞] e^{-πn^2} = θ_3(0, e^{-π}) = 1.086434811213308

Σ[m=-∞,∞] m^2 e^{-πm^2} = 0.0864557352758541

辺々割って 4π.

Σ[n=1,∞] (-π)^n / {n! ζ(2n+1)} = -0.568682

Σ[m=0,∞] (-π)^m / {m! ζ(2m+3)} = -0.0452297

辺々割って 12.5732
0052132人目の素数さん
垢版 |
2018/03/05(月) 21:54:28.59ID:/FRv3F/K
>>44

d = √{ab/(a+b)} とおく。0<a<b より
 √(a/2) < d < min{(1/2)√(a+b),√a}.

x = -d,
y = 0 (x軸),
(-d,d)と(d√2,0) を通る直線,
(-d,-d)と(√{(a+b)/2},0)を通る直線

これで13個だ…
0053132人目の素数さん
垢版 |
2018/03/05(月) 22:24:30.70ID:ziXyPtrs
平面をn本の直線で分割するとき、最大でa[n]個の領域に分割するとき、
a[n+1] = a[n]+n+1

平面上に円が1つあって、n本の直線で分割するとき、最大でb[n]個の領域に分割するとき、
b[n]は?

平面上に同心円が2つあって、n本の直線で分割するとき、最大でc[n]個の領域に分割するとき、
c[n]は?

平面上に同心円がm個あって、n本の直線で分割するとき、最大でd[n]個の領域に分割するとき、
d[n]は?
0054132人目の素数さん
垢版 |
2018/03/05(月) 23:14:45.10ID:7fA6cixH
>>49
7や17をL-x-yの素因数に含む場合は複数解を含むケースが多い

L-x-y=833=7・7・17である解3つ
(L,x,y)=(11492,7524,3135),(21125,7524,12768),(43700,7524,35343) いずれも L-y=8357

L-x-y=4879=7・17・41である解2つ
(L,x,y)=(13156,1440,6837),(14355,1440,8036) いずれも L-y=6319

L-x-y=5593=7・17・47である解3つ
(L,x,y)=(9375,7072,7896),(21853,5040,11220),(22472,13260,14805)

L-x-y=6713=7・7・137である解2つ
(L,x,y)=(20280,4795,8772),(282348,15960,259675)
0056132人目の素数さん
垢版 |
2018/03/06(火) 08:37:05.23ID:UZDPZhtT
>>4が未解決ですが、解を知りたいものですね

さて、エレ解の募集です
(用意した想定解は地道にやっています)

xy平面上を点Pが原点(0,0)からスタートして、以下に定められる規則に従って移動する。次の設問に答えよ。

点Pの移動規則:
サイコロを振り、1の目が出た場合(2,0)、2の目が出た場合(1,√3)、3の目が出た場合(-1,√3)、4の目が出た場合(-2,0)、5の目が出た場合(-1,-√3)、6の目が出た場合(1,-√3)だけ点Pは移動する。
サイコロを8回振る。
最終的な点Pの位置をP_8とする。

原点と点P_8の長さが整数になる確率を求めよ。
但し原点と点P_8が一致した場合は長さ0とし、整数に含む。
0057132人目の素数さん
垢版 |
2018/03/06(火) 11:34:45.51ID:XWVSzSDt
>>56
原点からの距離が14となる場合が18通りあるが、それらの場合における確率がいずれも等しいところが興味深い
自明な結果ではないはず
0059132人目の素数さん
垢版 |
2018/03/06(火) 13:53:40.61ID:29oWx1/5
>>58
内側の円が充分小さければ14個できる
0060132人目の素数さん
垢版 |
2018/03/06(火) 14:21:06.54ID:NuR3ze1m
四角形を角の付近を残して中を除けば11+3
0061132人目の素数さん
垢版 |
2018/03/06(火) 14:32:43.92ID:puCh8V/h
円の大きさは関係ないかな
4本すべての直線が内側の円の内部を通るようにし、かつ、
どの2直線も図形の内側(内側の円の外部かつ外側の円の内部)の、それぞれ異なる点で交わるようにすると14分割になる
0062132人目の素数さん
垢版 |
2018/03/06(火) 23:05:51.85ID:4AuUlXe8
【湖畔の街灯】

観測者が、光源から受ける光の明るさ・電荷から受ける静電気力の大きさ・物体から受ける引力の大きさ…は逆二乗則に従う(すなわち距離の二乗に反比例する)。
観測者が距離1の光源から受ける光の明るさを1とする。
次のとき、θ=0にいる観測者が受ける光の明るさはいくらか?

(n=0) 直径2/πの円周上のθ=πの位置に光源があるとき
(n=1) 直径4/πの円周上のθ=π/2, 3π/2の位置に光源があるとき
(n=k) 直径(2/π)*2^kの円周上のθ={aπ/(2^(k-1))}-{π/(2^k)}の位置に光源があるとき(ただしa=1,2,…,2^k)

一周2^(n+1)の円形の湖に、2^n個の街灯が等間隔で並んでいるイメージである。


無限に大きい円を考えると、どのような数論の公式が導けるか?
0063132人目の素数さん
垢版 |
2018/03/07(水) 02:06:27.88ID:Zv6uWX5c
>>53
a[n] = (nn+n+2)/2,

>>61 に従って
直線0:  y=0 (x軸)
 線分(√a,0)〜(√b,0)をn等分する点をP_k (k=1,…,n-1)
 線分(√a,δ)〜(√a,0)をn等分する点をQ_k(k=1,…,n-1)
直線k: P_k と Q_k を通る直線
とする。 
δ>0 がじゅうぶん小さいとき、n本の直線は小円の内部を通る。

c[n+1] = c[n] + n+2,
c[n] = n(n+3)/2,
0066132人目の素数さん
垢版 |
2018/03/07(水) 16:32:10.13ID:Zv6uWX5c
>>63 (補足)

d ' = √b - √a とおく。
線分kは放物線
 √{(x-√a)/d '} + √(y/δ) = 1
に接する。その接点は
(√a + (P_0 P_k)^2 /d ',(Q_k Q_n)^2 /δ)

次に、
線分 P_0(√a,0)〜 P_n(√a +d ',0)をλ:(1-λ)に内分する点をP
線分 Q_0(√a,δ)〜 Q_n(√a,0)を λ:(1-λ)に内分する点をQ とする。
λ = P_0 P / d ' = Q_0 Q / δ.

このとき、線分PQ も上記の放物線に接する。その接点は
(√a + λ^2・d ',(1-λ)^2・δ)
また、2本の接線の交点は
(√a + λ1・λ2・d',(1-λ1)(1-λ2)δ)
0068132人目の素数さん
垢版 |
2018/03/08(木) 12:23:37.55ID:YNRhAtaA
>>63-66

>>61 に従って
直線n: y=0(x軸)
大円内でx軸と小円に接する下に凸な曲線Cを書き、n-1本の接線を曳く。

曲線Cの例:
円(x - √{b-d 'd '})^2 + (y-d ')^2 = (d ')^2
 d ' = √b - √a とおいた。
007044です
垢版 |
2018/03/08(木) 18:24:18.81ID:thv0rQ4l
いちおう自分が考えたのと同じ 必ず14 が出ました。
読むのでもう少々お待ちを。
0071132人目の素数さん
垢版 |
2018/03/09(金) 09:58:52.79ID:uTCYTbKw
>>56
二次元で正六角形を表現するのは面倒だが、三次元内なら簡単に表現できる。サイコロの各出目に対し、
(1,-1,0),(1,0,-1),(0,1,-1),(0,-1,1),(-1,1,0),(-1,0,1) ・・・・(★)
を対応させればよい。この方法を用いると、n回のサイ振り後の、動点Pの位置(x,y,z)は、
x+y+z=0,|x|≦n,|y|≦n,|z|≦n、・・・・・・・・・(☆)
の整数解と1対1に対応できる
(n=1の時、原点もこの方程式の解に含まれるが、動点がここにいることはないのでこれだけは除外する)

問題では一回のサイ振りで2移動するが、この解法では√2の移動に留まる。
従ってこの座標系の距離で√2倍したものが、問題における距離と一致する。
つまり、「動点Pが原点から整数距離にある」⇔「mを整数として、2(x^2+y^2+z^2)=m^2と表せる」
m=2kとおき、整理すると 
x^2+y^2+xy=k^2   ・・・・・・・・・(☆☆)
n=8においてこの解は、{x,y,z}={0,k,-k},{8,-5,-3},{-8,5,3}に限られる。

(★)より、(x/y + x/z + y/z + y/x + z/x + z/y)^8  ・・・・・・(★★)
を展開したとき x^a y^b z^c の係数が、動点Pの8回のサイ振り後、(a,b,c)に到達するルート数に一致することが判る。
(☆☆)の解に対応させると、
(x/y),(x/y)^2,...,(x/y)^8 の係数の和の六倍、プラス、x^8/(y^5z^3) の項の係数の十二倍、プラス 定数項
を6^8で割った物が、この問題の答えになる。
0072132人目の素数さん
垢版 |
2018/03/09(金) 09:59:22.23ID:uTCYTbKw
飛び道具があれば、(★★) を展開し、各係数を見て、計算すれば、それで終了とできる。
だがそでは「エレガント」とは言えないので、別の方法を示す。
最も、煩雑なのが定数項の計算。飛び道具を使う以外に二通りの方法で確認したので、まずそれを示す。

展開式において、x/yをa回、x/zをb回、...z/yをf回掛け合わされ、「定数になる」等という条件を式にすると、
a+b+c+d+e+f=8、a+b=d+e,c+d=a+f,e+f=b+c → abcdef=004004,013013,021203,022022,...,400400 という21通りの非負整数解を見つけられ、
全ての解において 8!/(a!b!c!d!e!f!) を計算し、和を取れば、54810 を得られる。(これは、プログラムにより確認した)

問題では八回のサイ振りが求められているので、まずその半分四回までのサイ振り後のルートを全て計算する。
正三角形方眼紙を用いればパパパッとできる。4回後の各地点へ到達するルート数は、原点90、サイズ1の正六角形の頂点60、
サイズ2の正六角形の頂点34、辺の中点48、その外は頂点から順に12,16,16、その外側は頂点から順に1、4、6、4
このようになる。こうして得られた61カ所に数字の二乗和を取ると、90^2+6*(60^2+48^2+34^2+2*16^2+12^2+6^2+2*4^2+1^2)=54810が得られる。

ところで、(★★)を展開したときの定数項は原点のルート数=(54810)に当たるが、この式において、
x=yと置き換えた時の定数項は、正六角形の原点を通る対角線上の17個の数字の和に当たることが判る。
これは、(パスカルの三角形みたいなものを書けば)手でも十分計算可能で、2^8*1107という値を得る。
これの三倍で、原点を通る三本の対角線をカバーできるが、原点が3重に数えられているので、その超過分を減じ、
x^8/(y^5*z^3)の項の分8!/(5!3!)=56の12倍を加え、確率に直すと、
(1/6^8)(3*2^8*1107-2*54810+12*56)=741228/6^8=0.441308013... が得られ、これが>>67で示した結果。
0073132人目の素数さん
垢版 |
2018/03/09(金) 10:08:50.28ID:uTCYTbKw
訂正
>>71
誤:n=8においてこの解は、{x,y,z}={0,k,-k},{8,-5,-3},{-8,5,3}に限られる。
正:n=8においてこの解は、{x,y,z}={0,k,-k},{8,-5,-3},{-8,5,3}、及び原点に限られる。
0074イナ ◆/7jUdUKiSM
垢版 |
2018/03/09(金) 22:48:41.28ID:20dRp+V2
~人人~ 前>>50
(_)_) 二重の円を
(_(_)゙適当な大きさに
(-_-)) 描き、
○(`')゙バウムクーヘン
○⌒_ノ 上で交差する
(_人_)゙ように5本の線
_υ_υ_ で分割し、内側の円を掠めるようにうまく配置して一本の線をとると、外側の円を含むバウムクーヘンは8つできる。
内側の円を含むバウムクーヘンは二種類あって一つの鋭角を持つ切れ端3つと、一つの鈍角をもつごく小さな切れ端3つが確認できる。
∴8+3+3=14
0075イナ ◆/7jUdUKiSM
垢版 |
2018/03/09(金) 23:02:03.51ID:20dRp+V2
訂正。前>>74
やっぱり二つの円の中心がズレてました。一つの交点が外側の円を出ます。

8+3+2=13 が最大かと。
0076132人目の素数さん
垢版 |
2018/03/10(土) 01:14:51.00ID:Ta7osRmu
>>75
星形にこだわるとうまい解はでないかもしれないです
例えばこういう線を考えてみてはどうでしょうかね
・バウムクーヘン>>44のもの。原点を中心に持つ半径aとbの円に挟まれた図形
・1本目の線を、切片(a+b)/2で正の傾きをもち、内側の円を通る(原点からの距離がa未満)線とする
・2本目の線を、切片(a+b)/2で負の傾きをもち、内側の円を通る(原点からの距離がa未満)線とする
・3本目の線を、切片-(a+b)/2で正の傾きをもち、1番目と2番目の線と第1象限にある図形内の点でそれぞれ交わるようにする
・4本目の線を、切片-(a+b)/2で負の傾きをもち、1番目と2番目の線と第2象限にある図形内の点でそれぞれ交わるようにする
このようにすると、>>61に書かれているように、4本の線は互いに図形内の異なる点で交わり、かつ内側の円の内部を通るようにできます
分割数は14になります
0077イナ ◆/7jUdUKiSM
垢版 |
2018/03/10(土) 08:38:58.94ID:dRmoZgLS
>>76図を書いて確認しました。たしかに14個に分割できますね。前>>75
‖∩∩]‖
((-_-) ‖
(っφ)゚‖
「 ̄ ̄ ̄ ̄]
■/_UU\■_/_/_/_/_/_/_/_/_/_/_/_/_/_/_/_/_/_/_/_/_
0078132人目の素数さん
垢版 |
2018/03/12(月) 10:26:12.21ID:DSncSinI
>>45
>その因数は、7,17,47,217,257 などで、1の位の数字は7である。(なぜか?)
理由は不明ですが、既約解において、L-x-yの素因数は、その平方が48を法として1に合同なものに限られるらしい
(その場合、当然L-x-yの平方も48を法として1に合同)
何かそうでなければならない理由があるのでしょうか
0079132人目の素数さん
垢版 |
2018/03/13(火) 07:19:18.20ID:IdxYrbr8
奇素数 p、自然数 r、gcd(a、p)=1 をみたす整数 a に対して、
x^2≡a (mod p^r) をみたす整数 x が存在するならば、
y^2≡a (mod p^{r+1}) をみたす整数 y が存在することを示せ。
0080132人目の素数さん
垢版 |
2018/03/14(水) 01:28:54.32ID:hdbbxtzk
>>79

gcd(2a,p) = gcd(2,p) gcd(a,p) = 1,
∴ 1-2az ≡0 (mod p) を満たす z が存在する。

xx = a + b・p^r
に対して
y = x (1 - z b・p^r)
とおく。
yy = xx (1 - z b・p^r)^2
 = (a + b・p^r) {1 -2z b・p^r + zz bb・p^(2r)}
 ≡ a + (1 -2az)b・p^r  (mod p^(2r))
 ≡ a          (mod p^(r+1))
0082132人目の素数さん
垢版 |
2018/03/14(水) 18:29:55.30ID:hWHigWj1
半径1の円の周および内部に、

(A) どのようにm個の点を配置しても、ある2点間の距離が1以下になる。最小のmを求めよ。全ての2点間の距離が1より大きくなるm-1個の点の配置を示せ。

(B) どのようにn個の点を配置しても、ある2点間の距離が1未満になる。最小のnを求めよ。全ての2点間の距離が1以上になるn-1個の点の配置を示せ。

論点
(A) 5点の配置は余裕、またmが高々7なのは容易に示せるが、m=6,7のどちらだろうか?
(B) 7点の配置はギリギリ可能だが、n=8なのだろうか?
0083132人目の素数さん
垢版 |
2018/03/15(木) 12:01:35.87ID:0fp5JvfB
>>56
おそらく
原点は2-2-2-2型と3-3-2型(3は135ダブりか246ダブりのみ)からの計算

軸は、
23○
56□
14△
として
○がk個、□がk個、△が(8-2k)個を並べてから各○□△埋める
2^k×2^k×2^(8-2k)×8!/((8-2k)!k!k!)

が最も近道だと思う
0085132人目の素数さん
垢版 |
2018/03/15(木) 18:08:55.23ID:tXGeseDm
>>82
(B)n=8が答え。
8点の中に円の中心が含まれていたら、残りの7点は円周上に無ければならないが、そのような配置は必ずある2点間の距離を1未満にする。
したがって8点とも円の中心と異なる場合のみ考えれば良いが、
ピザを切るように円を7等分すれば、鳩ノ巣原理より同一のピースに含まれるような2点が存在。
2点とも円の中心とは異なるため、必ず距離は1未満になる。
0086132人目の素数さん
垢版 |
2018/03/15(木) 19:55:30.54ID:Vea/5imI
>>83
>>原点は2-2-2-2型と3-3-2型(3は135ダブりか246ダブりのみ)からの計算

aaaa(aaaa)~型: 3通り * 8!/(4!4!)
aaab(aaab)~型: 6通り * 8!/(3!3!)
aabb(aabb)~型: 3通り * 8!/(2!2!2!2!)
aabc(aabc)~型: 3通り * 8!/(2!2!)
(ABC)^2*AA~型: 6通り * 8!/(3!2!2!)

の21通り合計、54810ですね。
(※aに対し、「a~」で、aと反対の方法を、A,B,Cは、お互い120度をなす方向を表し、ABCで元の位置に戻ります。)
(※二つの数字は、方向パターンと並べ替えパターン)
この方法は、「型の列挙」に漏れや重複がないか核心で、別の独立な方法で確認できたなら、自信が持てますよね。

軸の方の
Σ[k=0,4]2^k×2^k×2^(8-2k)×8!/((8-2k)!k!k!) =283392=1107*2^8
はシンプルですね。
0087132人目の素数さん
垢版 |
2018/03/16(金) 04:18:56.25ID:4kz/tEYl
>>82
(A) m=6
・6点のどれかが円の中心ならば、他の点までの距離は1以下。
・6点とも円の中心でない場合、
 6つの中心角の合計が360゚ だから、最小のものは60゚ 以下となり、60゚ の扇形に含まれる。
 ∴ 距離は1以下。

 円周上の正5角形
0088132人目の素数さん
垢版 |
2018/03/16(金) 23:32:33.66ID:RLjYn1OD
有名問題かもしれないけど
「全ての自然数は、3の階乗の足し引きで表されることを示せ。」
例えば4=3+1 11=9+3-1 とかな

高校数学解法辞典? っていうのに載ってて、難易度が難だった
難レベルは同書に数問しか入ってなかった
やや難の問題の方が難しく感じたけどなw
0089132人目の素数さん
垢版 |
2018/03/17(土) 00:28:31.07ID:uUVJ+V3Z
>>88
3!=6だが...?
0091132人目の素数さん
垢版 |
2018/03/17(土) 00:42:15.69ID:uUVJ+V3Z
足すか、引くか、足しも引きもしない、の3通りが選べる。
これが全ての数について言えるから
0093132人目の素数さん
垢版 |
2018/03/17(土) 09:43:19.88ID:cb1s66eH
>>89
階乗じゃなくて累乗やんけ
ごめんなさい
0094132人目の素数さん
垢版 |
2018/03/17(土) 09:45:59.36ID:NB1tvxjk
n=3^0+3~0+…+3~0 (n個の和)
0095132人目の素数さん
垢版 |
2018/03/17(土) 09:59:07.80ID:cb1s66eH
>>88 だけど、問題の定義が曖昧すぎたので原文まんま上げます
考えてた人はごめんなさい
http://imgur.com/FJFbctS.jpg
0096132人目の素数さん
垢版 |
2018/03/17(土) 10:25:23.68ID:jS80gqxS
>>92
Sonntag する
ってそりゃ日曜日
0099132人目の素数さん
垢版 |
2018/03/18(日) 13:46:22.49ID:NgzA8uOp
平衡3進法、
天秤と重さ3^kの分銅がk=0,1,2,...について1個ずつあれば、正の整数の重さは全部表せるってやつだね

(天秤進法→)天進法の名前で商標登録してるやつがいるけどどうなの
0100132人目の素数さん
垢版 |
2018/03/18(日) 14:04:54.73ID:sfYdIshh
↑その人、コラッツ予想を証明してしまっているようだなあ。
数学というより、精神医学の話題なんじゃないの?
0101132人目の素数さん
垢版 |
2018/03/18(日) 23:28:02.32ID:kMHyRC84
>>98
ほー、そう呼ばれてるのか
勉強になったわ、サンクス
証明どうする? 載せた方がいい?
上のサイトに比べたら大したものじゃないけど
0103132人目の素数さん
垢版 |
2018/03/19(月) 18:19:55.56ID:JXYilKRY
80.6 < Σ[k=1→24]√k < 80.65 を示せ
0104132人目の素数さん
垢版 |
2018/03/20(火) 00:15:22.04ID:E4ArtLi4
お待たせ
当時の俺はこんなので感動したもんだ
ちなみにmが最終的に0になることの証明してないけど自明の理だよな?
もしあれだったら数学的帰納法で頑張って
http://imgur.com/3OwwL4R.jpg
0106132人目の素数さん
垢版 |
2018/03/20(火) 04:43:15.16ID:3dlcpbYb
mより大きな111...1[3]を、mに加えて三進法表示し、2→1、1→0、0→-1 とすればいいだけだろ
0107132人目の素数さん
垢版 |
2018/03/20(火) 05:04:38.68ID:HDkQdBLp
>>103 (右)

y=√x は上に凸だから
√k > ∫[k-1/2,k+1/2] √x dx,

(与式)> ∫[1/2,24+1/2] √x dx
 =[(2/3)x^(3/2)](x=1/2,49/2)
 =(2/3)(7^3 - 1)/(2√2)
 = 57√2
 = 80.610173

積分計算を避けたいなら、
AM-GM より
(kk -1/4)^3 ≧ kk・(kk -3/8)^2,

{(k+1/2)^(3/2) - (k-1/2)^(3/2)}^2 = 2k(kk +3/4) -2(kk -1/4)^(3/2)
 ≦ 2k(kk +3/4) -2k(kk -3/8)
 = 9k/4,

√k ≧ (2/3){(k+1/2)^(3/2) - (k-1/2)^(3/2)},
以下は同様。
0108132人目の素数さん
垢版 |
2018/03/20(火) 05:33:56.73ID:HDkQdBLp
>>103 (左)

y=√x は上に凸だから
{√k + √(k+1)}/2 < ∫[k,k+1] √x dx,

(与式) < 1 +√2 +√3 +(1/2)√4 + ∫[4,25] √x dx -(1/2)√25
 = 2 +√2 +√3 +[(2/3)x^(3/2)](x=4,25) - 5/2
 = 2 +√2 +√3 +(2/3)(125-8) -5/2
 = 80.6462644
0109132人目の素数さん
垢版 |
2018/03/20(火) 05:54:38.66ID:HDkQdBLp
〔問題〕
(2√6 + 5)/2 < ∫[24,25] √x dx,
を用いて
√6 < (485/6)/33 = 2.449494949…
を示せ。
0110132人目の素数さん
垢版 |
2018/03/20(火) 07:17:04.24ID:HDkQdBLp
〔応用問題〕
不等式
 {√k + √(k+1)}/2 < ∫[k,k+1] √x dx,   >>108
を用いて次を示せ。

(2) √2 < 99/70 = 1.41428571…    (k=8)
   √2 < 1393/985 = 1.41421320… (k=49)
   √2 < (19601/6)/2310 = 1.4142135642… (k=288)

(3) √3 < (1351/6)/130 = 1.73205128… (k=48)

(5) √5 < 2889/1292 = 2.236068111…  (k=80)

(6) √6 < (485/6)/33 = 2.4494949…  (k=24)

(7) √7 < 2024/765 = 2.645751634…  (k=63)

(10) √10 < 117/37 = 3.16216216…   (k=9)
   √10 < (27379/6)/1443 = 3.1622776622… (k=360)

(11) √11 < 3970/1197 = 3.316624895… (k=99)

(17) √17 < 268/65 = 4.123076923…   (k=16)

(37) √37 < 882/145 = 6.08275862…  (k=36)
0112132人目の素数さん
垢版 |
2018/03/20(火) 14:30:10.64ID:GpBOW+61
>>111
そんな難しい問題を解けるやつはこの板にいない
0114132人目の素数さん
垢版 |
2018/03/20(火) 14:41:21.16ID:E4ArtLi4
>>111
1枚目は行けそう
0115132人目の素数さん
垢版 |
2018/03/20(火) 20:01:34.88ID:GpBOW+61
>>114
それはそう
行けそうというか見ただけでいける
ただ2と3が難しい

(1)
|z||1-kw|=|w|
k|w-1/k|=|w|
1/kが表す点をAとする。wはOAをk:1に内分する点と外分する点をそれぞれ直径の両端とする円周上にある。

(2)
PQ=|z-w|=k|w|
また(1)より、
wの中心はk/(k+1)(k-1)、
半径は1/(k+1)|k-1|
したがって、
|w|の最大値
=k/(k+1)|k-1|+1/(k+1)|k-1|
=1/|k-1|
|w|の最小値
=|k/(k+1)|k-1|-1/(k+1)|k-1||
=1/(k+1)
以上より、
PQの最大値=k/|k-1|
PQの最小値=k/(k+1)
0116132人目の素数さん
垢版 |
2018/03/20(火) 23:21:38.16ID:eXRt6Wpn
大学学部レベル質問スレ 10単位目 https://rio2016.5ch.net/test/read.cgi/math/1519715377/647

nを正の整数、X={x_1,x_2,...,x_{2n+1}}を実数からなる(多重)集合とする。
Xから任意に1つの元を取り除いたとき、残った2n個の元を和の等しいn個ずつの
組に分けることができるならば、x_1=x_2=…=x_{2n+1} である。
0117132人目の素数さん
垢版 |
2018/03/21(水) 03:15:09.46ID:Y0EoMfqc
>>110
 kが平方数のときは、不等号が逆向きでござる。

(2) √2 > 1393/985 = 1.41421320… (k=49)

(10) √10 > 117/37 = 3.16216216…   (k=9)

(17) √17 > 268/65 = 4.123076923…   (k=16)

(37) √37 > 882/145 = 6.08275862…  (k=36)
0118132人目の素数さん
垢版 |
2018/03/21(水) 12:36:42.69ID:MWb2EIvX
>>116
Xが生成する加法群をYとおくと、Yは捩れなし有限生成アーベル群であるからZ^m(Zは整数全体からなる加法群、mはある非負整数)と同型。
したがって、x_1=…=x_{2n+1}でないならば、群準同型f:Y→ZであってfによるXの像f(X)が単元でないようなものが存在する。
x'_i=f(x_i-x_1) (i=1,…,2n+1)とおくと
X'={x'_1,x'_2,…,x'_{2n+1}} (⊂Z) も X と同様の性質を持つが、
S-x'_i (ただしS=x'_1+…+x'_{2n+1}) が全て偶数にならなければならないので、x'_iの偶奇は全て一致する。x'_1=0 は偶数であるから、他の全てのiについてもx'_i は偶数。
これより、 x''_i=(x'_i)/2 とおけば
X''={x''_1,x''_2,…,x''_{2n+1}} (⊂Z) もXと同様の性質を持つ。無限下降法よりx_1=…=x_{2n+1}でなければならない。
0120132人目の素数さん
垢版 |
2018/03/21(水) 23:16:51.36ID:NaAK8rgB
(a/p) を平方剰余記号とする。

(1) (123/769) の値を求めよ。
(2) (1234567/987654323) の値を求めよ。
(3) (a/p) の値を求めよ。ただし、a, pの値は以下とする。

a = 289589985200426886037189479736335834688462115581329068039
p = 579179970400853772074378959472671669376924231162658136139
0121132人目の素数さん
垢版 |
2018/03/22(木) 00:44:22.48ID:T9JdKZ5e
>>119
微分形式?平らじゃんR^n
0122132人目の素数さん
垢版 |
2018/03/22(木) 01:28:08.85ID:Qvak/x+C
>>111
2枚目は、分かスレ441-603,608-609 を参照

(1)
f(x) -(ax+b) =(1-a)x + log{1 + e^(-2x)}+ b,
∴ a = 1, b = - lim[x→∞]log{1 + e^(-2x)}= 0,

(2)
左 シュワルツ不等式で
 (x +1/2)・log(1 +1/x)= ∫[x,x+1] u du・∫[x,x+1]1/v dv >{∫[x,x+1] du}^2 = 1,

 GM-AM より
 1/x - 1/(x+1)= 1/(x(x+1))<{1/x + 1/(x+1)}/{2√(x(x+1))}= -{1/√(x(x+1))} '
 あるいは、√(x(x+1))- x は単調増加ゆえ
 1 <{√(x(x+1))} '
 1/x - 1/(x+1) = 1/(x(x+1))<{√(x(x+1))} '/(x(x+1))= -{1/√(x(x+1))} '
 x〜∞で積分して
 log{(x+1)/x}< 1/√(x(x+1)),

 なお、x → e^(2x)とすれば
 2e^(-2x)/{2 + e^(-2x)}< log{1 + e^(-2x)}< e^(-2x)/√{1 + e^(-2x)}

(3) e^x・dx = dθ/(cosθ)^2, より
 ∫[0,p]e^(-2x)/√{1 + e^(-2x)}dx = ∫[π/4,arctan(e^p)]1/(sinθ)^2・cosθdθ
 = [ -1/(sinθ)](θ:π/4〜arctan(e^p))
 = √2 - √{1 + e^(-2p)}
 → √2 - 1  (p→∞)

(4)
∫ 2e^(-2x)/{2 + e^(-2x)}dx = -log{2 + e^(-2x)},
∫[0,∞]2e^(-2x)/{2 + e^(-2x)}dx = log(3)- log(2)= 1.098612 - 0.693147 = 0.405465
S(∞)= ∫[0,∞]log{1 + e^(-2x)}dx = 0.4112335
√2 -1 = 0.41421356
0123132人目の素数さん
垢版 |
2018/03/22(木) 01:40:23.13ID:tjCH61Ex
>>120
(1) 320^2≡123 (mod 769) より 1
以下ヤコビ記号を使用する。すなわちbが奇数の合成数のときb=pb'なる素数pについて(a/b)=(a/p)(a/b')
(2) (1234567/987654323)=-(723/1234567)=(406/723)=-(203/723)=(114/203)=-(57/203)=-(32/57)=-1
(3) (a/p)=-(61/a)=-(57/61)=-(4/57)=-1
0124132人目の素数さん
垢版 |
2018/03/22(木) 01:41:25.13ID:Qvak/x+C
>>111
3枚目

∠ACB = θ とおく。
AC > AB > 0 より 0 < θ < π/2,

デカルト座標(x,y)を以下のようにとる。
A (0,0)
B (2 sinθ,0)     AB = 2 sinθ,
C (0,2 cosθ)     AC = 2 cosθ,
D (AD cosθ,AD sinθ)  AD = 2 AC sinθ = 2 sin(2θ),
E (2 sinθ,2 cosθ)  AE = BC = 2,
F (2 sinθ,2(sinθ)^2 /cosθ)
G (2 sinθ,1/cosθ)  FG = {1-2(sinθ)^2}/cosθ = cos(2θ)/cosθ,

直線AD: y = x tanθ,
直線BE: x = 2 sinθ,
直線CD: x/tan(2θ)+ y = AC = 2 cosθ,

以上により
△AFG = (1/2) AB FG = tanθ・cos(2θ)=(√T)(1-T)/(1+T),
ここに T =(tanθ)^2, (0<T<1)

φ =(1+√5)/2 = 1.618034 (黄金比と云う)を使うと
(5φ -8)(1+T)^2 - T(1-T)^2 =(φ-T)(T+3-2φ)^2 ≧ 0,
∴(△AFG)^2 = T(1-T)^2/(1+T)^2 ≦ 5φ -8 = 0.090170
∴ △AFG ≦ √(5φ -8)= 0.300283

等号成立は T = 2φ-3 = √5 -2 = 0.236068 のとき。
cos(2θ)= 1/φ =(√5 -1)/2 = 0.618034
θ = arctan(√T)= 0.452278447 (rad) = 25.91 (゚)
0125132人目の素数さん
垢版 |
2018/03/22(木) 09:47:30.38ID:vsUNKHqP
>>116
x_1からx_{2n+1}の中の最大値をM、最小値をmとする。
全ての元にTを加えた、X'={x_1+T,x_2+T,...,x_{2n+1}+T}という多重集合も、
「X'から任意に1つの元を取り除いたとき、残った2n個の元を和の等しいn個ずつの組に分ける」
ことができなければならない。

さて、X'において、ある元を除き、和が等しくなるようにn個ずつ分けた組の合計は、
下限がn*(m+T)、上限がn*(M+T)となるが、T >> M の様なケースを考えれば、下限、上限ともに、
n*Tが支配的な量になることから、X'の元の s 個の和 = X'の元の r 個の和 → s = r となる必要がある。

ところで、Tとして、(-1)*x_1 を考えると、(少なくとも)一つの元が0なので、
その元の加算は、和に影響を与えないので、左辺側にこの元が含まれると、
X'の元の n-1 個の和=X'の元の n 個の和 ;(左辺側にこの元が含まれる)
という事が起こる。この矛盾を回避するためには、「n 個の和」と思っていた物も、実質「n-1 個の和」
と等しければよく、これは、x_1と同じ値を持つ物が、右辺側にも含まれていることを意味する。
取り除く元としてx_1を選んだとき、どちらかのグループに、x_1と同じ値を持つ元が有るので、反対の
グループには、さらに、x_1と同じ値をもつ元がなければならない。
以下同様に、x_1と等しい元が、奇数個ある事が確認できている場合には、値不明の元を取り除く元として選び、
x_1と等しい元が、偶数個ある事が確認できている場合には、x_1と同じ値を持つ物を取り除く元として選べば、
順次、x_1と等しい新しい元の存在が確認でき、最終的に全ての元が、x_1と等しくなければならないことが示される。
0126132人目の素数さん
垢版 |
2018/03/22(木) 12:59:12.35ID:2lfOCr3y
次の条件を満たすn次正方行列の固有値を全て求めよ。

1≦m≦nを満たす全ての整数mについて第m行の行ベクトルは0が連続してn-m個並ぶその右に1/mがm個並んだものである。
0127132人目の素数さん
垢版 |
2018/03/22(木) 13:41:36.02ID:P81vFYvQ
矛盾してないから回避は不要。
0130132人目の素数さん
垢版 |
2018/03/23(金) 07:01:04.90ID:EuazrwzR
Le Veque の定理(1952)
 x-y = 1 のとき
 x^m - y^n = 1  …(1)
を満足する2以上の自然数解は x=3,y=2,m=2,n=3 に限る。

・カタラン予想に x-y=1 の制限を付加したもの。
・カタラン予想そのものは 2004年にミハイルスクにより証明された。

数セミ増刊 「数学 100の定理」 日本評論社(1984) p.104-105
数セミ増刊 「数学・物理 100の方程式」 日本評論社(1989) p.20-21
0132132人目の素数さん
垢版 |
2018/03/23(金) 07:07:00.80ID:EuazrwzR
(略証)
x = y+1 を (1) に入れると
 (y+1)^m - y^n = 1,
 ym +1 ≡ 1,  (mod yy)
 y|m   … (2)
また
y = x-1 を (1) に入れて
 x^m - (x-1)^n = 1,
 (-1)^n (nx-1) ≡ 1,  (mod xx)
 nは奇数 かつ x|n ∴ xも奇数 … (3)

(2)(3) より、yは偶数、mも偶数。

m=2r とおくと、
 x^m -1 = x^(2r) -1 = (x^r +1)(x^r -1),
右辺の2因子はともに偶数で、その差が2だから、
一方は 2×奇数、他方は 4の倍数。 …(4)
 y = (x-1)|(x^r -1) より
 gcd(x^r +1,y) = gcd(x^r +1,x-1) ≦ gdc(x^r +1,x^r -1) = 2,
x^r +1 が 奇素数pの倍数ならば yもpの倍数、gcd(x^r +1,y) も2pの倍数となり、矛盾する。
x^r +1 は2ベキである。
2^a = x^r +1 > x^r -1 ≧ x-1 = y ≧ 2,
a > 1,
(4) より
x^r -1 = 2×奇数,
y = 2K, (K:奇数) とおくと (1) より
(2K)^n = y^n = x^m -1 = (x^r +1)(x^r -1) = 2^(a+1)・K^n,
a = n-1,
2^(n-1) = x^r +1 > x^r -1 = 2・K^n,
2 > K,
K = 1,
y = 2K = 2,
x = y+1 = 3,
3^r -1 = 2K より r=1,
3^r +1 = 2^(n-1) より n=3. (終)

 H.B.Yu (1999) による。数セミ,38巻,6号(1999/June)
0135132人目の素数さん
垢版 |
2018/03/26(月) 21:19:56.09ID:IKnRwfdR
p、q、r を相異なる素数とするとき、[x/p] + [x/q] + [x/r] = x の実数解 x の個数を p、q、r を用いて表せ。
0136132人目の素数さん
垢版 |
2018/03/27(火) 05:37:52.70ID:H3+XdNyv
>>135

p<q<r とする。
(2, 3, 5) = 30,   {0,6,10,12,15,16,18,20,21,22,24,25,26,27,28,31,32,33,34,35,37,38,39,41,43,44,47,49,53,59}
(2, 3, 7) = 42,   {0,-6,-12,-14,-18,-20,-21,-24,-26,-27,-28,-30,-32,-33,-34,-35,-36,-38,-39,-40,-41,-44,-45,-46,-47,-49,-50,-51,-52,-53,-55,-57,-58,-59,-61,-64,-65,-67,-71,-73,-79,-85}
(2, 3, 11) = 14,  {0,-6,-8,-9,-10,-11,-12,-14,-15,-16,-17,-19,-23,-25}
(2, 3, 13) = 12,  {0,-6,-8,-9,-10,-11,-13,-14,-15,-16,-17,-19}
(2, 3, 17) = 8,   {0,-6,-8,-9,-10,-11,-13,-19}
(2, 3, r) = 7,  (19≦r)  {0,-6,-8,-9,-10,-11,-13}
(2, 5, 7) = 8,       {0,-4,-5,-6,-7,-8,-9,-11}
(2, 5, r) = 5,  (11≦r)  {0,-4,-5,-6,-7}
(2, q, r) = 3,  (7≦q<r) {0,-4,-5}
(3, q, r) = 3,  (5≦q<r) {0,-3,-4}
(p, q, r) = 2,  (5≦p<q<r){0,-3}
かなあ。
0137132人目の素数さん
垢版 |
2018/03/27(火) 23:45:01.17ID:Pc4avu0S
素数関係ないな。
0139132人目の素数さん
垢版 |
2018/03/28(水) 00:14:07.38ID:6Bea2jrG
>>4
アホくさ

(1)
(@)正三角形を含む場合
AB=BC=CA=yと置く
DA,DB,DCの内2つは等しい
DB=DCと置く
DはBCの垂直二等分線に在る
(@-1)DA=yの場合
DBCが頂角30度の二等辺三角形の形の答えと、凧型の答えを得る
(@-2)DB=y の場合(自動的にDC=y)
1つの候補はDとAが重なり、もう1つの候補から内角60度の菱形という1つの答えを得る
(i-3)何方でも無い場合 DA=DB=DC
DがABCの重心に在る場合という1つの答えを得る

(A)正三角形を含ま無い場合
AB=AC=x, BC=yと置く
DA,DB,DCのうち2つは等しい
(ii-1)DB=DCの場合
DB=DC=yだと正三角形ができるのでDB=DC=xの場合を考えれば良い
更にAD=xだとABDが正三角形なのでAD=yを考えれば良い
ABDCが正方形という1つの答えを得る
(ii-2)DA=DB の場合
同様にDA=DB=yの場合を考えれば良い
DC=yだとDBCが正三角形なのでDC=xを考えれば良い
辺の長さから△BAC≡△ACD
ACを底辺と扱うと点B,DのACからの距離は同じなのでBD//AC
故に4点は等脚台形を為す
対角線が長い方の平行辺と長さが等しい図という答えに至る

(2)
A,B,C,Dが与えられた時のEの候補は次の2つに分けられる
・EA=EB=EC の場合のような場合3点の外接円(4種類)
・EA=EB, EC=EDの場合2点と2点に分けて垂直二等分線の交点(3種類)
と考えて全部を検討するのが漏れなく其れなりに効率良さそうな1つの考え方だ
ABCDが正方形の時だけは対辺の垂直二等分線が一致し点が定まらず、更に長さを考えるか、或いは視点を変えてBCDEも又(1)の形を為すと要求すると不可能であると分かる

(3) BCDEFも(2)の形を為すと要求するとF=Aと成るしか無く不適
0140132人目の素数さん
垢版 |
2018/03/28(水) 04:54:36.75ID:v6aRBv4c
p≡1 (mod 4) のとき 1
p≡-1 (mod 4) のとき -1
これをまとめると、(-1)^{(p-1)/2}

p≡±1 (mod 8) のとき 1
p≡±3 (mod 8) のとき -1
これをまとめると、(-1)^{(p^2-1)/8}

----------------------------------------------
問題. (1)〜(4)のそれぞれについて、(-1)^x の形で表せ。

(1)
p≡1,3 (mod 8) のとき 1
p≡-1,-3 (mod 8) のとき -1

(2)
p≡±1 (mod 5) のとき 1
p≡±2 (mod 5) のとき -1

(3)
p≡1,3,7,9 (mod 20) のとき 1
p≡-1,-3,-7,-9 (mod 20) のとき -1

(4)
p≡±1 (mod 12) のとき 1
p≡±5 (mod 12) のとき -1
0141132人目の素数さん
垢版 |
2018/03/28(水) 23:36:57.84ID:rYPiNLPi
そろそろ>>62の正解

初等幾何の諸定理より(リンク先参照)、n=kのときの明るさはn=k-1のときと等しく、
後ろ向きの帰納法を用いると、任意のnのときの明るさは(π^2)/4である。

また、無限に大きい円の場合、観測者が受ける光の明るさは、「数直線上の原点にいる観測者が、…,-5,-3,-1,1,3,5,…の点にある光源から受ける光の明るさα」と同等である。
よって
α = 2Σ[t=1,∞] 1/(2t-1)^2 = (π^2)/4
すなわち奇数の二乗の逆数和は(π^2)/8に収束することが導ける。

更に、「数直線上の原点にいる観測者が、…,-6,-4,-2,2,4,6,…の点にある光源から受ける光の明るさβ」は、逆二乗則より「数直線上の原点にいる観測者が、…,-3,-2,-1,1,2,3,…の点にある光源から受ける光の明るさγ」の1/4になるになるはずである。
γ=α+β=(π^2)/4+(1/4)γよりγ=(π^2)/3, β=(π^2)/12
よって
β = 2Σ[t=1,∞] 1/(2t)^2 = (π^2)/12
γ = 2Σ[t=1,∞] 1/(t^2) = (π^2)/3
すなわち
偶数の二乗の逆数和は(π^2)/24に収束し、
自然数の二乗の逆数和は(π^2)/6に収束する(バーゼル問題)。
0142132人目の素数さん
垢版 |
2018/03/28(水) 23:37:59.71ID:rYPiNLPi
物理学で対応する事象を用いたバーゼル問題の初等的・幾何的・直感的な証明は今世紀に入ってから発表されたものである。

論文
http://www.math.chalmers.se/~wastlund/Cosmic.pdf
動画
http://youtu.be/d-o3eB9sfls
0143132人目の素数さん
垢版 |
2018/03/29(木) 11:59:27.65ID:ihUI7uvJ
a,bを自然数とする。a^2+b^2をa+bで割った商をq、余りをrとすると、q^2+r=1977が成り立つという。
(a,b)を全て求めよ。
(もちろんq,rは非負整数でありr<a+b)

ヒント:r<2qを示せて、q,rが確定する。
0144132人目の素数さん
垢版 |
2018/03/29(木) 12:48:02.60ID:MHic9gzf
>>143

qq+r = 1977,r<2q から q=44, r=41 が確定する。

aa+bb = 44(a+b) +41,a+b>r=41 から{a,b}={7,50}{37,50}
0145132人目の素数さん
垢版 |
2018/03/29(木) 17:31:24.50ID:ihUI7uvJ
>>144
解答は合ってるけどさすがにダメ
0147132人目の素数さん
垢版 |
2018/03/29(木) 22:45:57.64ID:/OSBVUz8
>>4
>>139
許される距離がm種類だったり、空間にしてみたり拡張を考えたくなる
できるかは別として
0148132人目の素数さん
垢版 |
2018/03/29(木) 23:54:31.17ID:ihUI7uvJ
>>143

(a+b)q+r=a^2+b^2≧(a+b)(a+b)/2よりq≧(a+b)/2-r/(a+b)≧(a+b)/2
∴2q≧a+b>r
q^2+r=1977で2q>rを満たすのは(q,r)=(44,41)のみである。
このときa^2+b^2=44(a+b)+41⇔(a-22)^2+(b-22)^2=1009
1009は2平方数の和では(±15)^2+(±28)^2, (±28)^2+(±15)^2とのみ表されるから
(a-22,b-22)=(15,28),(-15,28),(28,15),(28,-15) (∵a-22≧-21, b-22≧-21)
よって(a,b)=(37,50),(7,50),(50,37),(50,7)

一昔前(1977年)の数オリだけど、難問揃いの近年では考えられないくらい簡単
0149132人目の素数さん
垢版 |
2018/03/30(金) 00:07:07.17ID:Bx07PAfT
簡単と言いながら間違える。
0150132人目の素数さん
垢版 |
2018/03/30(金) 16:45:22.96ID:9jey3GD7
>>55
(1) E.T. the Extra-Terrestrial 『E.T.』
(2) The Matrix 『マトリックス』
(3) Velocity
(4) Leaving Las Vegas 『リービング・ラスベガス』
(5) La La Land 『ラ・ラ・ランド』
(6) 12 Monkeys 『12モンキーズ』
(7) Pi 『π』
(8) Dr. No 『007 ドクター・ノオ』
(9) Seven 『セブン』
(10) Home Alone 『ホーム・アローン』
(11) The Green Mile 『グリーンマイル』
(12) The Lord of the Rings: The Fellowship of the Ring 『ロード・オブ・ザ・リング』
(13) Catch Me If You Can 『キャッチ・ミー・イフ・ユー・キャン』
(14) Gravity 『ゼロ・グラビティ』
(15) All the Money in the World 『ゲティ家の身代金』
(16) The Da Vinci Code 『ダ・ヴィンチ・コード』
(17) 2001: A Space Odyssey 『2001年宇宙の旅』
(18) Dial M for Murder 『ダイヤルMを廻せ!』
(19) Signs 『サイン』
(20) 8 Mile 『8 Mile』
有力な別解
(3) Speed 『スピード』
reddit.com/r/math/comments/815ojr
0151132人目の素数さん
垢版 |
2018/04/01(日) 00:24:31.04ID:VCG34iJE
(0,1),(1,1)を結ぶ曲線のx軸周りの回転体の表面積の最小値を求めよ.
0152132人目の素数さん
垢版 |
2018/04/01(日) 00:28:40.25ID:VCG34iJE
>>151
ごめんなさいミスです
(0,0),(1,1)を結ぶ曲線です
0154132人目の素数さん
垢版 |
2018/04/01(日) 01:39:38.11ID:VCG34iJE
>>153
違います
0156132人目の素数さん
垢版 |
2018/04/01(日) 01:41:16.70ID:VCG34iJE
>>155
名前は付いてるよ
0158132人目の素数さん
垢版 |
2018/04/01(日) 01:42:46.67ID:VCG34iJE
>>157
違うかな
日常でも良く現れる曲線です
0159132人目の素数さん
垢版 |
2018/04/01(日) 01:43:08.83ID:noFB9/4S
ごめんな解くのがめんどいんだわ
解くのが面白い問題じゃないだろうし、ひたすら計算って、問題としてはつまんねーし
0161132人目の素数さん
垢版 |
2018/04/01(日) 01:44:58.65ID:VCG34iJE
>>159
まあ計算ゲーではあるけども
厳密にそれが最小解であることを証明するのはかなり高度な抽象論必要だし面白いと思う
>>160
そうです
0163イナ ◆/7jUdUKiSM
垢版 |
2018/04/01(日) 17:17:42.23ID:+Kemoei8
_人人_/_/_/_/_/
(_^_)_/_/_/_/_/
_((-_-)_/_/_∩∩_/
_(っц)~/_/_(^) )_/
_(`γ)_/_/_,U⌒ヽ_/
_υυ_/_/(___)/_/_/_/_/_/_/UU/_/_/_/_/_/_/_/_/_/_/_/_/_/_/_/_/_/_/_/_/_/_/_/_/要はろくろだろ。扇形なら小さくなるし、放物線なら大きくなるし、指数関数にすればもっと大きくなるんじゃない?
0164132人目の素数さん
垢版 |
2018/04/01(日) 17:55:01.26ID:kw1PD5xS
101頭の牛がいてどの牛も体重は整数sである
どの1頭を除いても残りの100頭を総体重が等しい50頭ずつのグループに分けることができる
このとき全ての牛の体重は同じであることを示せ
0165132人目の素数さん
垢版 |
2018/04/01(日) 23:17:44.11ID:Sq5gTv4H
3人の女性A,B,Cがいる。
この3人は、
100%本当のことを言う正直者
50%の確率で本当のことを言う気まぐれ
0%の確率で本当のことを言う嘘つき
が一人ずつであるが、あなたは誰がどれに対応するかはわからない。
女性間では誰がどれに対応するかわかっている。

あなたは彼女らに「はい」、「いいえ」で答えられる質問を2回行う。
2回目の質問で「はい」と答えさせることができればあなたの勝ちである。
2回の質問をどう行うと良いか?

ただし、各質問は一人ずつにしか行えない。
0166132人目の素数さん
垢版 |
2018/04/02(月) 00:56:48.50ID:66IqDDyK
1回目:Aさんに質問
「もしあなたに『Bさんは気まぐれですか』と尋ねたら『はい』と答えますか」
2回目:1回目の答えが「はい」の場合はCさんに、「いいえ」の場合はBさんに質問
「あなたは正直者ですか」

1回目の質問で「少なくとも気まぐれではない1人」を探すのがポイント。
気まぐれでさえないことがわかっていれば、事実を聞き出したり特定の答えに誘導するのは簡単。
0167132人目の素数さん
垢版 |
2018/04/02(月) 01:21:42.33ID:ZjjiJzGw
>>166
お見事。
論理の2回反転で嘘つきを正直者にする解法ですね。
エイプリルフールなので出してみました
あ、エイプリルフールが終わってしまったようですw
0168132人目の素数さん
垢版 |
2018/04/02(月) 10:00:45.81ID:rDlRBZ4q
>>164
整数kgの101頭の牛に於いて同じ体重であるもの同士を同じグループとして分類せよ
全ての牛の体重が同じであることは無いとすれば2グループ以上に分類できる筈である
n(n≧2)グループに分類されたとせよ
其々のグループの牛の体重を
A[i]kg(i=1,2,3,…,n)とせよ
則ちA[n]>A[n-1]>…>A[2]>A[1]の大小関係が従う
D[i]=A[i+1]−A[i](i=1,2,3,…,n−1)とせよ
D[i]の最小値をmとし、其の時のi(かつiの中でも最小であるもの)をpとせよ
m|{D[i]|i=1,2,...n-1}
今グループA[p]の牛の1頭Xを除いて100頭の牛が総体重が等しい50頭ずつのαグループとβグループに分かれていたとせよ
此処でXの代わりにグループA[p+1]の牛の1頭Yと入れ替え、Yを除く100頭の牛の牛を総体重が等しい50頭ずつのグループに分ける操作を考えよ

Yを除外する前にYはαグループに存在していたとせよ。単純にYとXを交換しただけなれば、則ちグループαの総体重がmだけ減る

2つのグループの総体重を均衡させるにはグループαの総体重をm/2kg増やし、グループβの総体重をm/2kg減らすことが必要…★

αとβグループで牛を交換する操作で此れを行う必要があるが、A[1],A[2],…,A[n]のグループ間の体重差はmの整数倍, 則ちαグループ、βグループ間でいくら牛を交換した所で★は達成され得無い

故に全ての牛の体重は同じである
0169132人目の素数さん
垢版 |
2018/04/02(月) 10:30:56.56ID:qydp8iS9
IMO系統の問題だね
0170132人目の素数さん
垢版 |
2018/04/02(月) 19:51:00.50ID:UtRAneS5
>>168
A[1],A[2],…,A[n]のグループ間の体重差はmの整数倍というのが何故言えるのかが分からないです
0171132人目の素数さん
垢版 |
2018/04/02(月) 22:04:17.89ID:ZjjiJzGw
>>170
確かにmの整数倍で無い
mより大きな体重差の牛を入れ替えてm/2kgの体重差を±し均衡させることは不可能という流れだろう
0172132人目の素数さん
垢版 |
2018/04/02(月) 22:30:49.48ID:ZjjiJzGw
>>168
いや、論理が破綻していた様だ
0173132人目の素数さん
垢版 |
2018/04/04(水) 00:46:02.74ID:f//H+LBj
>>164
ちょっとだけ一般化。
問題
2n+1枚のカードが有り、全てに正整数が書かれていていて
どの一枚を除いても、残り2n枚を、和が等しいn枚ずつに分けることができるとする。
この時、全てのカードには同じ正整数が書かれていることを示せ。

解答
2n+1枚の和が奇数の時、和の合計から、偶数が書かれているカードがあるとすれば、
偶数枚でないといけないが、取り除くカードとして偶数のカードを選んだとき
「どの一枚を除いても、残り2n枚を、和が等しいn枚ずつに分けることができるとする。」
ができないから、和が奇数の時は、全てのカードは奇数で無ければならない。

2n+1枚の和が偶数の時も同様の理由から、全てのカードは偶数でなければならないことがわかる。

和が正で、奇数の時は、全てのカードから1を減じ、偶数の時は、2で割る。この操作を繰り返しても、
「どの一枚を除いても、残り2n枚を、和が等しいn枚ずつに分けることができるとする。」という性質は
維持される。1を減じるか、2で割る、という操作を繰り返すと、いつかは必ず、0に到達する。
これは、最初に書かれていた正整数が、全て等しかったことを意味する。
0174132人目の素数さん
垢版 |
2018/04/04(水) 01:18:28.03ID:EmPoqxOk
>>152

曲線の式を y=f(x) とする。
曲面の表面積は S[f] = ∫[0,1] 2πf(x) √{1 + [f'(x)]^2} dx,
これは
L[f,f'] = 2πf(x)・√{1 + [f '(x)]^2},
を Lagrangian とする変分問題。

S[f] = ∫[0,1] L[f,f '] dx
を f(x) で変分すると、
δS[f] = ∫[0,1] δL dx
= ∫[0,1] {(∂L/∂f)δf +(∂L/∂f')δf'}dx
= ∫[0,1] {(∂L/∂f)-(d/dx)(∂L/∂f')}δf dx + [ (∂L/∂f')δf ](x=0,1)
 ↑ 部分積分した。
f(0) と f(1) が固定されていて δf= 0(x=0,x=1)のときは右辺第2項は0

任意の変分 δf に対して 右辺第1項が0となることから、

(∂L/∂f)-(d/dx)(∂L/∂f ') = 0,  … Euler-Lagrange方程式

本問では
 f(x)f "(x) - {f '(x)}^2 = 1,
により、懸垂曲面(カテナリー)
0177132人目の素数さん
垢版 |
2018/04/04(水) 07:19:54.51ID:xFWQXFxC
>>174
その微分方程式の一般解はf(x)=Acosh((x+B)/A)になると思うけどどんなA,Bに対しても(0,0)は通らなくね?
0178132人目の素数さん
垢版 |
2018/04/04(水) 09:11:37.84ID:E749QQfH
(0,0)-(1,0)-(1,1).
最小値π。
0179132人目の素数さん
垢版 |
2018/04/04(水) 11:07:27.37ID:vqWKdTt9
>>173
素晴らしいです!
この問題が載ってた本の解答では、最軽量の牛の体重を全ての牛から引いて体重0sの牛1頭と100頭の牛にするという手法でした
0180132人目の素数さん
垢版 |
2018/04/04(水) 12:21:21.04ID:DuTnz6IW
>>165
誰にでもいいから2回目に「あなたはこの質問に正直に答えますか」で良くないか?
0181132人目の素数さん
垢版 |
2018/04/04(水) 19:20:46.19ID:EqC9nuEi
>>8
近大数コン問題2つの解説
競争に参加するには去年から事前申し込みが必要になった

[24-437]
2005年A4
http://imgur.com/Fl4qnjr.jpg

[23-937,24-30]
2009年A6
http://imgur.com/dxasE4H.jpg

本は『白熱!無差別級数学バトル』
競技数学、趣味数学の本として面白いので買おう(ダイマ)
0185132人目の素数さん
垢版 |
2018/04/05(木) 13:20:46.75ID:HpOHoLwn
>>140
これ、x を p の有理数係数多項式で表す問題だと思ってたんだけど
それでは(2)が不可能であることが証明できてしまった。

(ちなみに(1)は、上の例をずらして x=((p-2)^2-1)/8 でできる。)

以下、分母が奇数であるような分数として表せる有理数全体の集合を U とし、
2U={ 2u | u ∈ U } とする。
すなわち、2U は分母が奇数、分子が偶数であるような分数として表せる有理数全体の集合である。

[補題]
f(x) を有理数係数多項式とすると、十分大きい正整数 k が存在して、
任意の整数 n に対し f(n+2^k)-f(n) ∈ 2U が成り立つ。

[証明]
f(x) が単項式の場合:
f(x)=ax^d とおく。
a*2^k ∈ 2U となるような正整数 k をとる。
すると、
 f(n+2^k)-f(n) = a{(n+2^k)^d-n^d} = a*2^k*(整数) ∈ 2U
となる。

f(x) が一般の多項式の場合:
各項に対して上のような k をとり、その最大値をとればよい。□

[命題]
x が p の有理数係数多項式であるとき、>>140の(2)は成り立たない。

[証明]
f(p) を p の有理数係数多項式とし、
 p≡±1 (mod 5) のとき (-1)^f(p)=1
 p≡±2 (mod 5) のとき (-1)^f(p)=-1
が成り立つと仮定する。

f(p) に対し、補題のように k をとる。
 5a + 2^k*b = 1
を満たすように整数 a,b をとる。すると
 5a + 1 + 2^k*b = 2
である。補題より、
 f(2) - f(5a + 1) = f(5a + 1 + 2^k*b) - f(5a + 1) ∈ 2U
である。一方、仮定より f(2) は奇数、f(5a + 1) は偶数であるから、
f(2) - f(5a + 1) は奇数であり、f(2) - f(5a + 1) ∈ 2U に反する。 □
0186132人目の素数さん
垢版 |
2018/04/05(木) 13:21:12.21ID:HpOHoLwn
で、有理数係数多項式以外で何かしら綺麗に表す方法がないか探した結果、
一応次のようなものがあった。
 x=(cos(2pπ/5)-cos(2π/5))/(cos(4π/5)-cos(2π/5))

ただ、これを許してしまうと(3),(4)も三角関数と多項式補完の組み合わせですぐにできてしまうので
なんだかなあという感じ。
0187132人目の素数さん
垢版 |
2018/04/05(木) 23:21:09.59ID:DTitQ5x8
>>184

x^4 ≡ 7 ≡ 7 + 19*126 = 2401 = 7^4 (mod 19)


(x+7)(x-7)(xx+49) ≡ 0 (mod 19)

-49 ≡ 8 は平方非剰余なので
x ≡ ±7 (mod 19)
0188132人目の素数さん
垢版 |
2018/04/06(金) 22:34:30.05ID:hYTmrE4N
一辺1の正n角形の各辺(頂点除く)に1点ずつとって作ったn角形の周長をl(n)とする。
3/2≦l(3)  (Fagnanoの問題の特別な場合)
2√2≦l(4) [『美しい不等式の世界』 演2.59]
3√3≦l(6) [『美しい不等式の世界』 演2.60]
を示せ。
0189132人目の素数さん
垢版 |
2018/04/07(土) 11:33:38.40ID:ozKr5R4w
>>188

正n角形の頂点をA_i、辺A_i A_{i+1} 上にとった点をB_i とする。(i=1,2,…,n)
∠A_i = π - 2π/n,

B_{i-1}A_i = x,A_i B_i = y とおくと、
第2余弦定理より
(B_{i-1}B_i)^2 = xx + yy +2cos(2π/n)xy
 = {cos(π/n)・(x+y)}^2 + {sin(π/n)・(x-y)}^2
 ≧{cos(π/n)・(x+y)}^2,
(x+y)cos(π/n)≦ B_{i-1}B_i ≦ x+y,
1周にわたって和をとれば
 n cos(π/n)≦ I(n) ≦ n,

・別解
 参考書のp.189の図に示されているように、辺に関する鏡映を使う。

・参考書
 佐藤淳郎(訳)『美しい不等式の世界』朝倉書店(2013)
0190132人目の素数さん
垢版 |
2018/04/07(土) 11:47:52.45ID:ozKr5R4w
>>188

等号成立条件(左側)は x=y より
 nが奇数のとき … B_i は A_i A_{i+1}の中点
 nが偶数のとき … 互い違いに並ぶ
0192132人目の素数さん
垢版 |
2018/04/08(日) 05:24:00.36ID:EiOPZE4m
(1) p≡1 (mod 4) をみたす素数pに対して、gがpの原始根ならば、-gもpの原始根であることを示せ。
(2) p≡1 (mod 4) をみたす素数pに対して、2はpの原始根であることを示せ。
(3) Σ[k=1 to 2001] k^(2001) を13で割った余りを求めよ。
0203132人目の素数さん
垢版 |
2018/04/09(月) 22:05:46.90ID:3IHyrdU+
>>192の続き
(4) 2^n + n^2 (n∈N、n≧2)が素数ならば、n≡3 (mod 6) を示せ。
(5) x≡1 (mod 24)、ab=x (a、b∈N) のとき、24 | a+b を示せ。
(6) m^2 + n、m^2 - n (m、n∈N) がともに平方数ならば、24 | m を示せ。
(7) 1111^6666 + 2222^5555 + 3333^4444 + 4444^3333 + 5555^2222 + 6666^1111 を7で割った余りを求めよ。
0204132人目の素数さん
垢版 |
2018/04/09(月) 22:22:00.84ID:uw9d+xOY
24|(a-b).
0205132人目の素数さん
垢版 |
2018/04/10(火) 10:55:04.05ID:Hhk3lh1l
24|n.
0206132人目の素数さん
垢版 |
2018/04/11(水) 01:14:00.27ID:ixEOJ+I8
>>192

(1)
 フェルマーの小定理 g^(p-1)≡ 1(mod p)より g^{(p-1)/2}= ±1(mod p)
 gがFpの原始根 ⇔ g^{(p-1)/2}≠ 1(mod p)⇔ g^{(p-1)/2}≡ -1(mod p)
 題意より p≡1 (mod4),(-1)^{(p-1)/2}= 1 だから、(-g)^{(p-1)/2}= g^{(p-1)/2}

(2) p ≡ ±1 (mod 8)のとき2は平方剰余だからFpの原始根でない。しかし
  p ≡ ±3(mod 8)のとき2は平方非剰余だがFpの原始根とは限らない。{2^14≡1(mod 43)}

(3)
 k^2001 = k^(12*166)・k^9 ≡ k^9 (mod 13)
 Σ[k=0,13-1] k^9 = 0^9 + Σ[k=1,6] {k^9 + (13-k)^9} ≡ Σ[k=1,6] (k^9 - k^9) = 0 (mod 13)
 Σ[k=0,2001] k^9 = Σ[k=0,13*154-1] k^9 ≡ 0 (mod 13)
0207132人目の素数さん
垢版 |
2018/04/11(水) 02:11:52.51ID:qdN2rXjI
偶数の逆数 の 偶数個の和 で1を表すのは可能(1/2+1/2=1)
偶数の逆数 の 奇数個の和 で1を表すのは可能(1/2+1/4+1/4=1)
相異なる偶数の逆数 の 偶数個の和 で1を表すのは不可能(∵1未満になる)
相異なる偶数の逆数 の 奇数個の和 で1を表すのは不可能(∵同上)

奇数の逆数 の 偶数個の和 で1を表すのは【A】(∵【B】)
奇数の逆数 の 奇数個の和 で1を表すのは可能(1/3+1/3+1/3=1)
相異なる奇数の逆数 の 偶数個の和 で1を表すのは【A】(∵【B】)
相異なる奇数の逆数 の 奇数個の和 で1を表すのは【C】(【D】)
0208132人目の素数さん
垢版 |
2018/04/11(水) 02:19:04.46ID:ixEOJ+I8
>>203

(4) 2^n は偶数だから nは奇数に限る。
  n = 6m±1 のとき、2^n + n^2 ≡ 8 + 1 = 9 (mod 12) ゆえ3の倍数。
  ∴ n = 6m+3.

(5) ab ≡ 1 (mod 24) より、a,b は正則元(24と互いに素)
 正則元{±1,±5,±7,±11}は位数がすべて2
  aa ≡ bb ≡ 1 (mod 24)
  24|(a-b)
 
(7) 
 1111^6666 ≡ 1111^0 = 1,
 2222^5555 ≡ 3^5 ≡ -2,
 3333^4444 ≡ 1^4 = 1,
 4444^3333 ≡ (-1)^3 = -1,
 5555^2222 ≡ (-3)^2 ≡ 2,
 6666^1111 ≡ 2^1 = 2  (mod 7)
より、3
0211132人目の素数さん
垢版 |
2018/04/12(木) 10:08:16.60ID:TgaFEakF
>>8 >>181

〔類題〕
3^k + 4^l + 5^m = 6^n を満たす非負整数の組 (k,l,m,n) をすべて求めよ。

(3,3,3,3) (3,1,1,2) (0,1,0,1)
0212132人目の素数さん
垢版 |
2018/04/12(木) 15:16:06.76ID:TgaFEakF
>>181 の続き

Aが大阪市 の場合
θ_A = 35.69゚N
φ_A = 135.50゚E

大円Oの法線nは、
経度φ_A = 135.50゚E、南緯54.31゚S の海面を向く。

大円Oの方程式は n・r = 0,

経度φの経線上では
|φ-φ_A| ≦ 90゚ のとき 北緯 Arctan(γcos(φ-φ_A)) N
|φ-φ_A| ≧ 90゚ のとき 南緯 Arctan(-γcos(φ-φ_A)) S
をとおる。ここに、γ = tan(θ_A) = tan(35.69゚) = 0.7184
0213132人目の素数さん
垢版 |
2018/04/12(木) 15:23:48.18ID:TgaFEakF
>>212 修正

Aが大阪市 の場合
θ_A = 34.69゚N

大円Oの法線nは、… 南緯55.31゚S の海面を向く。

ここに、γ = tan(θ_A) = tan(34.69゚) = 0.69225
0214132人目の素数さん
垢版 |
2018/04/12(木) 15:47:43.51ID:FlEPV0Tu
>>207
1/2+1/4+1/6+1/12=1
1/2+1/4+1/6+1/18+1/36=1

A:不可能、B:分母を払えば左辺が偶数、右辺が奇数となり矛盾するため。
C:可能。D:1/1=1. もしくは 1=(1/3+1/5+1/7+1/9+1/15+1/21+1/105)(1/1+1/11)+1/385+1/495+1/693.
0215132人目の素数さん
垢版 |
2018/04/12(木) 17:15:57.35ID:Mo9lTPQZ
>>214
確かに
相異なる偶数の逆数 の 偶数個の和 で1を表す
相異なる偶数の逆数 の 奇数個の和 で1を表す
は両方可能ですね…
なぜか2のべきの逆数を考えていました

A,B,C,D正解
Dは9個の和である
1/3+1/5+1/7+1/9+1/11+1/15+1/35+1/45+1/231=1
を用意していた
0226132人目の素数さん
垢版 |
2018/04/14(土) 23:00:50.00ID:EFpCaC7z
奇数の完全数Xは平方数にはならないので、すべての約数の個数が偶数個になる。だから、X自身以外の約数の和は完全数だからXであり、それをXで割れば、各項は分子が1で分母が奇数の和であり、合計は1になる。つまり、この奇数個の相異なる奇数の逆数の和は1になる。
だから、奇数個の相異なる奇数で逆数の和が1になるものを見つけたければ奇数の完全数を見つけてくれば簡単に求まる。
楽勝だな。
0227132人目の素数さん
垢版 |
2018/04/15(日) 04:31:16.64ID:fBnHdB0x
今この板で話題のネタはNG
0229132人目の素数さん
垢版 |
2018/04/15(日) 11:22:36.90ID:LGgAg+xm
Σ[n=1〜∞] 1/(n^2 -n -1) の値を求めよ
0230132人目の素数さん
垢版 |
2018/04/15(日) 14:33:10.20ID:MMDE1Y6Y
>>229
n^2-n-1はn=1/2で対称なのでΣ[n=1,∞]1/(n^2-n-1)=(1/2)Σ[n=-∞,∞]1/(n^2-n-1)

f(z)=πcot(πz)/(z^2-z-1)と置いて
(1-i)(N+1/2),(1+i)(N+1/2),(-1+i)(N+1/2),(-1-i)(N+1/2)を頂点とする正方形の周囲を
反時計回りに回る積分∫[C]を考えると、留数定理より
∫[C]f(z)dz = Res[z=(1-√5)/2]f(z)+Res[z=(1+√5)/2]f(z)+Σ[n=-N,N]Res[z=(1-√5)/2]f(z)
=-πcot(π(1-√5)/2)/√5+πcot(π(1+√5)/2)/√5+Σ[n=-N,N]1/(n^2-n-1)
=-2πtan(π√5/2)/√5+Σ[n=-N,N]1/(n^2-n-1)

ここでN→∞とすると、C上で|f(z)|=O(1/N^2)だから|∫[C]f(z)dz|→0
したがって
Σ[n=1,∞]1/(n^2-n-1)=πtan(π√5/2)/√5
0231132人目の素数さん
垢版 |
2018/04/15(日) 15:26:52.87ID:ZO3/JPf/
>>229
 a>0 として
Σ[n=1,∞] 1/{nn-n+(1/4-aa)}
= Σ[n=1,∞] 1/{(n-1/2-a)(n-1/2+a)}
= (1/2a)Σ[n=1,∞] {1/(n-1/2-a) - 1/(n-1/2+a)}
= (π/2a)tan(πa),
0232132人目の素数さん
垢版 |
2018/04/15(日) 21:21:21.49ID:ZO3/JPf/
>>231 (蛇足)

 S(a) = Σ[m∈Z] 1/(m -k/2 -a) {|m| の小さい順にたす}
 = (π/2a)tan(πa)  (a>0)
 = ππ/2   (a=0)


a>0,k∈Z として
Σ[n=1,∞] 1/{nn-kn+(kk/4-aa)}
= Σ[n=1,∞] 1/{(n-k/2-a)(n-k/2+a)}
= (1/2a)Σ[n=1,∞] {1/(n-k/2-a) - 1/(n-k/2+a)},

k=1 のとき
= S(a),

k<1 のとき
= S(a) - Σ[m=0,|k|] 1/(m+k/2-a),

k>1 のとき
= S(a) + Σ[m=1,k-1] 1/(m-k/2-a),
0233132人目の素数さん
垢版 |
2018/04/16(月) 04:41:29.91ID:0tJfbhfE
ストローに穴はいくつある?
0?1つ?2つ?
0234132人目の素数さん
垢版 |
2018/04/16(月) 15:52:07.44ID:Cr9cwYX2
>>203
> (5) x≡1 (mod 24)、ab=x (a、b∈N) のとき、24 | a+b を示せ。

>>208
> (5) ab ≡ 1 (mod 24) より、a,b は正則元(24と互いに素)
>  正則元{±1,±5,±7,±11}は位数がすべて2
>   aa ≡ bb ≡ 1 (mod 24)
>   24|(a-b)

解答の3行目から、いきなり4行目の結論が出せるん?
3行目から、(a+b)(a-b) ≡ 0 (mod 24) が得られて、
そこから a-b ≡ 0 (mod 24) って言えるの?
法24に対して零因子になっていることはないのかな?
0235132人目の素数さん
垢版 |
2018/04/16(月) 16:00:26.90ID:gRqM/Sq4
>>234
そもそも問題間違ってない?
a=b=x=1のとき
x≡1 (mod 24)、ab=x (a、b∈N)
だけど
24 | a+b
にならん希ガス
0236132人目の素数さん
垢版 |
2018/04/16(月) 21:05:56.06ID:Cr9cwYX2
n∈N に対して、θ= {(n-1)!+1}π/n とおく。大括弧 [・] はガウス記号とする。
(n-2)*[(cosθ)^2] + 2 の値を求めよ。
0237132人目の素数さん
垢版 |
2018/04/16(月) 22:09:49.08ID:I9VNB52o
>>234

 ab ≡ 1 (mod 24)
だから
 b ≡ (ab)b ≡ a(bb) ≡ a (mod 24)
 a ≡ a(ab) ≡ (aa)b ≡ b (mod 24)


>>236
 n:素数のとき n,
 n:合成数のとき n - (n-2)sin(π/n)^2
0239132人目の素数さん
垢版 |
2018/04/16(月) 22:58:14.46ID:pituM4NW
>>236は結局(n-1)!+1がnの倍数になるのはいつか聞いてるだけやね。Wilsonの定理ですな。
0240132人目の素数さん
垢版 |
2018/04/16(月) 23:07:50.97ID:Cr9cwYX2
(n-1)!+1がnの倍数でないときは、なんで [(cosθ)^2]=1 になるのが分からんぷー
0242132人目の素数さん
垢版 |
2018/04/16(月) 23:44:20.37ID:c/5dDqUx
半径r(>1)の円の周に中心をもつ半径1の円があるときこの2円の中心距離をrから少しずつ近づけていったときd(r)縮めたときに初めて2円の共通部分の面積が半径1の円の半分になったものとしてd(r)を定める。rd(r)の極限を求めよ。
なる問題を考えたのですがこれはsinxの3次マクローリン展開による不等式を用いれば1/6と分かりました。
この問題はrとd(r)のみの多項式による関係式が得られないことから角度を置くなどすることが難しい問題なのですが、この問題を球体で考えたらどうだろうかと思いまして、しかしすぐに球体なら簡単にπ(1-x^2)の積分でrとd(r)の関係が得られるではないかと考えました。
しかし実際には計算がかなり煩雑になってしまいました。どなたか解決してくださりませんか。この場合rの何乗のオーダーかも分かりません。
0243132人目の素数さん
垢版 |
2018/04/17(火) 00:23:18.43ID:OpQZlM6R
>>242
r→∞っすか?
0244132人目の素数さん
垢版 |
2018/04/17(火) 01:27:06.21ID:/l7sQR/P
1/4っぽい?
0245132人目の素数さん
垢版 |
2018/04/17(火) 02:44:39.60ID:+pEnOXwO
>>203
元ネタを見つけた。
数学発想ゼミナール1 問3.2.16(c)(d)、第24回シュプリンガー数学コンテスト

(5)の仮定はx≡-1 (mod 24)、(6)の結論は24 | n に訂正。

(5) x≡-1 (mod 24)、ab=x (a、b∈N) のとき、24 | a+b を示せ。
(6) m^2 + n、m^2 - n (m、n∈N) がともに平方数ならば、24 | n を示せ。
0246132人目の素数さん
垢版 |
2018/04/17(火) 03:31:36.63ID:JZUi2LJv
1998年3月号出題の数セミの問題持ってる人おる?
0247132人目の素数さん
垢版 |
2018/04/17(火) 03:32:06.84ID:JZUi2LJv
エレ解の問題
0250132人目の素数さん
垢版 |
2018/04/17(火) 09:43:20.28ID:d+hbLPaY
>>249
中心距離ってrジャン
0251132人目の素数さん
垢版 |
2018/04/17(火) 10:53:29.39ID:+pEnOXwO
>>245
むかしシュプリンガーの公式サイトで、秋山仁が全30回のシュプリンガー数学コンテストをやってて、
シュプリンガーがHPページリニューアルした後も、数学コンテストの解答解説を残していたけど、
いまみると、HPすら存在しないがな
0252132人目の素数さん
垢版 |
2018/04/17(火) 11:47:27.80ID:qz99Mxbe
>>248
1-γっぽい
0254132人目の素数さん
垢版 |
2018/04/17(火) 12:50:17.86ID:KM09+lmI
>>248
正の整数mが n≧m+1 を満たす時、
(1/n)Σ[k=1,n]{n/k}
=(1/n)Σ[t=1,m]Σ[k: t≦n/k<t+1](n/k-t) + (1/n)Σ[k: 1≦k≦n/(m+1)]{k/n}
=(1/n)Σ[t=1,m](∫[n/(t+1),n/t](n/x-t + (n/[x]-n/x)dx +O(1)) + O(1/m) (O(1)は積分区間の端点のズレ補正。すなわち絶対値は一様に2以下)
=log(m+1) - Σ[t=1,m]1/(t+1) +O(m)+O(n/m).
mは任意であったから、m=[√n] (n≧2) 等と定めればこの式のn→∞での極限は 1-γ. (ただしγはオイラーの定数)
0255132人目の素数さん
垢版 |
2018/04/17(火) 12:52:44.16ID:+pEnOXwO
p=4n+1 (n∈N)をみたす素数pに対して、以下を証明せよ.
(1) 1, 2, …, 2n の中に, 法 p の平方剰余と平方非剰余が n 個ずつ存在する。
(2) 1, 2, …, 4n における法 p の平方剰余の中には, 偶数と奇数が n 個ずつ存在する。
0256132人目の素数さん
垢版 |
2018/04/17(火) 14:17:53.48ID:5Ioo4LVI
>>255できた。
以下(x/p)を平方剰余記号, A={k | (k/p) = 1}, B={k | (k/p) = -1}として(-1/p)= 1だから
#A∩[1..2n] = #A∩[-2n,-1], #B∩[1..2n] = #B∩[-2n,-1] かつ #A∩[-2n,2n] = #B∩[-2n,2n] = 2nにより(1)を得る。
(2/p) = 1のときはA∩[1,2n]とA∩{2,4,…,2n]とA∩{-2n,…,-4,-2]とはxと2xと-2xを対応させてすべてn元とわかる。同様に#B∩{2,4,…,2n]=#B∩{-2n,…,-4,-2]=nである.
(2/p) = 1のときはA∩[1,2n]とB∩{2,4,…,2n]とB∩{-2n,…,-4,-2]とはxと2xと-2xを対応させてすべてn元とわかる。同様に#A∩{2,4,…,2n]=#A∩{-2n,…,-4,-2]=nである。
0257132人目の素数さん
垢版 |
2018/04/17(火) 23:34:48.83ID:sz8bxIx6
>>233
位相幾何学ではストローもドーナツもCDもネックレスも穴は1つ
0258132人目の素数さん
垢版 |
2018/04/18(水) 00:02:57.92ID:zfuntLnI
穴は1つしかないから(格言)
0259132人目の素数さん
垢版 |
2018/04/18(水) 00:12:39.98ID:cm2lWraW
3秒ほど考えた
便所荒らし糞ホモの考えですね
0260132人目の素数さん
垢版 |
2018/04/18(水) 00:26:23.03ID:3HkyYObn
(ホモロジーだけに)
0261132人目の素数さん
垢版 |
2018/04/18(水) 13:51:13.89ID:aemp1B+Z
p を素数とする。
整数 a は p の倍数でなく、ある x, y∈Z を用いて p = x^2 - ay^2 と表される。
このとき、a は p の平方剰余であることを示せ。
0262132人目の素数さん
垢版 |
2018/04/18(水) 14:57:48.83ID:OD1LF7hc
>>261
p|yならp|xとなりv_p(右辺) ≧ 2> 1 = v_p(左辺)より矛盾。よってyはpの倍数でないからx/yはp進整数。このときa = (x/y)^2 (mod p)。
0264132人目の素数さん
垢版 |
2018/04/19(木) 03:31:19.65ID:gkRveId7
>>192 (2) を弄ってみた
(2)’ 素数 p, q が、p≡1 (mod 4)、q=2p+1 をみたすとき、2 は q の原始根であることを示せ。
0265132人目の素数さん
垢版 |
2018/04/19(木) 12:19:25.95ID:ZpgZ64DJ
>>264
できた。
Fqの乗法群Gは位数2pの巡回群だから2が原始根でなければ2^2≡1(mod q)であるか2^p≡1(mod q)のいずれかである。
前者ならq=3であるが仮定に反する。
後者なら準同型写像f,g:G→Gをそれぞれ2乗,p乗の写像としてker g = im fと2+qZ∈ker gにより2が平方剰余となりq≡1,7 (mod 8)となる。
一方p≡1,5(mod 8)よりq≡3,5 (mod 8)となり矛盾をえる。
0266132人目の素数さん
垢版 |
2018/04/19(木) 21:07:55.93ID:gkRveId7
gcd(15,n)=1 をみたす奇数 n に対して、Jacobi記号 (-15/n) = 1 となる n の条件を求めよ。
0269132人目の素数さん
垢版 |
2018/04/20(金) 02:27:29.12ID:kp1G+YoD
2 以上の自然数 n に対して、n と互いに素で、n より小さな全ての自然数の算術平均を求めよ。
0270132人目の素数さん
垢版 |
2018/04/20(金) 04:36:18.70ID:8JvmETkN
n/2
0271132人目の素数さん
垢版 |
2018/04/21(土) 00:10:01.29ID:ZdHWeLtB
1−π^2/12。
0272132人目の素数さん
垢版 |
2018/04/21(土) 17:07:11.66ID:oKMSyftX
>>268
(約数の総和の1からnまでの和)/n^2の極限を1から引いたものになるね
これはどうやるのやらわからんが
0274132人目の素数さん
垢版 |
2018/04/22(日) 19:06:49.87ID:XmgrwCPE
>>268
(1/n^2)Σ[k=1,n]n%k
=(1/n^2)Σ[t=1,m]Σ[k: t≦(n/k)<t+1](n-tk) + O(1/m)
=(1/n^2)Σ[t=1,m] ( n・(n/t-n/(t+1)) - t・((n/t)^2-(n/(t+1))^2)/2 + O(n) ) +O(1/m)
=-Σ[t=1,m](2t+1)/(2t(t+1)^2) +1+O(m/n)+O(1/m)
=-(1/2)Σ[t=1,m]1/(t+1)^2+1/(t(t+1)) +1+O(1/√n) (m=[√n]と定めた時)
→1-(π^2)/12 (n→∞の時)
0276132人目の素数さん
垢版 |
2018/04/23(月) 09:05:17.63ID:WRc1u9WC
>>268
この問題の系として
自然数mの正約数の総和をS_mとするとき
lim[n→∞](S_1+S_2+...+S_n)/n^2=(π^2)/12
になると言えますが、初等的に(高校数学で)証明するやり方はありますか?
0277132人目の素数さん
垢版 |
2018/04/23(月) 10:14:07.56ID:csHJcyqY
>>275

 cot(θ/2) = (cosθ +1)/(sinθ +0) = cotθ + 1/sinθ = cotθ + √{1+(cotθ)^2},
と漸化式を見比べて
 a[n] = cot(c/2^n)
    = cot{π/2^(n+2)},   {←a[0] = cot(π/4)}

∴求める極限は 4/π
0288132人目の素数さん
垢版 |
2018/04/24(火) 13:22:19.46ID:imaaXaqT
単位正方形の面積を3等分する曲線(分岐あり)の長さの最小値を求めよ
0290132人目の素数さん
垢版 |
2018/04/25(水) 00:17:03.92ID:s9HOMEtU
>>289
不正解です
もっと短く出来ます
0291 【だん吉】
垢版 |
2018/04/25(水) 00:20:21.67ID:Y0UXfQnX
>>289
正方形をYの字で区切る。三つの区切り線それぞれの長さをxとすると、
x=(√3)/3
∴3x=√3
0292 【ぴょん吉】
垢版 |
2018/04/25(水) 00:25:41.12ID:Y0UXfQnX
>>290
面積(1/3)の三つのエリアがパッツンパッツンのパンティー履いた太ももになります。前>>291


それとも脚をななめらせろと?
0293132人目の素数さん
垢版 |
2018/04/25(水) 00:31:52.28ID:s9HOMEtU
ちなみに答えは直線じゃないです

>>291
直線の場合でも
正三角形よりもう少し折れたほうが短くなります
0294132人目の素数さん
垢版 |
2018/04/25(水) 00:34:09.51ID:s9HOMEtU
正三角形というか120°に折れたY字というか
0295 【末吉】
垢版 |
2018/04/25(水) 00:36:28.57ID:Y0UXfQnX
素直にTの字にします。
(与式)=1+2/3=5/3
>>292


1.66……<√3 たしかに。
0296132人目の素数さん
垢版 |
2018/04/25(水) 00:42:46.17ID:s9HOMEtU
>>295
線分だけパターンでももっとそれより短く出来ます
0297132人目の素数さん
垢版 |
2018/04/25(水) 00:43:05.10ID:CPKgHcHK
以下を証明せよ。
(1) 奇素数pが a^2 + b^2 (a、b∈Z) の約数で、aとbをともに割り切らないならば、p≡1 (mod 4).
(2) 奇素数pが a^2 + 2b^2 (a、b∈Z) の約数で、aとbをともに割り切らないならば、p≡1 (mod 8) または p≡3 (mod 8).
0298 【大吉】
垢版 |
2018/04/25(水) 00:52:37.09ID:Y0UXfQnX
ふつうのY字のパンティーよりTバックのほうがよりパッツンパッツンとは、おもしろい問題ですね。
>>295
>>296え、もっとパッツンパッツンにできる!?
ふんどし型か?

ちょっとおもしろいから、答え言わないで。また考えましょう。
0299132人目の素数さん
垢版 |
2018/04/25(水) 01:03:20.48ID:KoaEOy7E
>>297
(a/p)を平方剰余記号として
(1) (-1/p) = 1よりp≡1 (mod 4)
(2) (-2/p) = 1よりp≡1,3 (mod 8)

実質補充法則(2)の第二補充法則の証明は初等的とはいえ、そんなにスカッとは解けない希ガス
0300イナ ◆/7jUdUKiSM
垢版 |
2018/04/25(水) 01:18:15.36ID:Y0UXfQnX
やっぱりY字のきわどいパンティーのほうがTバックよりパッツンパッツンと仮定します。
太ももの境界をx、パンティーの境界をyとして(さっきはすべてxにしてた)、やり直し。
>>298

(与式)=x+2y
=1/3−(√3)/12+2(√3/3)
=1/3+(7√3)/12
≪(<(5/3)<√3)
かなり小さい。
0301132人目の素数さん
垢版 |
2018/04/25(水) 01:27:52.71ID:s9HOMEtU
>>300
さすがにそこまで短くはなりません
計算間違えてないですか?
0302132人目の素数さん
垢版 |
2018/04/25(水) 01:43:11.01ID:i3CGBkWM
>>296 >>300

y = √{(1/2)^2 + (4/3 - 2x)^2}   (←等積条件)

x + 2y ≧ (8+3√15)/12 = 1.6349

x = 2/3 - 1/(4√15) = 2/3 - (tanδ)/4 = 0.60212

sinδ = 1/4,
0303132人目の素数さん
垢版 |
2018/04/25(水) 01:46:01.43ID:s9HOMEtU
>>302
そうですね
線分パターンだとこれが最適になります

ただ曲線にするともっと短くなります
0304イナ ◆/7jUdUKiSM
垢版 |
2018/04/25(水) 01:55:47.69ID:Y0UXfQnX
>>301パンティー部分の半分(台形)の面積を1/3にしてました。1/6でした。
>>300
x=2/3−(√3)/12
y=(√3)/3
(与式)=x+2y
=2/3+(7√3)/12
=(8+7√3)12
0305132人目の素数さん
垢版 |
2018/04/25(水) 02:23:12.49ID:Q7D+oEWF
>>288
最小である根拠はないけど
一辺から中心方向に2/3+√3/4-π/6の長さの垂直二等分線を引き、そこから両隣の辺に向けて単位円の12分の1円弧を引いた場合(分岐点における接線の角をそれぞれ120°とし、各辺との交点における接線を辺と直交するように引く)
分割線の長さの総和=2/3+√3/4+π/6≒1.623
0306イナ ◆/7jUdUKiSM
垢版 |
2018/04/25(水) 03:00:37.62ID:Y0UXfQnX
やっぱり脚をななめらせたほうがいいということですか。
>>304
半径rの四分円に対角線を差した音叉のような形に三分割します。
(1/4)πr~2=1/3
r~2=4/(3π)
r=2/√(3π)
四分円の弧の部分
=(1/4)2πr
=(1/2)π×2/√(3π)
=√(3π)/3
対角線部分=(√2)−r
=√2−2/√(3π)
(与式)=(1/2)π×2/√(3π)+√2−2/√(3π)
=π√(3π)/3π+√2−2√(3π)/3π
={(π−2)√(3π)}/3π+√2
0307イナ ◆/7jUdUKiSM
垢版 |
2018/04/25(水) 04:45:57.16ID:Y0UXfQnX
>>306だめだ、脚が長すぎる。
単位正方形を左右対称な音叉のような形で三分割するとして、脚は一辺(底辺)に垂直に立てじゅうぶん短くします。音叉の弧と単位正方形でできる上下逆の蒲鉾形の面積は1/3であり、上に尖った扇形に等積変形できる。
(つづく)
0308132人目の素数さん
垢版 |
2018/04/25(水) 06:45:37.98ID:C3c2S/2O
>>305 が正解っぽい

左右の対称性を仮定して
J(f,λ)=2∫[0,1/2]√(f'(x)^2+1)dx+f(1/2)-λ(∫[0,1/2]f(x)dx-1/3)
の変分δJ(f,λ)=0を解くと
f(x)=√(1-x^2)+2/3-√3/4-π/6, λ=2
のとき極小値
J(f,λ)=2/3+√3/4+π/6
をとる
0309132人目の素数さん
垢版 |
2018/04/25(水) 07:34:20.20ID:spy7pyf4
それができたら次は立方体でやってね
0310132人目の素数さん
垢版 |
2018/04/25(水) 07:37:39.84ID:KwSfzGxO
>>305
>>308

おーすごい まさか一晩で解かれるとは
正解です

厳密には相分離モデルを応用して平均曲率が局所一定になることを示してそこから円、線分の組み合わせということが分かってあとは頑張る感じです
0311132人目の素数さん
垢版 |
2018/04/25(水) 07:45:41.00ID:KwSfzGxO
>>309
3次元の場合は3等分くらいなら出来るかもしれませんが未解決なケースもかなり多いので解こうとするのは危険かもしれないです
0312132人目の素数さん
垢版 |
2018/04/25(水) 08:56:04.91ID:CPKgHcHK
>>264
> >>192 (2) を弄ってみた
> (2)’ 素数 p, q が、p≡1 (mod 4)、q=2p+1 をみたすとき、2 は q の原始根であることを示せ。

少し弄ってみた。

(2)’’ 素数 p, q が、p≡1 (mod 4)、q=4p+1 をみたすとき、2 は q の原始根であることを示せ。
0313132人目の素数さん
垢版 |
2018/04/25(水) 08:57:43.06ID:CPKgHcHK
>>312
訂正。

(2)’’ 素数 p, q が q=4p+1 をみたすとき、2 は q の原始根であることを示せ。
0314132人目の素数さん
垢版 |
2018/04/25(水) 10:33:13.41ID:qlUN5/CP
>>313
q≡5 (mod 8)より2は法qの平方剰余ではない。よって2^2q≡1 (mod q)ではない。
q≠3,5より2^4≡1 (mod q)ではない。
0315132人目の素数さん
垢版 |
2018/04/25(水) 11:10:45.49ID:qlUN5/CP
>>314
訂正
×:よって2^2q≡1 (mod q)ではない。
○:よって2^2p≡1 (mod q)ではない。
0316132人目の素数さん
垢版 |
2018/04/25(水) 15:03:46.70ID:CPKgHcHK
つまり、こうでござるな。

(2)’’’ p≡±1 (mod 8) をみたす素数pに対して、2 は q の原始根でない
0317132人目の素数さん
垢版 |
2018/04/25(水) 16:35:50.56ID:CPKgHcHK
p、qは奇素数で、pが 2^q -1 の約数ならば、2はpの平方剰余であることを示せ。
0318132人目の素数さん
垢版 |
2018/04/25(水) 17:02:41.21ID:+BsFnCRa
>>317
qはp-1の約数であるがqは奇数だから(p-1)/2の約数でもある。よって2^((p-1)/2)≡1(mod p)。
0319イナ ◆/7jUdUKiSM
垢版 |
2018/04/25(水) 17:21:59.72ID:Y0UXfQnX
単位正方形の中心の真上rの位置を要とした半径rの扇形を描く。弧の中間点から底辺に垂線を下ろし、孤と垂線で三分割する。
>>307
扇形の面積は1/3
扇の端は単位正方形と単位正方形の上から(r^2)/4−1/16(0より大きく1/3より小さい)の点で接する。

∴1/2<r<5/6
弧の長さ=2/(3r)
弧の中間点から単位正方形の底辺までの距離=1+(r^2)/4−1/16−r
境界線の合計f(r)=1+(r^2)/4−1/16−r+2/(3r)
=(r^2)/4−r+15/16+2/(3r)
=(−24r^2+45r+48)/48r
微分f'(r)=0とすると、
3r^3−6r^2=4
f(r)=−r/2+15/16+1/r

扇形の面積について、
2/(3r)×r×(1/2)=(πr^2)/3
∴πr^2=3
r=√3/√π

(つづく)
0320イナ ◆/7jUdUKiSM
垢版 |
2018/04/25(水) 17:43:03.01ID:Y0UXfQnX
>>319修正
単位正方形の中心の真上rの位置を要とした半径rの扇形を描く。弧の中間点から底辺に垂線を下ろし、孤と垂線で三分割する。
>>307
扇形の面積は1/3
扇の端は単位正方形と単位正方形の上から(r^2)/4−1/16(0より大きく1/3より小さい)の点で接する。

∴1/2<r<5/6
弧の長さ=2/(3r)
弧の中間点から単位正方形の底辺までの距離=1+(r^2)/4−1/16−r
境界線の合計f(r)=1+(r^2)/4−1/16−r+2/(3r)
=(r^2)/4−r+15/16+2/(3r)
=(−24r^2+45r+48)/48r
微分f'(r)=0とすると、
3r^3−6r^2=4
f(r)=−r/2+15/16+1/r

扇形の面積について、
2/(3r)×r×(1/2)=(πr^2)/3
∴πr^2=1
r=1/√π
f(r)=15/16+√π−1/(2√π)
ちがうか。
(答え)不思議なルートパイ
0321イナ ◆/7jUdUKiSM
垢版 |
2018/04/25(水) 19:00:00.75ID:Y0UXfQnX
やっぱりπr^2=1ではない。前>>320
シャボン玉を正方形のタイルの上で三個均等にくっつけるみたいなことか。タイルの形の影響で、分岐点からタイルの一辺までが垂直なら境界線は直線で、そうでないなら曲線になるんじゃないか。

シャボン玉の境界は辺に対してより垂直になろうとするんじゃないか。
_
γ]
0322イナ ◆/7jUdUKiSM
垢版 |
2018/04/25(水) 21:09:49.78ID:Y0UXfQnX
正方形の土地をなるべく短い境界線で金をかけずに塀を作り三人の息子たちに分け与えたい父の気持ちを想像する。
「だれが曲線の塀などこしらえるものか。こっちは有り金をなるべくむだにしたくないんじゃ!!」父は言った。「直線や、直線や!!」
>>321「まず長男に北側の一辺をやろう。次男は東側の一辺のうち北からaだけいったところに杭を立てよ。三男は西側の一辺のうち北からbだけいったところに杭を立てよ。次男と三男の境界は東からcのところに、
0<a<b<c<1/2
となるように杭を立てよ。正方形の土地のまん真ん中に杭を立て、あとは縄を張って地境を決めろ」
息子のだれかが計算した。
「ただの三連立の一次方程式やないか」独りごちながら。
a=1/12
b=1/4
c=5/12
∴示された。
0323132人目の素数さん
垢版 |
2018/04/25(水) 21:48:01.05ID:BcUTTOXX
ナニコレ?
0324イナ ◆/7jUdUKiSM
垢版 |
2018/04/25(水) 21:58:48.29ID:Y0UXfQnX
a=1/12,b=1/4,c=5/12
>>322補足。
長男と次男の境界=√{(1/2)^2+(1/2−a)^2}
=√{1/4+(5/12)^2}
=(√61)/12
長男と三男の境界=√{(1/2)^2+(1/4)^2}
=√(1/4+1/16)
=(√5)/4
次男と三男の境界=√{(1/2)^2+(1/12)^2}
=(√37)/12

境界線の合計=(√61)/12+(√5)/4+(√37)/12
≒0.65+0.559+0.507
≒1.716

厳しいなぁ!! 縄ピンと張っても1.6台にならない。
0326イナ ◆/7jUdUKiSM
垢版 |
2018/04/25(水) 23:56:24.99ID:Y0UXfQnX
>>325単位正方形の左下に半径r、面積1/3の四分円を描く。
πr^2=4/3
r=2/√(3π)≒0.65147
四分円の孤ABと右辺に直交するように孤MCを描くと中心角は最大π/4だと思う。
(弧の長さ×半径÷2=扇形の面積)より、逆に面積×2を半径で割って境界線の長さを出す。
境界線AB=(1/3)×2÷2/(√3π)
=√π/√3
境界線ABとMCの最大値は作図によりこれの1.5倍と考えられる。
(√π/√3)×1.5
=1.0233256……×1.5
=1.5349884……
0328 【中吉】
垢版 |
2018/04/26(木) 00:23:24.70ID:TQ9j6XC/
1.5倍は感覚的ですが、
>>327
式で書くと、
境界線の最小値
=√(3π)/2
0329132人目の素数さん
垢版 |
2018/04/26(木) 00:50:30.83ID:ip5ulRQt
もうすでに>>305で解かれてるのに何やってんのこいつ?
0330132人目の素数さん
垢版 |
2018/04/26(木) 08:00:01.14ID:3zpz03fU
解かれていない。
0331イナ ◆/7jUdUKiSM
垢版 |
2018/04/27(金) 02:33:17.21ID:KVwn7NU0
T字
1+(2/3)
=1.66666666……
Y字(X+2Y)
=(8+7√3)÷12
=1.67702964……
これらを踏まえ、三本の境界線を分岐点からX=0.55ずつとり、一本は底辺の垂直二等分線上、底辺から上に0.55の位置でY字に分岐させ、あとの二本は左辺または右辺と分岐点の高さよりaだけ上の位置で交差させる。
分割した一つの体積(台形)=(0.55+0.55+a)×(1/2)÷2=1/3
a=(4/3)−1.1
=0.2333……
Y字(3X)
=1.65
>>328
0332イナ ◆/7jUdUKiSM
垢版 |
2018/04/27(金) 03:14:53.35ID:KVwn7NU0
底辺の垂直二等分線上、底辺から上に0.54(>0.5 ∵左右の辺に届かないといけないから)の位置に分岐点をとると、
分割した一つの体積(台形)=(0.54+0.54+a)×(1/2)÷2=1/3
a=(4/3)−1.08
=0.25333……
Y字(3X)
=1.62
>>331これ1.6
0333イナ ◆/7jUdUKiSM
垢版 |
2018/04/27(金) 03:19:06.23ID:KVwn7NU0
底辺の垂直二等分線上、底辺から上に0.53の位置に分岐点をとると、
分割した一つの体積(台形)=(0.53+0.53+a)×(1/2)÷2=1/3
a=(4/3)−1.06
=0.27333……
Y字(3X)
=1.59
>>332これ1.6切った!!
0334132人目の素数さん
垢版 |
2018/04/27(金) 09:30:37.15ID:NFZEifrM
>>333

>>303でも言いましたが線分だけの場合は>>302が最短になります
0.6を切ることはあり得ません
0335132人目の素数さん
垢版 |
2018/04/27(金) 09:31:18.84ID:NFZEifrM
0.6→1.6でした
0337イナ ◆/7jUdUKiSM
垢版 |
2018/04/27(金) 15:35:18.85ID:KVwn7NU0
三本の境界線を分岐点からXずつとり、一本は底辺の垂直二等分線上、底辺から上にXの位置でY字に分岐させ、あとの二本は左辺または右辺と分岐点の高さよりaだけ上の位置で交差させる。
分割した一つの面積(台形)={X+(X+a)}×(1/2)÷2=1/3
a=(4/3)−2X――@
斜めの分割線について三平方の定理より、
(1/2)^2+a^2=X^2――A
@をAに代入して整理すると、
108X^2−192X+73=0
1/2<X<1に注意して、
X=(16−√37)/18
3X=(16−√37)/6
=1.65287291……
>>333
0338132人目の素数さん
垢版 |
2018/04/27(金) 16:17:47.03ID:v+crcbPI
等積条件下で長さ最小⇒定曲率
はどうやって示すんですか?
0339イナ ◆/7jUdUKiSM
垢版 |
2018/04/27(金) 17:53:53.21ID:KVwn7NU0
底辺の垂直二等分線上の分岐を120°、1/12円弧が左右の辺に直交するとして、
境界線の最小値
=X+2Y+πr/3
>>337
円弧の半径r=1−Y√3
(1/2)^2+X^2=Y^2+r^2
整理すると、
3X+6Yr+πr^2=4
3X+6Y(1−Y√3)+π(1−Y√3)^2=4
3X+6Y−6√3・Y^2+π−2π√3・Y+3Y^2=4
X=4/3−(2−2π√3/3)Y+Y^2
境界線の合計F(Y)=X+2Y+π(1−Y√3)/3
=4/3−(2−2π√3/3)Y+Y^2+2Y+πr/3
=4/3+(2π√3/3)Y+Y^2+π(1−Y√3)/3
=Y^2+(π√3/3)Y+π+(4/3)
(つづく)
0340132人目の素数さん
垢版 |
2018/04/27(金) 20:42:46.25ID:nJOWrXzq
1 以上 1000000 以下の自然数のうち、各桁の数が 0, 1, 2 のいずれかであるような 7 の倍数は何個あるか。
0342イナ ◆/7jUdUKiSM
垢版 |
2018/04/27(金) 21:24:55.98ID:KVwn7NU0
底辺から底辺の垂直二等分線上の分岐点までをXとして120°の角度で分岐し、半径1の1/12円弧が左右の辺に直交するとして、
境界線の合計=X+π/3
>>339
分割した面積=X(1/√3)X(1/2)+π/12−(1/2−X/√3)(1/2−X/√3)√3(1/2)=1/3
=(1/2√3)X^2+π/12−(√3)/2・(1/2)^2−(√3)/2・(X/√3)^2+(2X/√3)(√3/2)=1/3
(1/2√3)X^2+π/12−(√3)/8−(√3)/6X^2+X=1/3
π/12−(√3)/8+X/2=1/3
X/2=1/3+(√3)/8−π/12X=2/3+(√3)/4−π/6
境界線の最小値=X+π/3
=2/3+(√3)/4+π/6
=0.6666666……+0.4330127……+0.5235987……
≒1.623278
0344132人目の素数さん
垢版 |
2018/04/27(金) 22:05:10.47ID:ldwAt9sW
>>343
定曲率になる解説をおながいしまつ
0345132人目の素数さん
垢版 |
2018/04/28(土) 11:41:45.00ID:9CKS2DSq
〔ウィア=フェラン予想〕

3次元空間を体積Vの泡に分割するとき、境界面積が最小になるのはウィア=フェラン構造(Weaire-Phelan structure)か?

D.Weaire & R.Phelan: Phil. Mag. Lett., 69, p.107-110 (1994) "A counter-example to Kelvin's conjecture on minimal surfaces"
0346132人目の素数さん
垢版 |
2018/04/28(土) 12:22:19.38ID:9CKS2DSq
>>345

切頂8面体(ケルビン14面体)の境界面積は
 S = (3/4){4^(1/3)}(1+√12) V^(2/3) = 5.3147397 V^(2/3)

Weaire-Phelan 構造の境界面積はこれより約 0.3% 小さい。
 S = 5.30 V^(2/3)
0347132人目の素数さん
垢版 |
2018/04/28(土) 22:31:07.34ID:Q7JYuciE
岩波 数学公式IIIのp.2より引用の公式:

Γ(1/4)=2^(3/4)√π[(3/5)・(7/9)・(11/13)・(15/17)…]^(1/2)

は正しいか?もし誤りであれば誤りの原因を考察し訂正せよ。
0348132人目の素数さん
垢版 |
2018/04/29(日) 00:45:55.20ID:FNqzl5v2
>>347
無限積のところ0にいくなぁ
0349132人目の素数さん
垢版 |
2018/04/29(日) 01:50:32.50ID:FNqzl5v2
無限乗積表示
Γ(1/4) = 1/4e^(γ/4)Π((4m+1)/4m)e^(-1/4m)
Γ(3/4) = 3/4e^(3γ/4)Π((4m+3)/4m)e^(-3/4m)
と倍角公式
Γ(1/2) = Γ(1/4)Γ(1/4 + 1/2)/√(2π)
をうまくつかってΓ(3/4)を消去しようとして失敗したくさいねぇ。無限乗積はΠ((4m+1)/4m)とかΠ((4m+3)/4m)は各々単独では収束しないからあとのe^~と切り離せないのに。信じられんミスですな。
0350132人目の素数さん
垢版 |
2018/04/29(日) 02:00:09.16ID:LZWvDOTX
>>347

岩波「数学公式I」p.229 を見ると

Γ(1/4) = 2 π^(1/4) √K(1/√2),

K(1/√2) = 1.85407467730137191843385034719526  (*)

Γ(1/4) = 3.625609908221908311930685155867672

(*) K(k) は第1種の完全楕円積分。
 K(k) = ∫[0,π/2] 1/√{1 - (k・sinθ)^2} dθ
    = (π/2){1 + Σ[r=1,∞] {(2r-1)!!/(2r)!!}^2・k^(2r) }
0351132人目の素数さん
垢版 |
2018/04/29(日) 02:57:34.53ID:FNqzl5v2
正しくΓ(3/4)を消去すれば

Γ(1/4)^4 = 48√2π Π(1+3/4m)/(1+1/2m)^3

ですな。
0352132人目の素数さん
垢版 |
2018/04/29(日) 03:01:03.51ID:FNqzl5v2
訂正
Γ(1/4)^4 = 48√2π Π(1+3/(4m))/(1+1/(4m))^3
0353132人目の素数さん
垢版 |
2018/04/29(日) 03:14:07.86ID:FNqzl5v2
>>349の左辺も正しくは逆数ですね。
まぁまぁあうなぁ
gamma(1/4)^4,numer;
48*sqrt(2)*%pi*(product ((1+3/4/i)/(1+1/4/i)^3, i, 1, 10000)),numer;
(%o45) 172.7922660636603
(%o46) 172.7955056790521
0354132人目の素数さん
垢版 |
2018/04/29(日) 03:35:41.70ID:us7WqjTP
岩波の関係者見てるか〜?
はよ改訂しろや
0355132人目の素数さん
垢版 |
2018/04/29(日) 10:14:34.00ID:n1kfIHw7
>>352
正解ですがsinの無限乗積を用いれば、もう少しきれいな形にできて
√2=2sin(π/4)=(π/2)Π(1-1/(4m)^2)
から√2を消去して
Γ(1/4)^4 = 24π^2 Π(1-1/(4m))(1+3/(4m))/(1+1/(4m))^2
= 8π^2 (3/1)・(3/5)・(7/5)・(7/9)・(11/9)・(11/13)・(15/13)…
が得られ、これを1/4乗したのが訂正式だと思われます。

ここまでの式は正しいのですが、不用意に分母を1つずらして
二乗でくくってしまうと例の誤りの公式になります。
0356132人目の素数さん
垢版 |
2018/04/29(日) 23:20:19.24ID:LZWvDOTX
>>341

フェルマーの小定理から
 10「abcdef」-「bcdefa」= (10^6 - 1)a ≡ 0 (mod 7)

∴ ローテートしても剰余は変わらない。

(1000000は7の倍数でないから省いてよい)
0358132人目の素数さん
垢版 |
2018/04/30(月) 01:03:27.45ID:bKuKTDT2
>>357
それが示せたとしてちょうど7個に一個は7の倍数ってしめせる?
そもそも>>340は6桁以下である意味ほとんどないけど。10桁以下でも[3^10/7]だよ。
0359132人目の素数さん
垢版 |
2018/04/30(月) 01:47:49.00ID:2V4BpPyt
>>340の話題まだ続いてたの?

「1 以上 1000000 以下の自然数のうち、各桁の数が 0, 1, 2 のいずれかであるような数」の集合は
S_10={s|s=Σa_i・10^iかつa_i∈{0,1,2}かつ1≦s≦10^6}となるが、この集合は
S_3={s|s=Σa_i・3^iかつa_i∈{0,1,2}かつ1≦s≦3^6}と、同一の有限数列{a_i}を持つ要素同士での一対一対応がある。
(S_10とS_3のいずれの定義でも、異なる{a_i}に対してsの値が異なるから)
また、10≡3 (mod 7) だから Σa_i・10^i≡Σa_i・3^i (mod 7) であり、これらのことから、S_10 と S_3 に含まれる7の倍数の個数は等しい。
S_3 は1以上3^6以下の自然数の集合となる。したがって、S_3 に含まれる 7 の倍数の個数は[3^6/7]個。
S_10 に含まれる 7 の倍数の個数もこれと等しく[3^6/7]=104個。
0360132人目の素数さん
垢版 |
2018/04/30(月) 07:45:47.91ID:unf6uQw9
>>347 の類題まだあるようです

岩波 数学公式IIIのp.13、Eulerの定数γの積分表示
γ = ∫[0,1] log|log t|dt
は正しいか?
0361IQの低い人
垢版 |
2018/04/30(月) 13:55:59.72ID:tdDKI26q
数学公式なんて必要なの
インターネットでじゅうぶんじゃないの?
0362132人目の素数さん
垢版 |
2018/04/30(月) 15:31:22.71ID:unf6uQw9
一般化して考えると、人間の発見した数学の知識は本にする必要があるか?
電子化してしまえば便利で使い勝手が良いではないか?
という質問になると思うけど、難しい質問ですね。
0364132人目の素数さん
垢版 |
2018/04/30(月) 16:16:13.98ID:9GopzljD
そりゃ書籍なんてどんどん厚くなってくわけだし
OEISみたいにとっとと電子化した方がよい
0365132人目の素数さん
垢版 |
2018/04/30(月) 16:28:40.42ID:YkZppX/u
電子化に「頼り切った」場合、データが吹っ飛んだ場合の復旧は大丈夫なのか。
紙なら数百年は持つが。
0367132人目の素数さん
垢版 |
2018/04/30(月) 20:39:39.75ID:lihGKJI8
>>365
データ構造とか
資料と知識ある人が亡くなれば
ブラックボックス化してしまうよな
0368132人目の素数さん
垢版 |
2018/04/30(月) 20:43:00.98ID:mYEYW+f+
そういえばCOBOLみたいな化石言語を使える後継者がいなくて、システムの維持が困難だとかあるらしいね
0369132人目の素数さん
垢版 |
2018/04/30(月) 21:22:05.39ID:EyjNbgxA
卒業アルバムをCD-ROMで配布したら数十年後にはみんな読めなくなってるみたいな
0370132人目の素数さん
垢版 |
2018/04/30(月) 21:51:16.80ID:f2DvPYO1
>>360
は数値計算させてみると
romberg(exp(-x)*log(x),x,0.01,1)+romberg(exp(-x)*x,1,500)+501*exp(-500),numer;
%gamma,numer;
(%o51) −.005043828410193907
(%o52) .5772156649015329
でo51>∫[0,1] log|log t|dt
だから全然ダメっぽいけど何をどう間違ったのかはさっぱりわからんorz
0372132人目の素数さん
垢版 |
2018/04/30(月) 22:07:54.94ID:f2DvPYO1
ああ、>>360は-抜けてるだけか。
0373132人目の素数さん
垢版 |
2018/05/01(火) 14:57:49.96ID:hppQFjS3
1≦k≦nをみたすkのうち2^(k-1)の最高位が4であるものの数をx_nとして(x_n)/nの極限を求めよ
東大模試の問題ですが良く分かりません
0375132人目の素数さん
垢版 |
2018/05/01(火) 15:36:24.87ID:QQuzwbBg
1,2,4,8,16,32,64,128,256,512,
1024,2048,4096,8192,16384,32768,65536,131072,262144,524288
1048576,2097152,…
(2^10≒10^3は有名)

帰納的に
10^(3n)≦2^(10n)<10^(3n)+10^(3n-1)
を示すとか
0376132人目の素数さん
垢版 |
2018/05/01(火) 15:38:52.55ID:QQuzwbBg
>>374
ほぇ^〜
0377132人目の素数さん
垢版 |
2018/05/01(火) 17:37:49.16ID:0rV/A0yL
>>373
なんとなくだけど
常用対数でlog(5/4)じゃない?
log2が無理数だから、n・log2の小数部は0以上1未満の間の値を均等にとる。それがlog5とlog4の間にある確率を出せばよい
0378132人目の素数さん
垢版 |
2018/05/01(火) 18:37:42.45ID:Be837sS1
>>373
なに使っても良いならそれですな。いわゆるワイルの一様分布定理。blog n/log10 の小数部は[0,1)で一様に分布する。でも最高位が4の場合はそんな難しいもん使わなくても解けるというのがミソですな。受験数学なら意味あるけどねぇというやつですな。
0379132人目の素数さん
垢版 |
2018/05/01(火) 18:43:23.06ID:Be837sS1
>>360に丸一日悩んでしまった。Γ(s)=∫[0,∞]x^(s-1)exp(-x)dxの両辺微分してs=1放りこんでるだけかorz。まぁおかげでいい勉強になった。
0380132人目の素数さん
垢版 |
2018/05/01(火) 18:53:28.81ID:ZkkSxFx4
>>379
正解です。単純な問題で申し訳ない。
おそらく著者はlogに絶対値をつけるときに符号を勘違いしたのではないかと推察。
0381132人目の素数さん
垢版 |
2018/05/01(火) 19:01:06.79ID:ZkkSxFx4
以下の収束性を議論し、収束するなら収束値を求めよ。

(1) lim[x→1-0]Σ[n=0,∞] (-1)^n x^(n^2)

(2) lim[x→1-0]Σ[n=0,∞] (-1)^n x^(2^n)
0392132人目の素数さん
垢版 |
2018/05/02(水) 13:52:22.80ID:A6AlBBbL
既出かもしれないけど

袋のなかに赤玉6球、白玉7球、黒玉8球入っている。一球ずつ順に取り出す。
黒玉が他の色より一番先にすべて取り出される確率を求めよ。
0393132人目の素数さん
垢版 |
2018/05/02(水) 18:24:28.84ID:cDk91oHu
>>392
29/105
0394132人目の素数さん
垢版 |
2018/05/02(水) 21:52:14.72ID:fjHvbvCm
>>393
御名算
0395132人目の素数さん
垢版 |
2018/05/03(木) 00:12:28.96ID:TIOaAmH9
{a[i]}は自然数の無限列である(i=0,1,2,...)
或るs∈ℝが存在し、任意の自然数iに於いて
0<a[i]-a[i-1]≦sが従う
此のとき、任意のn∈ℕに於いて
a[i]の相異なるn個の要素で等差数列が作れることを示せ
0396132人目の素数さん
垢版 |
2018/05/03(木) 01:13:02.60ID:CZ0Fa01r
>>381

(1)
 Σ[n=0,∞] (-1)^n x^(nn) = {θ_4(0,x) - 1}/2 → -1/2 (x→1-0)

ここに
 θ_4(a,x) = Π[k=1,∞] {1 - x^(2k)} {1 - e^(2ai)・x^(2k-1)} {1 - e^(-2ai)・x^(2k-1)}
はヤコビの楕円テータ函数
0397132人目の素数さん
垢版 |
2018/05/03(木) 04:56:51.43ID:yXlJeHv9
>>394
以下自然数の全体Nのm個の同値類に分けたとき各自然数の属する類を色とよぶ。

van der Waerdenの定理 (1927) 任意の正の整数k,mに対して、或る正の整数N(k,m)が存在して次が成り立つ: N≥N(k,m)なる任意の整数Nに対して、1からNまでの整数をどのようにm色に塗り分けたとしても、必ず同じ色で塗られた長さkの等差数列が存在する。

http://integers.hatenablog.com/entry/2017/03/08/025731

条件を満たす自然数列a[n]をとりf(n) = min{c|c+n = a[i]∃i}、C_c = {n | f(n) = c}とおけば N = C_0 ∪…∪ C_[s]である。任意のkに対してvan der Waerdenの定理よりいずれかのC_cは長さkの等差数列をもつがその各々の項にcを加えた列はa[i]の項からなる。
0398132人目の素数さん
垢版 |
2018/05/03(木) 09:51:47.13ID:xQqmo4zy
>>395
これはどうでしょう?

ai-a(I-1)は当然整数なので条件より取り得る値は1,2…[s-1],[s]のいずれか
iは無数の値をとるので鳩の巣原理より
ai-a(I-1)がある同じ値をとるiは無数のに存在する
よって題意は示された
0399132人目の素数さん
垢版 |
2018/05/03(木) 10:35:01.85ID:PQNVo0sN
>>398
だめ。
a[2j-1] = 2^j
a[2j] = 2^j + 1
と定めればa[i] - a[i-1] = 1となるiは無限にあるけど、a[i]が含む等差数列の長さは4以下。
0401132人目の素数さん
垢版 |
2018/05/03(木) 22:46:01.38ID:TIOaAmH9
うーん
ちょっと本質からずれた質問しますが
これもしvan der Weardenの定理を全く知らなかったら
どんな答案になりますか?
定理自体、色っていう概念使ってて知らないとできないから
そういう前提下だとどういったものになるのかなぁと
0402132人目の素数さん
垢版 |
2018/05/03(木) 22:50:08.28ID:umDHhDvC
(1)内角が全て等しく、辺の長さが全て整数の素数角形は必ず正多角形となることを示せ.

(2)任意の4以上の合成数nに対して、内角は全て等しくて辺の長さは全て整数であるが、正多角形ではないn角形が存在することを示せ.
0403132人目の素数さん
垢版 |
2018/05/03(木) 22:58:02.34ID:PQNVo0sN
>>401

どうなんだろうねぇ?色云々は単なる説明に “雰囲気” を出す為に持ち出されただけであんまり本質的な意味はないと思う。平たく掛けば

―――
N = A_1 ∪ A_2 ∪ … ∪ A_n

なる分割(disjoint でなくとも良い)をあたえればいずれかのA_iはいくらでも長い等差数列を含む。
―――

と色なんて言葉を持ち出す必要はない。ただ>>397のサイトの証明では証明の概念を直感的に理解しやすいように “色” だの “車輪” だのの言葉をつかってるだけ。
まぁこの定理使わないで力技でもできるとは思うけど、割と使いまわせそうな定理だから素直に “へぇ、こんな定理あるんだ” でいいと思う。力技にも興味はあるけど。
0404132人目の素数さん
垢版 |
2018/05/03(木) 23:13:12.65ID:PQNVo0sN
>>402
(1)pを素数とし各頂点の外角が2π/pで各辺の長さa_i(0≦i≦p-1)が整数であるものをとる。(ただしa_iは正の向きに順に図ったものとする)
ζ=exp(2π/n)、f(x) = Σa_i x^i とおけばf(ζ)= 0である。よってf(x)はx^(p-1)+…+1で割り切れるからa_iはすべて等しい。

(2)nを合成数としpをその素因子とする。
a_i = 1 (p|i),
  =2 (otherwise)
として辺の長さが正の向きに順にa_iで外角の大きさがすべて2π/nの多角形をとればよい。
0405132人目の素数さん
垢版 |
2018/05/03(木) 23:36:50.04ID:CZ0Fa01r
>>402

(2)
 n = k ・ L  (k≧2,L≧2)
とする。
内角は全て π - 2π/n とし、
辺の長さは任意の自然数 m_1, m_2, m_3, …, m_k をL回繰り返す、とする。
L回対称
0406132人目の素数さん
垢版 |
2018/05/04(金) 17:40:57.93ID:35MdHy9b
>>399

・公差が1のとき
 {a[i]} はある a[2j-1] と a[2j] を含む。(または、a[2] と a[3] を含む。)
 {a[i]} は i=1〜4 または長さ2以下。

・公差が2以上のとき
 a[k "] < a[k] < a[k '] が長さ3の等差数列だったとする。
 a[k] - a[k"] ≦ a[k] - a[1] = a[k] - 2,
 また、a[k'] - a[k] > 1 から
 a[k'] - a[k] ≧ a[k+1] - a[k] = a[k] - 1,
 したがって
 a[k'] - a[k] ≧ a[k] - 1 > a[k] - 2 ≧ a[k] - a[k"],
 となり矛盾する。
0407132人目の素数さん
垢版 |
2018/05/05(土) 05:01:24.50ID:cos8i+vX
>>406 (修正)

・公差が2以上のとき
  ……
k " < 2j < k ' のとき
 a[k '] - a[2j] ≧ a[2j+1] - a[2j] = a[2j] - 2 = a[2j] - a[1] ≧ a[2j] - a[k"]
∴ 長さ3の等差数列は a[1] < a[2j] < a[2j+1] に限る。
このとき
 a[2j+3] - a[2j+1] = a[2j+1] > a[2j] - 2
となるから、長さ4以上の等差数列はない。
0418132人目の素数さん
垢版 |
2018/05/17(木) 11:40:23.79ID:36lfcc24
2011広島大学後期改

qを6と互いに素な素数ベキ、Fをq元体とし

 X={(x,y,z)∈F×F×F | x^2+y^2+z^2=0}
 Y={(x,y,z)∈X | xyz≠0}
 Z={(x,y,z)∈Y | x≠y, y≠z, z≠x}

とする。X,Y,Zの元数を求めよ。
0419132人目の素数さん
垢版 |
2018/05/18(金) 06:17:07.83ID:539vwTx6
>>373

>>374 のリンクより。

aの最高位の数字が4 ⇔ a/4 と 2a が同じ桁数。

{1,2,…,2^(n-1)} のx_n カ所では同じ桁に4数、それ以外では同じ桁に3数がある。

(n -x_n -1)/3 ≦ n・log_10(2) < (n - x_n +1)/3,

∴ x_n / n → 1 - 3log_10(2) = 0.09691 (n→∞)
0420132人目の素数さん
垢版 |
2018/05/21(月) 04:38:31.67ID:kuqRYFm5
表面積1の八面体の体積の最大値を求めよ
0421イナ ◆/7jUdUKiSM
垢版 |
2018/05/21(月) 13:14:33.19ID:6i5QRyXS
>>420一辺xの正八面体の一つの面は正三角形で、面積は、
(1/2)x×(√3/2)x=1/8
x^2=1/(2√3)
八面体の体積=(1/6)(x/√2)^3
=(1/12√2)x^3 (/12√2)×√3)√(2√3)
=1/48√(3√3)
違うかも。
0422132人目の素数さん
垢版 |
2018/05/21(月) 14:35:46.68ID:3I0IwGqI
>>421
不正解
正八面体より大きく出来る
0423132人目の素数さん
垢版 |
2018/05/21(月) 14:58:44.22ID:oCVK8vvs
>>421
これ八面体ってのは例えば底面が六角形の走らないとか七角形の錐とかもありなん?
0424132人目の素数さん
垢版 |
2018/05/21(月) 15:00:08.42ID:oCVK8vvs
走らないでなく柱ね。底面が六角形の注柱。これもありやとアホほど計算しなあかん希ガス
0425イナ ◆/7jUdUKiSM
垢版 |
2018/05/21(月) 15:25:53.51ID:6i5QRyXS
>>420鉛筆を水平に斬る。前>>421

一辺xの正六角形を上底下底とする高さyの正六角柱の体積P(x,y)=(√3)/4x^2・y
P'(x,y)=0でyを消すと、
P'(x)=0を満たすxに対して、P(x)の最大値が出そうな気がします。
0426132人目の素数さん
垢版 |
2018/05/21(月) 16:24:23.18ID:3I0IwGqI
>>423
>>424
ありです

とにかく面が八つある多面体は八面体です

ただとある法則を使えばそういうものは除外出来ます
0427132人目の素数さん
垢版 |
2018/05/21(月) 16:28:56.50ID:3I0IwGqI
ごめんなさい「ある法則」で除外できる多面体は七角錐だけでした
六角柱は個別で議論する必要あるかも
0428132人目の素数さん
垢版 |
2018/05/21(月) 16:32:02.75ID:+913qm6o
正六面体の角を2つ削ったようなやつもはいるよね。
三角柱から角5つ削るとか。アホほどあるなぁ。
0430132人目の素数さん
垢版 |
2018/05/21(月) 17:57:16.22ID:9YF4F+CN
>>425

正6角柱だと、正8面体と変わらない...orz

1/{6√(3√3)} = 0.073115223

もっと丸い形にすればいい?
0431132人目の素数さん
垢版 |
2018/05/21(月) 18:11:57.11ID:0vZfF+dt
四面体の4つの頂点から小さく四面体を取り除いてできるものとかだいぶ球に近くなるんじゃないかなあ
0434132人目の素数さん
垢版 |
2018/05/21(月) 23:22:14.24ID:YccZYzuR
とある法則で除外できるのが7角錐だけだとかなり残る希ガス。
だいたい8面体と同じ配置とか立方体から2角おとしたのと同じ配置とかに制限したとして、それぞれの場合に最大値求めんのもどえらい面倒くさい気が…。
ほんとに面白いスパっと解ける解法あるんかな?
0435132人目の素数さん
垢版 |
2018/05/22(火) 04:39:03.52ID:RuE2vaj6
>>431

辺長のk倍だけ切り落とす。(0<k<1)
4つの頂点から、k倍サイズの4面体を切り落とす。

体積: V = (1-4k^3) V_0
表面積: S = (1-2kk) S_0

V / S^(3/2) = (1-4k^3)/{6√(3√3)・√2・(1-2kk)^(3/2)}
 ≦ V_0 / {S_0^(3/2)}
 = 1 / {6√(3√3)},  (正8面体)
等号成立は k=1/2 (中点) のとき。
0436132人目の素数さん
垢版 |
2018/05/22(火) 07:10:03.28ID:mp+7pS00
>>434
>>427は語弊がありました

六角柱と七角錐だと七角錐は除外出来るってことです

ある法則を使えば他のパターンも除外出来ます

例えば正八面体のタイプもダメだということが分かります
0437イナ ◆/7jUdUKiSM
垢版 |
2018/05/22(火) 22:06:05.02ID:6b1wDh1x
正六角柱の上底下底の一辺をx高さをyとする。
正六角柱の表面積Sについて、
S=(3√3)x^2+6xy=1
y=(1/6x)-(√3/2)x――@
正六角柱の体積V=(√3/4)x^2・y――A
@をAに代入。
V(x)=(√3/4)x^2{(1/6x)-(√3/2)x}
=(√3/24)x(1-3√3・x^2)
V'(x)=(√3/24)-(9/8)x^2=0
x^2=(√3)/27
x≒0.0487287のときV(x)は最大。
V(0.0487287)=(√3/24)0.0487287(2/3)
=0.0487287(√3)/36
=0.0013831
0438イナ ◆/7jUdUKiSM
垢版 |
2018/05/22(火) 22:47:55.38ID:6b1wDh1x
>>437修正。
正六角柱の上底下底の一辺をx高さをyとする。
正六角柱の表面積Sについて、
S=(3√3)x^2+6xy=1
y=(1/6x)-(√3/2)x――@
正六角柱の体積V=(√3/4)x^2・6・y――A
@をAに代入。
V(x)=(√3/4)x^2・6{(1/6x)-(√3/2)x}
=(3√3)/2・x^2・y
=(3√3)/2・x^2・{(1/6x)-(√3/2)x}
=(√3/4)x-(9/4)x^3
V'(x)=(√3/2)-(27/4)x^2=0
x^2=(√3)/27のとき、
V(x)=(√3/4)x-(9/4)x^3
=(√3/4)x{1-3√3x^2}
=(√3/4)x(2/3)
=(√3)x/6
=√(√3)/18
≒0.0731152
0439132人目の素数さん
垢版 |
2018/05/22(火) 23:58:04.62ID:QxWTmuux
八面体の種類がいくつあるか自力で調べようとして挫折したので検索してみたところ、何をどう数えたのかは不明ながら257種類という数値が出てきた。これではまるでお手上げである。
なんの計算もしていないが、個人的には「デューラーの立体」ど呼ばれる三角形2枚、五角形6枚でできた立体が気になる。
0441イナ ◆/7jUdUKiSM
垢版 |
2018/05/23(水) 03:02:44.37ID:Rj3qNk6E
>>439
一辺xの正三角形1枚と正五角形3枚をサッカーボールみたいにたがいに百八十度回転させて噛み合わせるように貼りつける。
表面積S=1
八面体の各頂点から中心までの距離aは一意に決まる。正三角錐の底面同士は百八十度回転して平行。おそらく正五角錐の底面同士も百八十度回転して平行な位置にあるんじゃないかと。
八面体の体積V=正三角錐の体積×6+正五角錐の体積×2

正三角形同士の距離と対面する正五角形の距離は同じにできるのかな?
>>438
0442イナ ◆/7jUdUKiSM
垢版 |
2018/05/23(水) 03:07:00.40ID:Rj3qNk6E
>>441逆々。訂正。
八面体の体積=正三角錐の体積×2+正五角錐の体積×6
0443132人目の素数さん
垢版 |
2018/05/23(水) 05:25:56.47ID:bPXXYTiJ
>>439 >>441 >>442
 歪重角錐ですか?
 正8面体や正6角柱よりは改良すると思います。

             V / S^(3/2)
・メディアル8面体  0.074488       (4角形×4,5角形×4)
・歪重角錐      0.074217
・正8面体      0.07311522294   (アルキメデスの正プリズム)
・正6角柱      0.07311522294
・反プリズム     0.07311522294 = 1/{6√(3√3)}  (アルキメデス)

http://www.geocities.jp/ikuro_kotaro/koramu/684_eq3.htm
M. Goldberg: Tohoku Math. J.,40,p.226-236 (1935)
"The isoperimetric problem for polyhedra"

---------------------------------------

f = 4  正4面体      0.05170027 = 1/{6√(6√3)}
f = 6  立方体       0.06804138 = 1/(6√6)
f = 8  メディアル8面体 0.07311522 = 1/{6√(3√3)}
f = 12  正12面体     0.08168837
f = 20  メディアル20面体 0.0866101            (5角形×12,6角形×8)
球面に外接する?
0444132人目の素数さん
垢版 |
2018/05/23(水) 05:29:31.94ID:bPXXYTiJ
>>443 訂正

f = 4  正4面体      0.05170027 = 1/{6√(6√3)}
f = 6  立方体       0.06804138 = 1/(6√6)
f = 8  メディアル8面体  0.074488
f = 12  正12面体      0.08168837
f = 20  メディアル20面体 0.0866101  (5角形×12,6角形×8)
0445132人目の素数さん
垢版 |
2018/05/23(水) 08:38:50.14ID:cqx5U6TU
>>441
細かくてすまんが、どう考えても正五角形ではないよな>デューラーの立体の五角形
0446132人目の素数さん
垢版 |
2018/05/23(水) 09:21:27.45ID:cqx5U6TU
「歪重角錐」ってワード、検索してもikuro_kotaro氏しか使ってないようなのだけど、
もともとはどういう言葉の訳語?
言葉の印象から、デューラーの8面体とは別物のような気もするが。
0447132人目の素数さん
垢版 |
2018/05/23(水) 09:42:46.47ID:bPXXYTiJ
「メディアル f面体」
  [ 6-12/f ] 角形と [ 6-12/f ] +1 角形のみからなるf面体。
 f≧12 のときは 5角形×12,6角形×(f-12)

f = 8   0.074488  4角形×4,5角形×4
f = 10         4角形×8,4角形×2  (シリコンフラーレン)
f = 14   0.0833652  5角形×12,6角形×2  ねじれ重角錐台(ゴールドバーグ)
f = 20   0.0866101  5角形×12,6角形×8
f = 32         5角形×12,6角形×20  切頂20面体(サッカーボール)
f = 42         5角形×12,6角形×30  切稜12面体
 
0449イナ ◆/7jUdUKiSM
垢版 |
2018/05/23(水) 10:45:35.61ID:Rj3qNk6E
>>443正六角柱の体積の値があってたみたいでうれしいです。正八面体と同じだったとは。銅メダル二人みたいな。前>>442


歪重角錐か。歪んでるような気がしたんだよなぁ。するとその歪重角錐のさらにその上のビジュアル八面体とやらが0.074いくらで最大値か。
0450132人目の素数さん
垢版 |
2018/05/23(水) 11:04:54.26ID:SaS67Pru
ではそろそろ420の正解発表を聞こうか
0451132人目の素数さん
垢版 |
2018/05/23(水) 11:35:49.76ID:UmkZrt7x
正解ぷりーず
0452イナ ◆/7jUdUKiSM
垢版 |
2018/05/23(水) 11:51:08.00ID:Rj3qNk6E
今のところ>>438が最大。正八面体と同じだったことは残念だが。前>>449アンカー訂正。前々>>442前々の前>>441
精度を増すと、
V=√√3/18
≒0.0731152229


歪重角錐は今のところ言葉による想像にすぎないし、四角形と五角形を貼りつける八面体の存在も確認できない。
0453132人目の素数さん
垢版 |
2018/05/23(水) 12:10:18.05ID:zzKr5Jg7
四角形4枚と五角形4枚ってこんな感じ?
頂点数=12 (A〜Lとする)
五角形:ABCDE,DEFGH,GHIJK,JKLAB
四角形:AEFL,BCIJ,CDHI,FGKL
0454イナ ◆/7jUdUKiSM
垢版 |
2018/05/23(水) 14:10:25.08ID:Rj3qNk6E
>>453展開図を描いた。正方形ととなりあうのは正五角形3個と正方形1個。正五角形ととなりあうのは正五角形3個と正方形2個。たしかに存在しますね。
一辺xの正方形と正五角形から中心までの距離をそれぞれa、bとして、
V=(1/3)x^2・a・4+(1/3)(正五角形の面積)・b・4=1

一辺xの正五角形の面積がわからない。
(○+√5)/△
こんな感じだったような。
>>452
0455イナ ◆/7jUdUKiSM
垢版 |
2018/05/23(水) 14:38:08.07ID:Rj3qNk6E
>>454だんだん球体に近づくと考えて、
半径rの球の表面積S=4πr^2=1
r=1/2√π
半径rの球の体積V=(4/3)πr^3
=r/3
=1/6√π
≒0.0940316
0456132人目の素数さん
垢版 |
2018/05/23(水) 15:10:18.65ID:cqx5U6TU
>>454
だから、「正方形」とか「正五角形」ではなく
ただの「四角形」とか「五角形」だと何度言えば。

>たしかに存在しますね。
してません。

>>453
極大となるのが強い対称性を持つ場合だと仮定するなら

五角形CDEAB,FEDHG,IJKGH,LKJBAが互いに相似な
左右対称な五角形(CDEABであればEAの垂直二等分線が対称軸)であり、

四角形LAEF,FGKL,CBJI,IHDCが互いに相似な
等脚台形(LAEFであれば,LF//AE,LA=FE)

となるケースで考えればよいですかね。
五角形の形状が決まれば自動的に四角形の形状も決まります。
0457132人目の素数さん
垢版 |
2018/05/23(水) 15:15:29.02ID:/54JzC6H
>>420
正解をどうぞ
0459イナ ◆/7jUdUKiSM
垢版 |
2018/05/23(水) 16:07:07.76ID:Rj3qNk6E
>>455
ねじれでも腕ひしぎ逆十字でも、俺が出したこれは超えられまい。>>438
V=√√3/18
≒0.0731152229
0460132人目の素数さん
垢版 |
2018/05/23(水) 18:02:08.85ID:bPXXYTiJ
>>453

1辺がxの正6角形から1つの頂点を取り去った5角形4つを ∧∨∧∨ と並べて正方形柱にする。
辺がx,x√3 の長方形2枚で屋根を葺く。底も同様ですね。

このままだと 1/{6√(3√3)} つまり正8面体と同じ。
よって歪ませて正5角形に近づける?
0461132人目の素数さん
垢版 |
2018/05/23(水) 18:41:14.48ID:zzKr5Jg7
まず五角柱を考えて、その側面の四角形1枚に着目したとき、
上底面と下底面に接続する2辺それぞれに頂点を設けてそれらを結ぶと六角柱になります。
その代わりに、底面でない2辺上に頂点をそれぞれ設けてそれらを結ぶと五角形4枚+四角形4枚の立体になります。
そう考えると、後者の方がなんとなく球体に近い形にできそうなそうでないような…?
0462132人目の素数さん
垢版 |
2018/05/23(水) 21:49:27.54ID:XdPIqpjy
ところで「デューラーの立体」って平行六面体の反対の角を切り落としたものだね
0464132人目の素数さん
垢版 |
2018/05/24(木) 00:05:56.52ID:ksY6GGNA
対称性のあるメディアル8面体を一般化するため
>>453 に合わせて、実際の空間座標を設定してみた。パラメータはa,b,rの3つ。
これで実際に表面積と体積を計算して、最大になるケースを求めればよい。
A(1+a,1-a,-b),B(1-a,1+a,b),C(1-ar,0,br),D(1-a,-1-a,b),E(1+a,-1+a,-b),
F(0,-1+ar,-br),G(-1-a,-1+a,-b),H(-1+a,-1-a,b),I(-1+ar,0,br),
J(-1+a,1+a,b),K(-1-a,1-a,-b),L(0,1-ar,-br)

ただし,パラメータは
-1<a<1,b>0,r>1,ar<1
を満たす範囲で動く。

実際には,0<a<1の範囲を考えればいい気はする。
あとで暇なら計算するが,だれかやって。

なお、線分BA,ED,HG,KJの中点が,xy平面上の原点を中心とする1辺2の正方形をなすように
配置してます。
0466132人目の素数さん
垢版 |
2018/05/24(木) 02:26:45.42ID:iiG4vaf/
>>462

ねじれ双3角錐(切り落とす前)の例
8つの頂点
 ±(0,0,3c/√12)
 ±(2a/√6,0,c/√12)
 ±(-a/√6,a/√2,c/√12)
 ±(-a/√6,-a/√2,c/√12)

辺の長さL = √{(2aa+cc)/3}
体積V = aac,
表面積S = 6LL・sinβ = 2a√{3(aa+2cc)},
  β = arccos((cc-aa)/(2aa+cc))
  V / S^(3/2) ≦ 1/(6√6),
等号は a=c のとき (立方体)
0467132人目の素数さん
垢版 |
2018/05/24(木) 06:17:07.64ID:iiG4vaf/
>>466
 頂点から k・L まで(Lは辺長、0<k<1) の正3角錐を切り落とす。
 底辺:(√2)ak,底面積:(√3 /2)aak^2,高さ: (1/√3)ck,体積:(1/6)aac・k^3,
 表面積の減少:(1/2)(√3){√(aa+2cc) -a}ak^2,
 V(k) = V(0) - (1/3)aac・k^3
 S(k) = S(0) - (√3){√(aa+2cc) -a}ak^2,
0468132人目の素数さん
垢版 |
2018/05/25(金) 06:34:50.00ID:ohjGIEVt
>>466 >>467

V/S^(3/2) が最大となるのは、
k = 1(反正3角柱、アルキメデスの反プリズム)でかつ
c = 2a,β = 60°のとき。
一辺 L = (√2)a の正4面体を切り落とす。残ったのは一辺 L の正8面体か。

>>462 が正しいなら、 >>439 もハズレのような。。。
0471132人目の素数さん
垢版 |
2018/05/26(土) 05:58:03.55ID:Tm+bfCXy
>>464
計算した。

AE = DH = GK = JB = 2(1-a),
BD = EG = HJ = KA = 2(1+a),
CI = FL = 2(1-ar),
CI〜DH,CI〜JB の距離 √{(1+a)^2 + bb(r-1)^2}

5角形ABCDE = {4 + (r-1)(1+a)}√(aa+bb),

4角形CDHI = {2-(r+1)a}√{(1+a)^2 + bb(r-1)^2}

S = 4{4 + (r-1)(1+a)}√(aa+bb) + 4{2 - (r+1)a}√{(1+a)^2 + bb(r-1)^2}

V = 8(1-aa/3)b + 4(1+a)b(r-1){1-(2+r)a/3},

・a = 0,b = 1/√3,r = 2 のとき
 S = 12√3, V = 4√3, V/S^(3/2) = 1/{6√(3√3)},    >>460

・a = 0.1035  b = 0.379  r = 3.180 のとき
  S = 18.7092102  V = 6.0163648  V / S^(3/2) = 0.074344865

 やっと正8面体、正6角形、アルキメデス、デューラー etc を超えた。。。
0473イナ ◆/7jUdUKiSM
垢版 |
2018/05/26(土) 12:17:34.10ID:IfNt0hdl
>>438これ、正解でいいと思うんだけど、ビジュアル八面体とやらが、メディアルか、が座標設定して計算で最大値を更新したのは確からしいな。
ただ、V≒0.074をどうやって出したかまだわからない。
‖∩∩‖V/S^(3/2)の
((-_-)3/2ってなんだ?
(っц)~  V/S√Sか。
「 ̄ ̄ ̄]前>>469
0474132人目の素数さん
垢版 |
2018/05/26(土) 12:25:25.83ID:p7ZlenKz
そろそろ正解が聞きたいなぁ。いろんな計算結果は上がってるけどあくまで数学なんだからそれは正解にはなり得ない。意味ないわけではないけど。
0475イナ ◆/7jUdUKiSM
垢版 |
2018/05/26(土) 12:34:00.52ID:IfNt0hdl
そもそも題意はS=1だろ。S=1のときVがいくらになるかを求める問題だったはず。
>>438これが正解だ。
‖∩∩‖
((-_-)
(っц)~
「 ̄ ̄ ̄]前>>473
0476132人目の素数さん
垢版 |
2018/05/26(土) 12:47:38.91ID:Zk6GPK3+
まさか、正解を用意していなかったとかあるまいな
0477132人目の素数さん
垢版 |
2018/05/26(土) 13:32:39.72ID:p7ZlenKz
最初の頃は出題者らしき人がのレス付いてたけど途中から出てこなくなってるから、ちょっと危ない感じもするけど。
0478132人目の素数さん
垢版 |
2018/05/26(土) 13:51:29.80ID:idqdAluV
自分は出題者ではないし>>443でもないが
正解発表という意味では、
>>443 >>444あたりで紹介されてる
http://www.geocities.jp/ikuro_kotaro/koramu/684_eq3.htm
に記述のある
ゴールドバーグが見つけたメディアル多面体での
0.074488
ってのが現時点でのチャンピオンデータってことなんでしょ?
で、ikuro_kotaro氏の書き方も若干曖昧でゴールドバーグの論文を読んでみないと
本当のところはわからないけど、おそらくそれはまだ局所最適解に過ぎず
すべてのケースをくまなく調べたわけではないから、8面体についても未解決で、
ただゴールドバーグは一般にn面体についても
メディアル多面体において最大値をとると予想してる、って話だよね?

それが現時点での最大の結果でしょ?
それ以上の結果が出たらこんなところに書いてる場合じゃなくて論文を書くべき案件。

別に、答えが用意されてるパズルだけが「面白い問題」じゃないよね。
普通に思いつくところが正解ではなくて、まだ正解の探索の途中である問題だって
面白い問題には違いないのだから、それでいいじゃん。
0479132人目の素数さん
垢版 |
2018/05/26(土) 13:57:49.09ID:idqdAluV
ちなみに、V/S^(3/2)の意味を理解せずに議論に参加してるつもりの人がいるようだが
ある8面体の体積がV、表面積がSのとき、
その8面体と相似で表面積が1の立体の体積がV/S^(3/2)になる。
http://www.geocities.jp/ikuro_kotaro/koramu/684_eq3.htm では
V/S^(3/2) が最大、ではなく S^3/V^2 が最小という言い方をしてるが、同じこと。
>>443にある値は、そこに紹介されてるS^3/V^2の値をV/S^(3/2)に換算してるだけ。
0480132人目の素数さん
垢版 |
2018/05/26(土) 14:06:15.01ID:idqdAluV
>>471
お疲れさまです。ちゃんと、他のすぐ計算できるケースを超えるポイントが見つかったのですね。
SとVの式は自分も計算してみて同じ結果になりました。

ゴールドバーグはこんな計算から局所最適解を求めたのだろうけど
1935年だから、計算機による数値計算ではなくおそらく手計算だよな。
どうやったんだ…
0481132人目の素数さん
垢版 |
2018/05/26(土) 14:30:57.93ID:p7ZlenKz
別に答えが用意されてようが何だろうがそれはかまわないけど、それならそれでそれは明示しとかんとダメだと思う。
0482132人目の素数さん
垢版 |
2018/05/26(土) 16:37:28.91ID:N2EQPiGo
>>479
その辺のメトリックを理解しない数学徒は皆無だろう
理解しない非徒は説明されても理解しない可能性が大
0483イナ ◆/7jUdUKiSM
垢版 |
2018/05/26(土) 17:10:10.91ID:IfNt0hdl
立方体の一辺をx、切りとる二つの直角三角錘の二辺と高さをaとする。
八面体の表面積Sについて、
S=6x^2-3a^2+{(√3)/4}(a√2)^2・2=1
6x^2-1=3a^2-a^2・√3
a=√{(3+√3)(x^2-1/6)}――@
一辺xの立方体から一辺aの直角三角錘2個を引く。
八面体の体積V=x^3-2(a^3)/6
V=x^3-(a^3)/3――A
@をAに代入。
V(x)=x^3-(1/3)(3+√3)(x^2-1/6)√{(3+√3)(x^2-1/6)}
V'(x)=3x^2-(2x/3)(3+√3)√{(3+√3)(x^2-1/6)}-(1/3)(3+√3)(x^2-1/6)√(3+√3)=0
を満たすxにより、
V(x)=x^3-(1/3)(3+√3)(x^2-1/6)√{(3+√3)(x^2-1/6)}
=x^3-(1+1/√3)(x^2-1/6)√{(3+√3)(x^2-1/6)}

≒?>0.0731152229
>>475
0484132人目の素数さん
垢版 |
2018/05/26(土) 19:41:49.72ID:ypScq2Bz
せめて「とある法則」ってのだけは教えてほしいわな

おそらく七角錐,正八面体はダメで六角錐は除外できないってことから
各頂点に接している面の数が3の多面体ってことなんだろうけど
0485イナ ◆/7jUdUKiSM
垢版 |
2018/05/26(土) 21:31:07.58ID:IfNt0hdl
F'(x)=3x^2-(2x/3)(3+√3)√{(3+√3)(x^2-1/6)}-(1/3)(3+√3)(x^2-1/6)√(3+√3)=0
3x^2=(2x/3)(3+√3)√{(3+√3)(x^2-1/6)}+(1/3)(3+√3)(x^2-1/6)√(3+√3)
>>483辺々を二乗すると、
9x^4=(4/9)x^2・(12+6√3)・(3+√3)(x^2-1/6)+(1/9)(12+6√3)・(x^2-1/6)^2・(3+√3)+2(2x/3)(3+√3)√{(3+√3)(x^2-1/6)}(1/3)(3+√3)(x^2-1/6)√(3+√3)

9x^4=(4/9)x^2・(36+18+30√3)・(3+√3)(x^2-1/6)+(1/3)(4+2√3)・(x^2-1/6)^2・(3+√3)+(4x/9)(3+√3)√{(3+√3)(x^2-1/6)}(3+√3)(x^2-1/6)√(3+√3)
81x^4=12(18+10√3)・(3+√3){x^4-(1/6)x^2}+3(4+2√3)・(x^2-1/6)^2・(3+√3)+4x(3+√3)√{(3+√3)(x^2-1/6)}(3+√3)(x^2-1/6)√(3+√3)
81x^4=24(7+4√3){6x^4-x^2}+3(4+2√3)・(x^2-1/6)^2・(3+√3)+4x(3+√3)√{(3+√3)(x^2-1/6)}(3+√3)(x^2-1/6)√(3+√3)

大変。
0487 【大吉】
垢版 |
2018/05/27(日) 00:45:56.58ID:UhzuItQI
F'(x)=0でF(x)の最大値を出す法則と四則演算ならわかる。
>>485

144・7-81+576√3)x^4-24(7+4√3)x^2+3(4+2√3)・(x^2-1/6)^2・(3+√3)+4x(3+√3)√{(3+√3)(x^2-1/6)}(3+√3)(x^2-1/6)√(3+√3)=0

(927+576√3)x^4-(168+96√3)x^2+(54+30√3)(x^2-1/6)^2+4x(3+√3)√{(3+√3)(x^2-1/6)}(3+√3)(x^2-1/6)√(3+√3)=0
0488132人目の素数さん
垢版 |
2018/05/27(日) 01:33:00.38ID:i5aSKt1a
>>483 >>485

 辺長xのうち、頂点からaまでの部分を切り取るのだな。
 V = x^3 - (a^3)/3,
 S = 6xx - (3-√3)aa,

a = {(3-√3)/2}x = 0.6339746x のとき最大で

 V/S^(3/2) = (1/6)√{(1+√3)/15} = 0.0711291315

>>467 で a = c,β = 90゚,L = x の場合でござるな。
0489132人目の素数さん
垢版 |
2018/05/27(日) 02:03:28.95ID:aeCCXXNU
>>471
自分でもプログラムで探してみたけど、
>>464の設定ではそのあたりが限界なのね…
自分の結果は
(a, b, r) = (0.103402, 0.379226, 3.177760) で 0.074344868
S^3/V^2でいうと180.92476とかで、
ゴールドバーグの結果と言われてる180.23とはまだ随分ギャップがあるなあ。
それに近づくには、>>464の対称性を崩さないといけないということ?
(まあ、その値が正しいかどうかもよくわからないが)
0491イナ ◆/7jUdUKiSM
垢版 |
2018/05/27(日) 02:40:09.42ID:UhzuItQI
(927+576√3)x^4-(168+96√3)x^2+(54+30√3){x^4-(1/3)x^2+1/36}+4x(3+√3)√{(3+√3)(x^2-1/6)}(3+√3)(x^2-1/6)√(3+√3)=0

(773+546√3)x^4-(186+106√3)x^2+(9+5√3)/6+4x(3+√3)√{(3+√3)(x^2-1/6)}(3+√3)(x^2-1/6)√(3+√3)=0
>>487
もっかい紙の上でやったほうがいいみたい。x出したいわけじゃないし。そうか、aがxの半分超えるぐらいおっきなることもあるんか。
0492132人目の素数さん
垢版 |
2018/05/27(日) 03:46:25.26ID:i5aSKt1a
>>480

>>443 の文献はここら辺↓に…
http://www.jstage.jst.go.jp/article/tmj1911/40/0/40_0_226/_pdf

Fig.1 の VIII の欄では >>453 >>457 >>460 と同じ配置

Table 2 で K は等周定数 (S^3 /V^2)
n=8,VIII の欄はたしかに K = 180.23
また傾き角(軸となす角)は 53゚07',15゚23'

>>489
 >>464 のような高い対称性をもつか不明でござる。(英語不得手により)
0493132人目の素数さん
垢版 |
2018/05/27(日) 04:35:27.73ID:i5aSKt1a
>>483

S = 6xx - (3-√3)aa = 1
から
aa = (3+√3)(xx -1/6)  … (1)

V(x) = x^3 -(1/3)a^3  … (2)
 = x^3 - (1/3)(3+√3)^(3/2)・(xx -1/6)^(3/2),

V '(x) = 3xx - x(3+√3)^(3/2) x√(xx -1/6) = 0,
xで割って
 3x = (3+√3)^(3/2) √(xx-1/6),
 9xx = (3+√3)^3 (xx-1/6),
 xx = (√3 +1)/15,
これを (1) に入れて
 aa = (√3 -1)/10,
>>490 が出る。
0494132人目の素数さん
垢版 |
2018/05/27(日) 04:52:26.77ID:aeCCXXNU
>>492
実は自分も今その論文を眺めてたところ。

>>489で求めた値は、最大になるようなa,b,rの値の組を最初粗い格子点の中から探し
その周辺でさらに細かい格子点の中から探し、というような作業を
スクリプトを使って繰り返して(範囲の設定は手作業)
その精度での局所最適解を求めたのだけど、
その作業をすり抜けるような特異点が存在するとも思えないし、
実際そのa,b,rから各面の傾きを計算すると
その論文の値とほぼ一致するし。
本来は自分の計算の方を疑うべきなのだろうが、
>>471氏の計算とも合致してるので、
今は180.23という値だけが何か間違っているという疑いの方が強まってる。

論文に載ってる2つの角度だけではその8面体の形状は特定できないので
もっと詳しく書いておいてくれればよかったのに>ゴールドバーグ氏
ネットで検索しても、その立体の展開図みたいなものは見つかるのだが、
細かいサイズや実際のS^3/V^2の値とかの定量的な話が全然書いてないんだよな
0495132人目の素数さん
垢版 |
2018/05/27(日) 11:27:35.29ID:CGYiTgTM
この手のはいくらでも先に進めるけれど進んだところで意味が無い計算の1つ
数学の袋小路
これが役に立つ例を他の学問分野から必要とされない限り
よくできましたで賞にしかならない
0496132人目の素数さん
垢版 |
2018/05/27(日) 11:58:20.93ID:6sMTwTbT
4色問題もそうだし、おおよその整数野未解決問題もそうなんだよなあ
0497イナ ◆/7jUdUKiSM
垢版 |
2018/05/27(日) 19:52:43.78ID:UhzuItQI
>>491
F(x)=x^3-(1+1/√3)(x^2-1/6)√{(3+√3)(x^2-1/6)}
もしやただ単にx^2=1/6のときF(x)は最大とかいう話?
F(1/√6)=1/(6√6)
≒0.0680413817

こんな簡単でいいの?
0498イナ ◆/7jUdUKiSM
垢版 |
2018/05/27(日) 20:03:51.93ID:UhzuItQI
>>497ちがうか。やっぱり>>438でF'(x)=0で最大値が出たことを思うと、
F(x)=x^3-(1+1/√3)(x^2-1/6)√{(3+√3)(x^2-1/6)} これもF'(x)=0でできんかなと思うんだよ。
F(x)=?
≒0.074
0500イナ ◆/7jUdUKiSM
垢版 |
2018/05/28(月) 20:06:22.78ID:DLDfn9F5
>>490式と計算過程も書いてほしいよ。
>>499
S=6{x^2-(a^2/2)}+2(√3/4)(a√2)^2
=6x^2-3a^2+√3・a^2=1
(3-√3)a^2=6x^2-1

a^2=(6x^2-1)/(3-√3)
=(x^2-1/6)(3+√3)

V=x^3-2(1/3)(a^2/2)a
=x^3-a^3/3
=x^3-(a/3)(x^2-1/6)(3+√3)
=x^3-(x^2-1/6){1+(√3)/3}a
=x^3-(x^2-1/6){1+(√3)/3}√{(x^2-1/6)(3+√3)}
V'(x)=3x^2-(1/3)(2x)(3+√3)√{(x^2-1/6)(3+√3)}+(1/3)(x^2-1/6)(3+√3)√(3+√3)x=0
3x^2-(2x){1+(√3/3)}√{(x^2-1/6)(3+√3)}+(1/3)(x^2-1/6)(3+√3)√(3+√3)x=0
3x^2-(2x){1+(√3/3)}√{(x^2-1/6)(3+√3)}+(1/3)(x^2-1/6)(3+√3)√(3+√3)x=0


{x-(1/√6)}^2=x^2-2x/√6+1/6=x^2-1/6-2x/√6+1/3

x^2-1/6={x-(1/√6)}^2+
2x/√6-1/3


(休憩)
0501イナ ◆/7jUdUKiSM
垢版 |
2018/05/29(火) 02:49:24.32ID:2oLTtdTc
>>488
V=0.0711……だったら0074どころか、正六角柱の0073より小さいじゃないか!!
>>500
感覚的に正六角柱には及ばないと思ったんだよ。計算しようとして、できたわけじゃないけど、無駄な計算だった。
じゃああれだ、大御所、五角形4つと四角形4つで微分、お願いします。
数式で出した最大値はいまだ正六角柱の0.073……ですもんで。
0502イナ ◆/7jUdUKiSM
垢版 |
2018/05/29(火) 09:22:46.22ID:2oLTtdTc
>>501
長方形4つの長辺およびホームベース形五角形4つの上辺をxとし、今仮に五角形4つの上辺と向かいあう内角を60°として垂直に柱を建て屋根と同じ形状のものを地下に天地逆で90°水平回転で作る。
S=x(x/√3)4+{x(x/√3)+(x/2√3)(x/2)}・4
(4/√3)x^2+(4/√3)x^2+(x^2)/√3=1
9x^2/√3=1
x^2=(√3)/9
V=x^2{(x/√3)+(x/2√3)}
=(√3)/9{(x/√3)+(x/2√3)}
{(√3)/9}・3x/(2√3)
=x/6
=√(√3)/6・3
=√(√3)/18
=0.0731152……
このゴキブリホイホイを切って天地逆にくっつけたような無用の立体は、先の正六角柱と同値。

さらに体積を増すには、五角形の角度を60°より大きくする手が考えられる。たとえば72°は無理かもしれないが、五角形を正五角形に近づけるべく柱を傾けてはどうか。つまり柱も梁も棟もすべて同じ長さにしたとき体積は最大になるんじゃないか。仮説です。
0503132人目の素数さん
垢版 |
2018/05/29(火) 15:00:58.00ID:n11ck1yy
>>502

五角形の内角が 120゚×3、90゚×2 で4辺の長さが (x/√3)
長方形の辺長が x と (x/√3)

>>460 を 1/√3 倍に縮小したものと同じですね。

頂点を垂直方向にずらすだけでは、それ以上改良しないと思われ…
0504132人目の素数さん
垢版 |
2018/05/29(火) 16:16:31.38ID:RTi38Ocg
私はやってないのでいう権利ないかもしれないけど、ともかくこれだけ頑張って計算してる人いるんだから、間違ってないならそろそろWとある法則”上げてもいいんじゃね?なんか計算の足しになるかもしれないし。
0505イナ ◆/7jUdUKiSM
垢版 |
2018/05/29(火) 16:19:46.35ID:2oLTtdTc
>>502
五角形の内角を72°として斜めに柱を突きだし屋根と同じ形状のものを地下に天地逆で90°水平回転で作る。一辺xの正五角形と三辺xの四角形を貼りあわせた八面体の鳥瞰図を描く。
四角形の残りの一辺y(棟と地下のねじれの位置にある一辺も同じく)はやや小さく(y<x)、四角形は面積(x+y)x/2の台形。
S=4(x+y)x/2+4{(1+√5)/2}x^2=1
2x^2+2xy+2(1+√5)x^2=1
2xy+2(2+√5)x^2=1
2xy=1-2(2+√5)x^2
y={1/(2x)}-(2+√5)x

上の棟から真っ二つに切った七面体V/2(上辺t=y→x)
五角形の辺はt一つ、{(1+√5)/2}x二つ、x二つ。
V/2
=∫(t=y→x)



=
>0.0731152……


ちょっと超えるはず。五角形の内角を72°にできれば、八面体V=0.074……
0506132人目の素数さん
垢版 |
2018/05/29(火) 18:12:11.36ID:n11ck1yy
>>471>>489 の結果から、(AE + BD)/2 = x として

五角形ABCDEは
 ∠A = ∠E = 104.73844゚,∠B = ∠D = 113.06566゚,∠C = 104.3918゚ (正五角形: 108゚)
 AB = DE = 0.406444x,BC = CD = 0.69826x,AE = 0.89660x,BD = 1.10340x
 S_5 = 0.62920xx

台形CDHIは
 CI = FL = 0.67141x,CI〜DH 0.68912x
 ∠C = ∠I = 99.2793゚,∠D = ∠H = 80.7207゚
S_4 = 0.54027xx

合計で
 S = 4.6779xx
 V = 0.75219x^3

>>464>>471 の設定は x=2 です。

計算はチラシの裏でもできますが…
0507イナ ◆/7jUdUKiSM
垢版 |
2018/05/29(火) 19:01:38.40ID:2oLTtdTc
―/ ̄ ̄ ̄/\
_/____/ |
‖ ̄ ̄ ̄‖ |
‖   ‖/~~~~
~\  /~~~~~
―-\/~~~
>>505訂正。72°⇒108°
0508イナ ◆/7jUdUKiSM
垢版 |
2018/05/29(火) 22:07:16.16ID:2oLTtdTc
>>507訂正。>>502角度を60°⇒内角を120°おもしろい問題でしたね。0.074は自力では綺麗に決まりそうにないですが、0.073が二通りも示せたことをうれしく思います。
□ ‖◇/n_n__n n___
。 ‖>// ̄ ̄ ̄ ̄ ̄ ̄//
_∩∩//______//|
( (`)(-^-)( -~-)zz..
(っц)_U_Uzz.````_/|_/
_|υυ ̄ ̄ ̄ ̄ ̄ ̄|_/_/_/_/_/_/_/_/_/_/_/_/_/_/_/_/_/_
0509イナ ◆/7jUdUKiSM
垢版 |
2018/05/30(水) 02:31:18.23ID:HVx7wYPO
一辺xの正方形4つと正五角形4つをおじゃみか玉入れの玉のように互い違いに編んだ八面体の表面積は、
>>508
4x^2+4{(1+√5)/2}x^2=1
(6+2√5)x^2=1
(1+√5)^2・x^2=1
x={(√5)-1}/4≒0.309……
V(x)=
八面体を正四角錘4つと正五角錐4つに分解したい。
V'(x)=0
0510132人目の素数さん
垢版 |
2018/05/30(水) 05:32:35.18ID:ddqYu1Wl
>>505

一辺xの正五角形では、
BD = φx,
CI = FL = x/φ,
5角面の傾き: arctan{1/√(2φ^3)} = 18.9607099°
となる。ここに、
 φ = (1+√5)/2 = 1.61803399  (黄金比)
 sin(18゚) = (φ-1)/2,   cos(18゚) = (1/2)√(2+φ),
 sin(36゚) = (1/2)√(3-φ), cos(36゚) = φ/2,

S = 4(S_5 + S_4)
 = {5φ/√(φ+2) + √(3φ-1)} φxx
 = 4(1.7204774 + 0.7941257) xx
 = 10.0584124 xx,

V = {(7φ+4)/12}√(2φ) x^3
 = 2.297540285 x^3,

V / S^(3/2) = 0.072022630

う〜む
>>506 を見ると、内角は108゚に近いが、辺長は不均衡(AB と DE が短く、AE が長い)
 辺長を変えれば改善するか?
0511132人目の素数さん
垢版 |
2018/05/30(水) 06:12:08.84ID:ddqYu1Wl
>>510

BC = CD = x
BD = φx,
CI と DH,BJ の距離 (1/2)√(3φ-1) x,
CI と BDHJ の高低差 x/√(2φ),
5角面の傾き: arctan{1/√(2φ^3)} = 18.9607099°
>>510 のままで

AB = DE = x-φz,
AE = x+z,
CI = FL = x/φ + z, ( =y )
BDHJ と AEGK の高低差 (x-φz)√(φ/2),
とすると…

>>505
 台形の面積は (1/4)(x+y)^(3/2)・√(3x-y) ...
0512132人目の素数さん
垢版 |
2018/05/30(水) 07:12:00.25ID:6H7MnT2U
コンピュータを使うなら以下のような方式はどうか
(1)8枚の面の緒元(法線ベクトルと原点からの距離)をランダムに決める
(2)各面の微小変異(傾きと距離)に対する評価関数(V^2/S^3など)の変量を計算し、最も変量の大きい面について評価関数が最良になるよう調整する
(3)手順(2)を繰り返す

面積は交線で囲まれる図形から、体積は面積×原点距離÷3の総和、ってことで機械的に求められるのではないか
0514132人目の素数さん
垢版 |
2018/05/30(水) 09:56:31.72ID:ddqYu1Wl
>>511

S(x,z) = 4(S_5 + S_4)
 = √(2+φ)・{(φ+2)xx -2φxz +φzz} + √(3φ-1)・(φx+2z)x,

V(x,z) = (1/6)√(φ/2)・{(5φ+2)xx + (2φ^3)xz +zz}(x-φz) + √(φ/2)・{(φφ/3)x + z}xx,

最大となるのは z = 0.2509325x のときで
 BC = CD = x,
 AB = DE = 0.5939827x,
 AE = 1.2509325x,
 CI = 0.8689665x,
 S = 4 (S_5+S_4) = 4 (1.2858838 + 1.0404394)xx = 9.30529256xx,
 V = 2.104005x^3,
 V / S^(3/2) = 0.07412278177

>>471 >>489 には及ばないが、正8面体、正6角柱、などは超えた…
0515132人目の素数さん
垢版 |
2018/05/30(水) 10:09:10.39ID:ddqYu1Wl
>>514 訂正スマソ

S(x,z) = 4(S_5 + S_4)
 = √(2+φ)・{(φ+2)xx -2φxz −φzz} + √(3φ-1)・(φx+2z)x,


なお、以前の解( >>471 >>489 >>506 ) で BC=CD=1 とすると
AB = DE = 0.58208, AE = 1.28405 となる。。。
0516イナ ◆/7jUdUKiSM
垢版 |
2018/05/30(水) 17:10:49.78ID:HVx7wYPO
一辺xの正五角形の面積
(x^3)(√(25+10√5)}/4
>>509
三辺x、一辺(√5-1)/4の台形の面積
{x+(√5-1)x/4}・xsin18°・(1/2)
={(3+√5)/4}x^2・(√5-1)/4・(1/2)
(√5+1)/16・(x^2)

S=(x^3){√(25+10√5)}+(√5+1)(x^2)/4=1

V(x)=
V'(x)=0

台形が2つずつとなりあってるのに対し、正五角形は4つ連なってる。不思議な美しさ。
0517イナ ◆/7jUdUKiSM
垢版 |
2018/05/30(水) 17:39:59.82ID:HVx7wYPO
>>516
体積V(x)の八面体の天地を90°ねじれの位置にある短い棟として、台形の長さxの三辺のうちの真ん中の一辺で水平に切る。上中下3つの物体は上と下がまったく同じかたちで、V=V
2つの断面はx×(√5+1)/2の長方形である。
すべての辺がxの関数で表され、正五角形が3次、台形が2次、V(x)は高々4次、微分してV'(x)=0で3次方程式が出て、V(x)=
雨だ――……
0519132人目の素数さん
垢版 |
2018/05/31(木) 03:44:32.13ID:eo/xqWlC
そろそろ次の問題出していい?
0520132人目の素数さん
垢版 |
2018/05/31(木) 11:12:45.39ID:1i3xzGBS
三辺x、一辺(3-√5)x/2の台形の面積

{x + (3-√5)x/2}・x cos(18゚)・(1/2)
= {(5-√5)/2}xx・√(10+2√5)/4・(1/2)
= √(50-10√5)/8・xx
= 0.6571639 xx,
0521イナ ◆/7jUdUKiSM
垢版 |
2018/05/31(木) 12:03:08.39ID:oQbVMAkg
>>520台形の一辺が違ってましたか?
一辺xの正五角形の面積
(x^3)(√(25+10√5)}/4
三辺x、一辺(√5-1)x/4の台形の面積
台形の一辺は(3-√5)x/2ですか――。
>>518
0522132人目の素数さん
垢版 |
2018/05/31(木) 12:22:47.56ID:LSNoQXxv
拘束条件がなさすぎて数学の問題としてやるには難しすぎるし、そういうアプローチをしてる人もいないし無意味
0523イナ ◆/7jUdUKiSM
垢版 |
2018/05/31(木) 14:11:34.74ID:oQbVMAkg
一辺xの正五角形4つと、
三辺x、一辺xよりやや小さい台形4つで、表面積が1で、
>>521
八面体の体積出して微分して=0にして、体積0.074が出ればそれでじゅうぶんです。
もっと大きな八面体があるいは存在するかもしれませんが、一辺xの正五角形4つと、三辺x、一辺xよりやや小さい台形4つで、表面積が1のやつは一意に決まると思うもんで、それがまずは知りたい。
この問題、競りのようでおもしろい。自分が最高値を出すのを楽しみにしてる。
0525132人目の素数さん
垢版 |
2018/05/31(木) 15:04:47.94ID:fih1epUm
>>519
少し遡れば解かれずに放っとかれてる問題ある程度あるし気にせず出しちゃえ
0526132人目の素数さん
垢版 |
2018/06/01(金) 22:29:12.26ID:ISxYUmxo
左右の次数が一致しない漸化式(例えばa_(n+1)=(a_n)^2+1)は一般には解けないが、初項を置き換えるとうまくいくことがある。

(1)
-1≦a_1=A≦1
a_(n+1)=2(a_n)^2-1
の一般項を求めよ。

(2)
b_1=B∈R
b_(n+1)=(b_n)^2-2
の一般項を求めよ。
0528132人目の素数さん
垢版 |
2018/06/02(土) 01:38:49.70ID:L/4yyydP
>>526

(1) a_n=cosθ[n] と置くと cosθ[n+1]=cos(2θ[n])
この解は θ[n]=θ[1]*2^(n-1),
a_n=cos(2^(n-1) arccos A)

Wlfram Alphaの解答 http://www.wolframalpha.com/input/?i=RSolve%5Ba%5Bn%2B1%5D%3D%3D2*a%5Bn%5D%5E2-1,a%5Bn%5D,n%5D


(2) (1)と同様に b_n=e^(z[n])+e^(-z[n]) と置くと
b_n=α^(2^(n-1))+β^(-2^(n-1)), (α,β=(B±√(B^2-4))/2)

Wlfram Alphaの解答 http://www.wolframalpha.com/input/?i=RSolve%5Bb%5Bn%2B1%5D%3D%3Db%5Bn%5D%5E2-2,b%5Bn%5D,n%5D


類題:√Xの開平計算で使うNewton法 x_0=1, x_(n+1)=((x_n)^2 +X)/(2x_n)
の一般項はx_n=coth(y_n)と置くと
x_n=(√X){(1+√X)^(2^n) + (1-√X)^(2^n)}/{(1+√X)^(2^n) - (1-√X)^(2^n)}

Wlfram Alphaの解答 http://www.wolframalpha.com/input/?i=RSolve%5Bx%5Bn%2B1%5D%3D%3D(x%5Bn%5D%5E2-X)%2F(2x%5Bn%5D),x%5Bn%5D,n%5D
0529132人目の素数さん
垢版 |
2018/06/02(土) 02:27:05.94ID:G2b8XSzz
(2) は 両辺2で割って a_n = (1/2)b_n とおいて(1)使えばよいのでわ?
0530イナ ◆/7jUdUKiSM
垢版 |
2018/06/02(土) 05:51:13.38ID:zaslUUou
正五角形4つの各辺と台形4つの三辺をxとし、台形どうしの接する屋根の棟および逆屋根の下端に位置する辺をy(x>y)とすると、
>>523(図は省略)
y^2-2xy+(38/√5-19)x^2=0
y={1-√(20-38/√5)}x
棟の長さがxで表せた。
あとは体積。
V(x)=
V'(x)=0
上棟するには微分するしかない。
0531イナ ◆/7jUdUKiSM
垢版 |
2018/06/02(土) 08:38:40.89ID:zaslUUou
y^2-2xy+(38/√5-19)x^2=0
>>530おかしい。
y=x-√{x^2-(38/√5-19)x^2}
=x-x√(20-38/√5)
={√(20-38/√5)-1}x
=0.7337482……・x
台形の接合線yは、xの7割ぐらいだとは思うんだが。
V(x)をどうやって出すか。V(x)=y/10ぐらいだと理想的。
0532132人目の素数さん
垢版 |
2018/06/02(土) 08:46:00.26ID:qc99k5Fr
>>528

√a の開平計算で使うNewton法は

x_{n+1} = x_n - 2x_n {(x_n)^2 -a}/{3(x_n)^2 +a}

  = x_n {(x_n)^2 +3a}/{3(x_n)^2 +a}

でござる。これの一般項も出せぬか??
0533132人目の素数さん
垢版 |
2018/06/02(土) 11:37:24.31ID:L/4yyydP
>>352

cothの3倍角の公式:coth3x=(coth x)(coth^2x + 3)/(3coth^2x+1)から
x_n=(√a)coth z_n と置くと coth z_{n+1} = coth 3z_n
∴ z_n=3^n z_0,
x_n=(√a)coth(3^n arccoth(x_0/√a))
0534132人目の素数さん
垢版 |
2018/06/02(土) 11:51:16.40ID:L/4yyydP
>>532 の間違い。

一般にp次収束するNewton法は、(収束先)*[1+(p^nの指数関数)]という形になり、
一般項が簡単な式であらわされる場合があります。
0535132人目の素数さん
垢版 |
2018/06/02(土) 16:54:14.55ID:ItLI/UY3
某botで唯一☆12(Legend)を付けられている問題

a,b,c,dが正のとき
(a-b)(a-c)/(a+b+c)
+(b-c)(b-d)/(b+c+d)
+(c-d)(c-a)/(c+d+a)
+(d-a)(d-b)/(d+a+b)
≧0
を示せ。

模範解答は3つあるが、いずれもエレガントな解き方ではない
0536132人目の素数さん
垢版 |
2018/06/02(土) 16:56:52.26ID:ItLI/UY3
ちなみに縮小(拡張の反対)したバージョン2通り

(1)
(a-b)(a-c)/(a+b+c)
+(b-c)(b-a)/(a+b+c)
+(c-a)(c-b)/(a+b+c)
=(a^2-ac+b^2-ba+c^2-cb)/(a+b+c)
=(1/2)(2a^2-2ac+2b^2-2ba+2c^2-2cb)/(a+b+c)
=(1/2)((a-b)^2+(b-c)^2+(c-a)^2)/(a+b+c)
≧0
等号成立はa=b=c

(2)
(a-b)/(a+b)
+(b-c)/(b+c)
+(c-d)/(c+d)
+(d-a)/(d+a)
≧0?

(a-b)/(a+b)
+(b-c)/(b+c)
+(c-d)/(c+d)
+(d-a)/(d+a)
=((a-b)(b+c)(c+d)(d+a)+(a+b)(b-c)(c+d)(d+a)+(a+b)(b+c)(c-d)(d+a)+(a+b)(b+c)(c+d)(d-a))/((a+b)(b+c)(c+d)(d+a))
=2(abca+abdd+accd-acda+bbcd-bbda-bcca-bcdd)/((a+b)(b+c)(c+d)(d+a))
=2(a-c)(b-d)(ac-bd)/((a+b)(b+c)(c+d)(d+a))
a≦c≦b≦dのとき負であるから偽
0537132人目の素数さん
垢版 |
2018/06/02(土) 16:58:09.50ID:ItLI/UY3
>>536
訂正
最後の行は「a<c<b<dのとき負であるから偽」
0538526
垢版 |
2018/06/03(日) 22:40:41.09ID:8Esgc1bN
>>528 (1),(2)正解

(2)は(c+1/c)^2=(c^2)+1/(c^2)+2が思い出せるかが鍵であった。

(1)も初項を(1/2)(c+1/c)(これはc=e^xとすればcosh(x)の定義)とおけば解ける。

>>529
b_1∈[-2,2]の場合は確かに2cosθとおくと解ける


初項を三角関数でおくと解けるような漸化式のパターンは整理してまとめる必要があるかもしれない
0539132人目の素数さん
垢版 |
2018/06/04(月) 09:18:52.07ID:tzM+Pvvj
>>533 >>534
 かたじけのうござる。

>>535 [100]
  IMO-2008 Short List A.7
 a-c=x,b-d=y の2次式と考えて、半正定値となることを示す。
 確かに面倒くせぇ…
0540イナ ◆/7jUdUKiSM
垢版 |
2018/06/04(月) 14:37:57.78ID:8ezyuuAm
>>420
ほととぎすの鳴き声を聴きながら、黄金二等辺四面体の中にある、正五角形4つと三等辺台形4つからなる八面体を切りだすことを考えています。前>>531
0541イナ ◆/7jUdUKiSM
垢版 |
2018/06/04(月) 14:54:40.80ID:8ezyuuAm
>>540
y={(√5-1)/2}x≒0.618x<x
これはあってるはず。

八面体を囲む黄金二等辺四面体の高さh'も、二等辺三角形{二辺(1+√5)x、底辺2x}の高さhと同様、xで表せるはず。

あとはそこから切り落とす部分。
0542イナ ◆/7jUdUKiSM
垢版 |
2018/06/04(月) 14:55:01.70ID:8ezyuuAm
>>540
y={(√5-1)/2}x≒0.618x<x
これはあってるはず。

八面体を囲む黄金二等辺四面体の高さh'も、二等辺三角形{二辺(1+√5)x、底辺2x}の高さhと同様、xで表せるはず。

あとはそこから切り落とす部分。
0543132人目の素数さん
垢版 |
2018/06/04(月) 15:38:18.38ID:tzM+Pvvj
>>536

(1) は (a+b+c) を掛けた方が分かりやすい…

(2) の類題
[166] a〜d>0 のとき
  (a-b)/(b+c) + (b-c)/(c+d) + (c-d)/(d+a) + (d-a)/(a+b) ≧ 0,
  クロアチア MEMOTST-2009 day1-Q.1
0544イナ ◆/7jUdUKiSM
垢版 |
2018/06/05(火) 05:22:33.74ID:+cO1MVWm
>>420近似値を求めよとは言ってない。
>>438にあるように、
V=√(√3)/18
のような確定的な値を示すべきだ。
>>542
一辺xの正五角形4つと三等辺台形[三辺がxで、あと一辺は{(√5-1)/2}x]4つからなる八面体の体積を0.074という近似値ではなく、
V=√(√3)/18
のような確定的な値で示せないか。これが示せて初めて正解だろ。
もっともxやx^2の値は、表面積1の条件から近似値が求まる。が、求める値はxやx^2の値ではないし、そもそも近似値は近い似た値であって正解じゃない。
0547132人目の素数さん
垢版 |
2018/06/05(火) 10:22:27.38ID:Ysbk5G+R
>>544
方程式が代数的に解けない可能性があることを忘れずに。
先の>>464を偏微分で解こうとして式をたててみたところ、rについて5次の項が出てきたので、その可能性はあると考えている。
なので、まず解析的に解くという方針は正しいかもしれない。
0548132人目の素数さん
垢版 |
2018/06/06(水) 07:05:16.27ID:xxwxn7ab
>>543 (2)

(a-b)/(b+c) = (a+c)/(b+c) -1
(c-d)/(d+a) = (a+c)/(d+a) -1
辺々たすと
(a-b)/(b+c) + (c-d)/(d+a) = (a+c){1/(b+c) + 1/(d+a)} -2
 ≧ 4(a+c)/(a+b+c+d) -2,   (← AM-HM)
同様に
(b-c)/(c+d) + (d-a)/(a+b) ≧ 4(b+d)/(a+b+c+d) -2,
辺々たす。
0549イナ ◆/7jUdUKiSM
垢版 |
2018/06/06(水) 16:11:11.85ID:NQEcakbH
>>420八面体の体積をxで表せれば解ける可能性がある。
六角柱は微分した。
ホームベース型五角形4つと長方形4つは微分しなかったが六角柱と同値。
>>544正五角形4つと台形4つが最大かどうかはともかく、これらを超えることは明らか。体積をxで表せれば。
台形同士がとなりあう辺は2つあり(仮に棟木と底辺と名づける)、長さは、
(√5-1)/2
たがいにもっとも離れていて、90°ねじれている。
正五角形を含む二等辺三角形を延長してできる、4つの二等辺三角形で囲まれた四面体から、棟木と底辺の2つの水平面で切り分けた、断面2x・(√5-1)x/2の長方形を底面とする五面体(八面体の外にある)の体積は、
(x^3)√(25+10√5)/4
この因数は、かなりあってると思う。
0550イナ ◆/7jUdUKiSM
垢版 |
2018/06/06(水) 16:15:29.57ID:NQEcakbH
>>549修正。
八面体の体積をxで表せれば解ける可能性がある。
六角柱は微分した。
ホームベース型五角形4つと長方形4つは微分しなかったが六角柱と同値。
正五角形4つと台形4つが最大かどうかはともかく、これらを超えることは明らか。体積をxで表せれば。
台形同士がとなりあう辺は2つあり(仮に棟木と底辺と名づける)、長さは、
(√5-1)x/2
たがいにもっとも離れていて、90°ねじれている。
正五角形を含む二等辺三角形を延長してできる、4つの二等辺三角形で囲まれた四面体から、棟木と底辺の2つの水平面で切り分けた、断面2x・(√5-1)x/2の長方形を底面とする五面体(八面体の外にある)の体積は、
(x^3)√(25+10√5)/4
この因数は、かなりあってると思う。
0551132人目の素数さん
垢版 |
2018/06/06(水) 18:03:07.23ID:n39uR33J
ちょっと方向性を変えて、現時点で8面体の場合は難しいとして、何面体までなら現時点で求まるんだろう?5面体ぐらいまでならなんとか厳密解は求まる希ガス。6面体位が解決出来るか否かの瀬戸際?
0554イナ ◆/7jUdUKiSM
垢版 |
2018/06/06(水) 22:13:34.88ID:NQEcakbH
>>552ある。
2x、(√5-1)x/2の長方形を底面とする五面体。
x、2x、x、(3+√5)x/2の台形で前後、
x、x、(√5-1)x/2の二等辺三角形で左右を挟む。
>>550名づけて、ニベアの底。
{(x^3)/4}√(25+10√5)
0555132人目の素数さん
垢版 |
2018/06/06(水) 22:48:25.85ID:EP2keEH6
>>554
それ結局三角柱と同形だろ
0557イナ ◆/7jUdUKiSM
垢版 |
2018/06/06(水) 23:20:48.78ID:NQEcakbH
>>554
>>555>>556違う。三角柱じゃない。左右の二等辺三角形の面が底辺の長方形に対して外に開いてる。五面体だ。
2xより(3+√5)x/2のほうが少し大きい。ラミネートチューブのニベアだぞ。八面体を四面体の中に封じこめた。水平に斬った。断面は正方形だ。
一辺(3+√5)/4
斬った2体を横に並べて同じかたちに等積変形していく。直方体にすれば絶対求まる。二方向は足し算や平均で出る。一方向だけは三平方の定理が必要。ニベアの底でもやったし、二種類の台形と二等辺三角形2つと長方形で囲まれた五面体でもすでにやった。
全体が正しく求まればあとは引き算で出る。八面体をxの三次式で出すぞ。微分して二次だして=0で字数を下げる。シンプルな式になればいいんだけど。
0560132人目の素数さん
垢版 |
2018/06/07(木) 01:22:47.10ID:Y05WzRQF
4面体は正四面体だろうけど、5面体はどうだろう?
歪めた3角柱だろうなとは思う。正三角柱くさいが。
4面体で最大も結構示しにくい。
0561イナ ◆/7jUdUKiSM
垢版 |
2018/06/07(木) 04:17:01.76ID:fHjWRa4l
>>557

>>560正五角形と三等辺台形でできる八面体がすっぽり入る黄金比二等辺三角形四面体の体積を求めました。

最大値をxで表して微分して=0で次数下げてx^2からx出して最大値求めて電卓使って近似したら11.686935

0.074よりはるかに大きくなりました。
0562132人目の素数さん
垢版 |
2018/06/07(木) 10:08:48.01ID:n/uc+6Kl
表面積1の球の体積が約0.094
これより大きくはなりようがないと思うが如何か
0563132人目の素数さん
垢版 |
2018/06/07(木) 10:43:47.55ID:BLDmLkdD
計算間違いと逆数にしたのと。
0564132人目の素数さん
垢版 |
2018/06/07(木) 11:23:31.73ID:55KNO4WC
>>551
n=4,5,6,7,12以外は未解決らしい

「離散幾何学における未解決問題集」シュプリンガー・ジャパン(2009) p.394
0565イナ ◆/7jUdUKiSM
垢版 |
2018/06/07(木) 14:45:01.84ID:fHjWRa4l
>>561
四角形どうしがとなりあう辺がやや短い{(√5-1)/2}xと気づいたが、もしかすると五角形どうしがとなりあう辺と、五角形と四角形がとなりあう辺とでは、その条件の違いから長さを変えてくるんじゃないか、自然界は。

あと、計算間違いの可能性はある。
√(√3)/18を超えたい。
中に水溜めてメスシリンダーに移しかえて実測値を量りたいわけじゃない。背面跳びでもベリーロールでもいい、ただ超えたいだけ。
0566132人目の素数さん
垢版 |
2018/06/07(木) 14:47:26.69ID:1rV5JhAO
初めてなのでルール違反があれば教えてください
g.c.d(a_1,a_2,…,a_n)はa_1,a_2,…,a_nの最大公約数である
a_1,a_2,…,a_nが互いに素であるとは、g.c.d(a_1,a_2,…,a_n)=1である事を意味する
a_1,a_2,…,a_nがk個ごとに素であるとは、a_1,a_2,…,a_nの中からどの様にk個取ってきても互いに素である事を意味する
σ_k(a_1,a_2,…,a_n)はk次の基本対称式とする
ただし、σ_0(a_1,a_2,…,a_n)=1
この時
「a_1,a_2,…,a_nがk個ごとに素である」…@

「σ_(n-k+1),σ_(n-k+2),…,σ_nが互いに素である」…A
が同値である事を示せ
0567132人目の素数さん
垢版 |
2018/06/07(木) 15:21:37.68ID:3xfkAX0Q
>>566

f(x)=(x-a1)…(x-an)とおく。

@⇄∀p f(x)のxの多重度がmod pでk未満
⇄∀p f(x)のk以下の次数の係数のいずれかが0でない
⇄A
0568132人目の素数さん
垢版 |
2018/06/07(木) 17:07:02.73ID:1rV5JhAO
>>567
想定していなかった解答ですが、自分が考えた物より遥かにエレガントです
もう自分の解答なんか出せない…
ので代わりに、用意していたもう片方の問題をば

Λを原点を除く2次元列ベクトル空間の格子点の集合とする
列ベクトル(a,b)∈Λに対し、aとbが互いに素である事を(a,b)〜1と表記する事にする
全てのv∈Λに対してAv∈Λなる行列Aを格子行列と呼ぶ
この時、格子行列Aが「任意のv∈Λに対して、v〜1ならばAv〜1である」を満たす時、Aの行列式を求めよ

これもすぐ解かれたらもう少し勉強します
0569132人目の素数さん
垢版 |
2018/06/07(木) 22:42:17.40ID:HuW8Hkll
>>420
未解決問題じゃねえか
糞が
タヒね
お前の頭劣等感婆か?
0570イナ ◆/7jUdUKiSM
垢版 |
2018/06/07(木) 23:18:10.31ID:fHjWRa4l
>>569未解決なの?
0.074超えの数値が出ただけか。
>>565
>>438てことは一辺x高さyの六角柱を微分して出した√(√3)/18か、三つのパラメーターで空間座標設定した人のが今のとこ人力で導かれる最大値か。


五角形は正五角形なのかとなりあわない二辺が少し短いのか、四角形は三等辺台形なのかただの等脚台形なのか、まだなぞがいっぱいです。
0571132人目の素数さん
垢版 |
2018/06/07(木) 23:51:56.94ID:zA2lx+rZ
>>564
>n=4,5,6,7,12
の場合の答えはなにか載ってました?
n=4,6,12のときは流石に正多面体の時っぽいけど。
0574132人目の素数さん
垢版 |
2018/06/08(金) 00:53:07.64ID:DFewMgI5
>>568
泥臭いけど。
整係数の行列A,Bについて
B=[[1,k],[0,1]]A, A[[1,k],[0,1]], [[1,0],[k,1]]A, A[[1,0],[k,1]]→A≡B
をみたす最小の同値関係を≡とする。
一般にPID上の行列はこの変形で対角化される。(証明はry)
行列は行ベクトルに右から作用させるものとする。
(旧課程の高校の教科書の逆だけど気にしない。)
(※):任意のv∈Λに対して、v〜1ならばAv〜1である
と定める。
以下整係数行列Aに対しBをA≡BでBは対角行列となるものとする。
このとき
ユークリッドの互除法の議論より
Aが(※を満たす)⇔Bが(※を満たす)。
また
detA = detBより
detA=±1⇔detB=±1
一方でB=[[p,0],[0,q]]とするとき
Bが(※を満たす)⇔p,q=±1
detB = ±1⇔ p,q=±1
0576132人目の素数さん
垢版 |
2018/06/08(金) 01:05:42.81ID:OwX5577s
http://www.geocities.jp/ikuro_kotaro/koramu/684_eq3.htm


>多面体の等周問題は,単位球に外接する多面体では,
> V=S/3
>となることから,
> S^3/V^2=9S=27V
>したがって,与えられた面数nをもつ多面体に関する等周問題は,最小の体積または
>最小の表面積をもち,球に外接するn面体を定めるという問題に帰着されます

がわからない??これなんでかだれかわかります??
もしV^2/S^3で球に外接しないものがあるとすると何故矛盾するんでしょう?
0577イナ ◆/7jUdUKiSM
垢版 |
2018/06/08(金) 03:10:33.54ID:n7O1sKDD
>>573
ゴールドバーグさんは示されたんですね。前>>570その歴史の存在はわかりました。

でも我々は、式とそこから導かれた答えでないと認めないルールでやってきてます。それにできればそのビジュアル八面体とやらを図示していただきたい。
手書きでもいいですが、できれば実物の模型がいいですね。辺の長さとかバランスとかどうなってんですかね。
四角形と五角形がどんな四角形とどんな五角形か、納得いくようにその立体のかたちを説明してください。実在するんですか。
0579132人目の素数さん
垢版 |
2018/06/08(金) 08:08:27.84ID:ITi87fOm
>>578
しかもフランス語…

例のGoldbergの論文の最初のあたりをちゃんと読むと、
同じ面の数の多面体でS^3/V^2を最小にするのは
すべての面が1つの球に各面の重心で接するときだ、ということを
Lindelofが示した、とたしかに書いてますね。
直感的に考えると、そういう状態でないと極小値にならない
(少し動かしたらもっと小さくできる)
というような話なのかとは思います。
そうすると、>>486のメディアル8面体で、そのような状態になるような
パラメータa,b,rを求める、というのはできそうですかね。
それが>>471>>489あたりの数値計算の結果と一致するか
もしくは、Goldbergの論文の180.23という値と一致するか、も
知りたいところです。
0580132人目の素数さん
垢版 |
2018/06/08(金) 11:08:18.22ID:uyKvXzxT
次の定理を下に示す4つの場合について証明せよ。

多面体の面の全てに、その面積を大きさにもちそれぞれの面から垂直に多面体を飛び出す向きにベクトルをとると、そのベクトルの和は零となる。

(1)四面体
(2)角錐
(3)凸多面体
(4)多面体
0581132人目の素数さん
垢版 |
2018/06/08(金) 11:23:46.15ID:E/oBAxNr
すべての多面体は、有限個の四面体に分割できる
と仮定していいの?
0582132人目の素数さん
垢版 |
2018/06/08(金) 12:07:26.06ID:r5bxmv7X
・正8面体 … 明らかに球面に外接する。

・正6角柱
  一辺xの正六角形を底面とする高さyの正六角柱
  y = (√3)x のとき最大で、球面に外接する。>>425 >>430

・メディアル8面体
 >>464 の座標を使うと、

5角面ABCDE は x +(a/b)z = 1,
原点〜5角面の距離は d_5 = b/√(aa+bb),

台形面CDHI は z = br + my,m = b(r-1)/(1+a),
原点〜台形の距離は d_4 = br/√(1+mm),

>>489 の結果の数値から

 d_5 = 0.96477885
 m  = 0.74846993
 d_4 = 0.96477948

∴ 球面に外接する。(誤差の範囲内で)
0583132人目の素数さん
垢版 |
2018/06/08(金) 12:47:23.35ID:r5bxmv7X
>>580

i番目の面に対するベクトルをS_iとおく。

任意の向きの単位ベクトルeをとる。
この多面体をeに垂直な面に射影する。

向こう向きの面(S_j) の影の面積は e・S_i
手前向きの面(S_j) の影の面積は e・(-S_j) である。
・は内積。
それぞれ合計したものは多面体の輪郭と一致するから
 e・(ΣS_i) = e・(-ΣS_j),
 e・ΣS = e・(ΣS_i + ΣS_j) = 0,
eの向きは任意だったから
 ΣS = O.
0584132人目の素数さん
垢版 |
2018/06/08(金) 12:57:46.26ID:E/oBAxNr
七面体の解はどんなだろう?
正五角柱か、あるいは立方体の角を落としたものか
0585132人目の素数さん
垢版 |
2018/06/08(金) 13:30:54.48ID:r5bxmv7X
>>582 追加

・切頂立方体
立方体の一辺をx、切り取る二つの直角三角錐の3稜をyとする。
 y = {(3-√3)/2}x のとき最大で、   >>483 >>485 >>488
 原点と(x/2-y/3,x/2-y/3,x/2-y/3) の距離は (x/2 -y/3)√3 = x/2,
∴ 球面に外接する。
0586132人目の素数さん
垢版 |
2018/06/08(金) 14:53:42.28ID:ITi87fOm
>>582
Goldbergの論文によると、球に外接する正五角柱です。

どんな場合もメディアル多面体の中に正解があると予想しているのだから、
少なくとも今まで正解が判明しているものはすべて答えはメディアル多面体。

7面体の場合は、四角形5つと五角形2つの五角柱と同じ面/辺/頂点の構成と
なってるのがメディアル多面体で、その中で、辺と頂点からなるグラフの持つ
対称性がそのまま図形としての対称性となってる正五角柱が正解なのは
自然な結論。
0587132人目の素数さん
垢版 |
2018/06/08(金) 15:41:39.47ID:9F3pIZPF
>>578の定理めっちゃ面白いのにネットに証明転がっってないのは残念。
自分で示せそうにないし。
なんかMinkowskiの不等式なるものを使うっぽいけどそれ自体も見つからんし。
0588イナ ◆/7jUdUKiSM
垢版 |
2018/06/08(金) 19:01:20.32ID:n7O1sKDD
メディアル八面体がわかるなら式を書いてよ。辺をxとかyとか未知数で表した式を。未知数は二つぐらいのほうがいいと思う。メディアル八面体より小さくてもいいよ。
>>577ホームベース型五角形4つと長方形4つで
√(√3)/18になったのを進化させた次のやつがあるんじゃないの。屋根120°って勝手にきめてこの値が出たんだ。体積が大きくなりそうなかたちを勝手に決めようよ。未知数2つで三次式なら微分して表面積=1で一文字消えて解けるよ。だからなるべく簡単な構造がいい。
0589132人目の素数さん
垢版 |
2018/06/08(金) 19:12:17.79ID:Mfly9++H
>>571
最大を達成する形については特に載ってなかったな
やっぱりGoldberg氏の論文が紹介されてる

それとGoldbergの面白い予想(当然未解決)
「当たられた面数、表面積と最大体積をもつ全ての3次元多面体は単純である
(多面体が単純⇔各頂点がちょうど3本の辺に属しているもの)」
ってのが載ってた
0590132人目の素数さん
垢版 |
2018/06/08(金) 19:47:16.52ID:Mfly9++H
>>588
何変数あろうとラグランジュの未定常数法みたいなの使えばいいだけでしょ
0592イナ ◆/7jUdUKiSM
垢版 |
2018/06/08(金) 20:23:46.96ID:n7O1sKDD
>>591
xの三次式とかそういうかたちでお願いします。
S=1なんで式の中にSがあれば計算して消えます。
>>588
0594イナ ◆/7jUdUKiSM
垢版 |
2018/06/08(金) 20:33:37.98ID:n7O1sKDD
>>592
超えられない斜めの壁がある。計算しやすい簡単な数字を探す。
六角柱=長方形屋根120°壁ホームベース型上下点対称八面体=√(√3)/18
<√(√π)/18
<0.0074
<メディアル八面体
0595132人目の素数さん
垢版 |
2018/06/08(金) 21:22:34.50ID:ITi87fOm
>>587
ミンコフスキーの不等式なら、Wikipediaにも載ってるし、
解説もあちこち落ちてる気がする。
0596132人目の素数さん
垢版 |
2018/06/08(金) 21:26:44.53ID:ITi87fOm
(あげてしまったorz)
なんか自由度3の状況をパラメトリックに表現したものに対して
変数を2個に減らせと言ってるのを目撃した気がするが、きっと気のせい。
0597132人目の素数さん
垢版 |
2018/06/08(金) 21:59:37.50ID:pJljV8zI
式本体は見かけるんだけど証明が見つかんない。
とても自力では解ける気かしない。
しかもそれをどう使えば>>578が出るのかもさ〜っぱり。
0598イナ ◆/7jUdUKiSM
垢版 |
2018/06/08(金) 22:16:18.57ID:n7O1sKDD
>>594
いつか体積、
√(√3)/18=1/6√(3√3)
を超える八面体の鳥瞰図を描きたい。
0.074は超えられなくてもいい。ちゃんと辺の長さをつきとめたい。
五角形の長い辺は、四角形の短い辺の二倍ぐらいなのかなぁ。
0599132人目の素数さん
垢版 |
2018/06/08(金) 23:50:26.48ID:r5bxmv7X
・一辺がxの正5角形を切り詰めたもの (内角は108゚のまま) >>510 >>511

中心Oを通る水平断面で考えると、OからABDEの中央までの距離は
 (EG + DH)/4 = {φx + (x+z)}/4 = {(φ+1)x + z}/4,
5角面の傾きθ = arctan{1/√(2φ^3)} = 18.9607099゚
∴ cosθ = 0.9457416
∴ d_5 = {(φ+1)x+z}/4・cosθ = 0.6189959x + 0.2364354z,

C と B,D の高低差は x/√(2φ) = 0.5558930x,
BD = φx,
∴ CI と DH の距離は √{(φ/2)^2 + 1/(2φ)}・x = (1/2)√(3φ-1)・x = 0.9815933x,
Cと中心Oの高低差は x/√(2φ) + √(φ/8) (x-φz),
これに φ/√(3φ-1) = 0.8241875 を掛けて
d_4 = {x/√(2φ) + √(φ/8)(x-φz)}・φ/√(3φ-1) = 0.8288193x - 0.5997393z,

z=0 のときは
 d_5 = 0.6189959x
 d_4 = 0.8288193x
となり、球面に外接しない。 >>510

最大となるのは z = 0.2509325x のときで
 d_4 = d_5 = 0.67832525x
∴ 球面に外接する。   >>511
0603132人目の素数さん
垢版 |
2018/06/09(土) 01:29:58.35ID:bO8NYEjH
>>602 のサイトの下のほうにP_8(と彼は呼んでいるn=8の場合の表面積極小メディアル8面体)の面のjpeg画像がある。
4種類出てくるらしい。
数式だせよな……
http://schoengeometry.com/a2-poly-media/the-best-8(1).jpg
http://schoengeometry.com/a2-poly-media/the-best-8(2).jpg
http://schoengeometry.com/a2-poly-media/the-best-8(3).jpg
http://schoengeometry.com/a2-poly-media/the-best-8(4).jpg
0604132人目の素数さん
垢版 |
2018/06/09(土) 09:56:15.29ID:U/DEwkMV
>>603
いや、4種類あるわけじゃなさそうだけど。
図形は2種類で、その4つのテンプレートのミソは、頂点に番号が振ってあるとこで
同じ番号の頂点同士が一致するように組み立てれば8面体の半分ができるから
それを2つ組み合わせて立体をイメージしてね、ってことでしょ?
面の形は2種類。
0605132人目の素数さん
垢版 |
2018/06/09(土) 10:43:16.31ID:hShDyG0j
>>604
そうなんすか?8面なのに画像が4枚しかないから合同なのは除いてると思った。一度誰か厳密な頂点の座標出してくれません?
0606イナ ◆/7jUdUKiSM
垢版 |
2018/06/09(土) 10:51:23.69ID:ASELe/sj
もういいよ、頂点は。
それより辺の長さとバランスだよ。前>>598
意外と台形太いね。
気づいていたさ、ずっと眺めてんだから。寝ても覚めても。

五角形の屋根と底は違うのかもね。文字数多すぎるだよ。
0607イナ ◆/7jUdUKiSM
垢版 |
2018/06/09(土) 12:25:33.04ID:ASELe/sj
目標0.074488
二種類の平面図のおかげで鳥瞰図が描けた。
棟木と垂木は9x
若干垂木が長く見えるが、誤差と見た。
軒桁を12xにして垂木は若干放射状になる。
柱は5x少し斜めに立て、立体の重心だか中心に対して点対称の立体を地下に作る。
>>598できそうだ。
0608イナ ◆/7jUdUKiSM
垢版 |
2018/06/09(土) 12:32:27.47ID:ASELe/sj
棟木と垂木は9x
軒桁を12xにして垂木は若干放射状になる。
柱は5x少し斜めに立て、立体の中心に対して点対称の立体を地下に水平90°回転で作る。
>>607前々>>606
0609132人目の素数さん
垢版 |
2018/06/09(土) 14:18:02.73ID:1x4Xd21b
立体視にするとこんな感じかな

http://imgur.com/xEVxtDg.png

A( 0.255096, 0.207265, 0.0876588)
B( 0.207265, 0.255096,-0.0876588)
C( 0.155171, 0.000000,-0.278602)
D( 0.207265,-0.255096,-0.0876588)
E( 0.255096,-0.207265, 0.0876588)
F( 0.000000,-0.155171, 0.278602)
G(-0.255096,-0.207265, 0.0876588)
H(-0.207265,-0.255096,-0.0876588)
I(-0.155171, 0.000000,-0.278602)
J(-0.207265, 0.255096,-0.0876588)
K(-0.255096, 0.207265, 0.0876588)
L( 0.000000, 0.155171, 0.278602)
0611イナ ◆/7jUdUKiSM
垢版 |
2018/06/09(土) 17:06:23.64ID:ASELe/sj
>>608
実測値から式を推定する。

なんどか言ったが、少数にするのは近似値を出すためじゃない。√や比を推測して体積を表す式を導きたいからだ。
0612イナ ◆/7jUdUKiSM
垢版 |
2018/06/09(土) 18:32:27.65ID:ASELe/sj
今回は期待できる。前>>511台形の高さが9xとか五角形の下半分の高さが5xとか、平方根が出ない。壁の傾きを考えると出ないわけないが、ゴールドバーグさんが言った簡単な構造になるだったか、あの言葉を信じたい。
0613132人目の素数さん
垢版 |
2018/06/09(土) 22:22:54.65ID:U/DEwkMV
なんか無理数の存在を認めなかったピタゴラス学派の時代からほとんど進歩してない奴がいるな
0616132人目の素数さん
垢版 |
2018/06/10(日) 05:26:41.33ID:73iwKoh1
いや、やっぱり解けたかな。
でもなんか面白い。
立て方がヘタクソなのか、最初立式した時は未知数4つが入り乱れててこんなん解けるかボケって思えたけど、いざ整理していくと不思議とまとまっていく。
やっぱり受験問題みたいにムリクリ作った問題とは一足違う。
0618132人目の素数さん
垢版 |
2018/06/10(日) 15:29:05.27ID:Cphvbc4E
>>617
だいたいそんな感じです
データはWolframAlpha先生が教えてくれたものを使いました
http://imgur.com/KnEKaE1.gif
(変数 p,q,r は >>464 の a,b,r-1 に対応しています)
0619132人目の素数さん
垢版 |
2018/06/10(日) 15:32:58.71ID:KetZUwRK
>>510
 >>464
 a = 2φ-3 = 0.236068
 b = √(2√5 -4) = 0.6871215
 r = 2φ-1 = √5 = 2.236068
とした場合。
 x = 4(2-φ) = 1.527864

>>511
 >>464 で、(AE+BD)/2 = {(x+z)+φx}/2 = 2 として
 a = (BD-AE)/4 = {φx - (x+z)}/4,
 b = AB/2・√(φ/2) = (x-φz)/2・√(φ/2),
 r = 1 + x/(2φφa)
とした場合。

最大のときは
 a = 0.127956
 b = 0.3724407
 r = 3.0809832
とした場合。
 x = 1.3942303
 z = 0.3498577 = 0.2509325x

>511 は >464 に含まれるゆえ、>514 は >471 >489 には及ばぬでござる。
0621132人目の素数さん
垢版 |
2018/06/10(日) 17:44:15.34ID:Cphvbc4E
データは >>489 のほうがWolfram先生より良い(体積が大きい)みたいです。
あと、>>609 の座標は計算に誤りがあったので計算しなおしました。

A( 0.25508091, 0.20727281, 0.087668243)
B( 0.20727281, 0.25508091,-0.087668243)
C( 0.15521551, 0.00000000,-0.27858864)
D( 0.20727281,-0.25508091,-0.087668243)
E( 0.25508091,-0.20727281, 0.087668243)
F( 0.00000000,-0.15521551, 0.27858864)
G(-0.25508091,-0.20727281, 0.087668243)
H(-0.20727281,-0.25508091,-0.087668243)
I(-0.15521551, 0.00000000,-0.27858864)
J(-0.20727281, 0.25508091,-0.087668243)
K(-0.25508091, 0.20727281, 0.087668243)
L( 0.00000000, 0.15521551, 0.27858864)
0622イナ ◆/7jUdUKiSM
垢版 |
2018/06/10(日) 18:15:14.87ID:VRYWqgPw
もっとずっと簡単な整数比をみつける。前>>612ゴールドバーグさんも言ってた。自然界は簡単な構造だというようなことを。

計算が楽になりそうな数字がみつかるまで、もう計算しない。

自然界はもっとずっと簡単な構造で、楽に計算できる数学を選んでくるはずだから。簡単な整数比じゃないと計算したくない。
0623132人目の素数さん
垢版 |
2018/06/10(日) 18:34:29.44ID:y9Cpd902
まだ無駄なことやってんのか
0624イナ ◆/7jUdUKiSM
垢版 |
2018/06/10(日) 18:53:24.67ID:VRYWqgPw
>>622

整数比じゃなくてもいい。黄金比とか自然界にはなるべくしてなる比が存在している。
整数比だったり√3だったり√5だったり。

無駄なことが報われるときってのは、無駄を回避したことが無駄じゃなかったとわかったときだ。
0626イナ ◆/7jUdUKiSM
垢版 |
2018/06/10(日) 23:51:51.67ID:VRYWqgPw
>>626すげー!! まわってる、まわってる!!
>>625すごいね。写メ保存したら動画のまま保存できてびっくりした。棟木をズームして実測、3.6xにしたら、V(x)=86.192(x^3)、x^2=1/131.68あんまり大きくならなかった。計算間違いかな。見た目は球体に近づいてるのに。
 ̄]/\_______
_/\/ ∩∩ ∩∩ /|
 ̄\/ ((^o^)((ー_ー)/ |
 ̄|\_,U⌒U、(っu~) |_
]| ‖~UU~  ̄`υυ / /
_| ‖ □ □ ‖ |/ /
_ `‖____‖/_/
 ̄ ̄ ̄ ̄ ̄ ̄ ̄ ̄ ̄‖ 
□  □  □  ‖ /
_________‖/
 ̄ ̄ ̄ ̄ ̄ ̄ ̄ ̄ ̄ ̄
0628132人目の素数さん
垢版 |
2018/06/11(月) 03:01:19.91ID:zkIOdW8D
これだけ長い議論になると八面体の別スレを設けたほうがいいレベルだな
0629132人目の素数さん
垢版 |
2018/06/11(月) 09:13:19.80ID:E/XZVJ+8
八面体だけだと狭すぎる
多面体一般にしたら需要あるかも
0631132人目の素数さん
垢版 |
2018/06/11(月) 10:23:50.28ID:TnGShdQw
>>466 >>472
菱形6面体(菱面体)のとき
 辺の長さL = √{(2aa+cc)/3},
 体積 V(0) = aac,
 表面積 S(0) = 6LL・sinβ = 2a√{3(aa+2cc)},
 中心Oから各面までの距離 d = (√3)/2・ac/√(aa+2cc),

>>467
頂点から k・L まで(Lは辺長、0≦k≦3/2) の部分を切り落とす。
 体積 V(k) = V(0){1 - (1/3)k^3}    (0≦k≦1)
      = V(0)(9/4){1 - (1-2k/3)^2}(1-2k/3) (1≦k≦3/2)
 表面積 S(k) = S(0) - (√3){√(aa+2cc) -a}ak^2, (0≦k≦1)
 k = 3/2 -(√3)d/c のとき、平面z=±d で切るので球面に外接する。

 (1) c=a (立方体、β=90゚)のとき、k=(3-√3)/2,d=c/2 で外接。 >>488
 (2) c=2a(β=60゚) のとき、k=1,d=c/√12 で外接。(正8面体)
 (3) c>2a のときは k>1 となり、やや面倒でござる。
0632132人目の素数さん
垢版 |
2018/06/11(月) 11:36:32.77ID:f053/Yvw
>>629
何で3次元に限るのって話
0633132人目の素数さん
垢版 |
2018/06/11(月) 16:31:57.45ID:alvL18N0
皆さんにお詫びと訂正を。昨日解けたといってた>>616ですがやっぱり解けてませんでしたorz。
参考までに私の失敗した方法です。
>>621さんと同じ配置で内接球の半径を1に規格化した点をA〜Lとします。
A(a,c,b)、L(0,d,e)とします。
OからALFEにおろした垂線の足をX(cosα,0,sinα)、Y(cosβ,0,sinβ) (0<β<α<π/2)とおきます。
xz平面への射影の図をみれば
a=cos((α-β)/2) / cos((α+β)/2)、
b=sin((α-β)/2) / cos((α+β)/2)、
e=1/sinα、
c cosβ+ b sinβ=1、
d cosβ+ e sinβ=1
がわかります。
ここからLindeloefの条件
・ALFEの重心=X、ABCDEの重心=Y ……(※)
から条件が2つでてα、βの満たす方程式をだしていこうとしました。
そこで恥ずかしながら
ALFEの重心=ALFEの座標の和/4
とかわけのわからんミスをして失敗しました。
原理的には(※)からtanα/2, tanβ/2の代数方程式がでて、その終結式からtanα/2、tanβ/2の満たす方程式がでるはずですが五角形の重心の計算が面倒くさすぎて断念。
もちろん∂面積/∂α=∂面積/∂β=0を再利用して方程式2つ出す手もありますし、一般論ではでない本問特有の必要条件をみつけて利用する手もあるかとおもいます。
ご参考までに。
0635イナ ◆/7jUdUKiSM
垢版 |
2018/06/11(月) 20:41:25.63ID:FNFK9r9K
>>626前々>>624
>>625まわってるこの立体が最大として計算した。
五角形の水平な対角線より上の屋根の部分の体積は、三角柱から三角錐をとりのぞくと出るはず。
五角形の水平な対角線で切り分けた真ん中の部分は、直方体から三角柱と三角錐をとりのぞくと出るはずだが、コーナーを引きすぎたのかも。
台形2つの面積は(上底+下底)・(高さ)で出るはず。
この台形、(上底)=(高さ)だろう。なぜこうなるかは化学で解明されるべきことだと思う。原子がこういう配列をとるとか。
五角形の面積は、対角線より上が直角三角形の二倍、対角線より下が等脚台形。
三平方の定理がうまく使えてないか、あるいは最初の辺の長さの読みとりが甘いか。
0636132人目の素数さん
垢版 |
2018/06/12(火) 00:46:46.53ID:YFJLrlqV
>>464 をチョト拡張…

A(1+a1,1-a2,-b),B(1-a2,1+a1,b),C(c-ar,0,br),D(1-a2,-1-a1,b),E(1+a1,-1+a2,-b),
F(0,-c+ar,-br),G(-1-a1,-1+a2,-b),H(-1+a2,-1-a1,b),I(-c+ar,0,br),
J(-1+a2,1+a1,b),K(-1-a1,1-a2,-b),L(0,c-ar,-br)

ここに、a = (a1+a2)/2,c = (2+a1-a2)/2 とおいた。
AE = DH = 2(1-a2),
BD = 2(1+a1),
CI = FL = 2(c-ar),
CI〜DH,CI〜JB の距離 d '= √{(1+a1)^2 +bb(r-1)^2}

線分BA,ED,HG,KJの中点が 1辺2cの正方形をなす。

5角形ABCDE
 x + (a/b)z = c,
 d_5 = bc/√(aa+bb),

台形CDHI
 z = br + {b(r-1)/(1+a1)}y,
 d_4 = (1+a1)br/d '
 S_4 = (1-a2+c-ar)d '
0637132人目の素数さん
垢版 |
2018/06/12(火) 02:41:39.82ID:BNGFcTmJ
>>636
着想はいいと思う
ただこれだとc+a=1+a1、c-a=1-a2となるから本質的には>>464と変わりがない気がするんだ…
0638132人目の素数さん
垢版 |
2018/06/12(火) 11:52:09.79ID:YFJLrlqV
>>636

5角形ABCDE
 傾きθ = arctan(a/b),
 d_5 = bc/√(aa+bb),
 S_5 = {4c +(r-1)(1+a1)}√(aa+bb),

台形CDHI
 d ' = √{(1+a1)^2 + bb(r-1)^2},  CI〜DHの距離
 d_4 = (1+a1)br/d '
 S_4 = {2c-(r+1)a}d '

これより
 S = 4(S_5 + S_4)
  = 4{4c +(r-1)(1+a1)}√(aa+bb) + 4{2c-(r+1)a}d '

 V = (4/3)S_5・d_5 + (4/3)S_4・d_4
  = (4/3)bc{4c+(r-1)(1+a1)} + (4/3)(1+a1)br{2c-(r+1)a}
  = 8([2cc+(1+a1)(1-a2)]/3)b + 4(1+a1)b(r-1){c-(2+r)a/3},

・a1=a2,c=1 の場合は >>471 に戻る。
0639イナ ◆/7jUdUKiSM
垢版 |
2018/06/12(火) 18:46:36.80ID:TK3A96C9
>>635
棟木を3.6xとする。
八面体を五角形の水平な対角線での三つに分ける。
V(x)=V(x)+V(x)+V(x)
=2V(x)+V(x)
S(x)=4(台形)+4(五角形)
=2(上底+下底)・(高さ)+8(直角三角形)+4(等脚台形)
=2(3.6x+4.8x)・3.6x+4(2.9x・2.3x)+2(5.8x+4.8x)・2.1x
=16.8x・3.6x+5.8x・4.6x+21.2x・2.1x
=7・2.4x・3.6x+5.8x・4.6x+7x・6.36x
=7x・6x・1.44+5.8x・4.6x+7x・6x・1.06
=(7・6・2.5+5.8・4.6)x^2
=(105+26.68)x^2
=131.68x^2
V(x)の高さ=√{(2.3x)^2-(0.6x)^2}
=x√(5.29-0.36)
=x√4.93
V(x)の高さ=x√{(2.1)^2-(0.5)^2}
=x√(4.41-0.25)
=x√4.16
V(x)=(三角柱)-(三角錐)=2.9x・4.8x・x√4.93-2.9x・x√4.93・(1/2)0.6x(1/3)4
=2.9x・x√4.93・(4.8-0.4)x
=12.76x^3√4.93
V(x)=(直方体)+4(直角三角柱)+4(直角等面四面体)
=(4.8x)^2・x√4.16+4・0.5x・4.8x(1/2)+4(0.5x)^2x√4.16(1/6)
={(4.8x)(4.8x+x)+(1/6)}x√4.16
=(28.00666……)√4.16
V(x)=2・12.76√4.93+(28.0066……)√4.16
=25.52√4.93+(28.0066……)√4.16
≒55.663594+57.122614
=112.7862
題意よりS=1
x^2=1/131.68
x=1/11.47519
V(x)=112.7862/131.68・11.47519
≒0.0746407
0640イナ ◆/7jUdUKiSM
垢版 |
2018/06/12(火) 19:17:17.82ID:TK3A96C9
>>639(x^3)が抜けた。
終盤修正。
V(x)=2・12.76x^3√4.93+(28.0066……)x^3√4.16
=25.52x^3√4.93+(28.0066……)x^3√4.16
≒55.663594x^3+57.122614x^3
=112.7862x^3
以下同じ。
0641132人目の素数さん
垢版 |
2018/06/12(火) 23:21:04.90ID:10uSb+lc
数学なのに数字がたくさんある……
0642132人目の素数さん
垢版 |
2018/06/12(火) 23:59:03.56ID:VSdptTNG
1. 球面上にランダムにn個の点を取る
2. それらの点におけるn枚の接面で囲まれた多面体を作る
3. 接点と重心が最も離れている面を見つけ、その接点を球面上で重心の方向にずらす
4. 誤差が一定値以下になるまで 1〜3 を繰り返す
という方法でn=4〜20で極大値を計算してみた結果。

4面体 0.051700269950116652 (正四面体)
5面体 0.059698329545752334 (正三角柱)
6面体 0.068041381743977170 (正六面体)
7面体 0.071398254996602697 (正五角柱)
8面体 0.074344868093230002 (四角形×4+五角形×4)
9面体 0.076898933926867766 (四角形×3+五角形×6)
10面体 0.078734752898039745 (四角形×2+五角形×8)
11面体 0.080055026399577983 (四角形×2+五角形×8+六角形×1)
12面体 0.081688371824182537 (正十二面体)
13面体 0.082432267303420806 (四角形×1+五角形×10+六角形×2)
14面体 0.083349245941114841 (五角形×12+六角形×2)
15面体 0.084068875807253640 (五角形×12+六角形×3)
16面体 0.084742718358283536 (五角形×12+六角形×4)
17面体 0.085264872589057683 (五角形×12+六角形×5)
18面体 0.085771192859653247 (五角形×12+六角形×6)
19面体 0.086199376384128973 (五角形×12+六角形×7)
20面体 0.086626966830007951 (五角形×12+六角形×8)
(もっと良い解があるかもしれない)
0643イナ ◆/7jUdUKiSM
垢版 |
2018/06/12(火) 23:59:54.21ID:TK3A96C9
>>641せやて問題文に数字が1一個しかないんですって。

>>420←これですよ。数字は図描くなり作って上げるなりして自分で設定せいいう問題なんですよ。

なんでこうなるかはまだわかりませんが、屋根の部分は棟木と最短の垂木が同じ長さで、棟木と軒桁の長さの比が3:4になってます。
>>640研究が要ります。
0646132人目の素数さん
垢版 |
2018/06/13(水) 01:02:41.54ID:5ZmF3Enb
>>643

CI = FL = 3.60000 x, … 棟木
とするなら
AB = DE = 2.17929 x,
BC = CD = 3.743965 x,
AE = DH = 4.807435 x, … 軒桁
BD = 5.91628 x,
S_5 = 18.08915 xx,

CI〜DH間 3.69496 x,  … 垂木?
S_4 = 15.53246 xx,

S = 134.4864 xx,
V = 115.9502 x^3,

V/S^(3/2) = 0.07434486809323… になるらしい。

>>621
> データは >>489 のほうが…良い(体積が大きい)みたいです。
おっしゃるとおり。

>>637
そうですねぇ...
0647132人目の素数さん
垢版 |
2018/06/13(水) 01:58:33.50ID:5ZmF3Enb
>>642
理論値

f=4 (正4面体) 1/{6√(6√3)},
f=5 (正3角柱) 1/{9√(2√3)},
f=6 (立方体)  1/(6√6),
f=7 (正5角柱) 1/{3√30・(5-2√5)^(1/4)},
f=12 (正12面体)1/{(3√15)(√5 -1)(5-2√5)^(1/4)},
0648イナ ◆/7jUdUKiSM
垢版 |
2018/06/13(水) 05:24:42.71ID:Oj2yj/8D
>>646正確な長さが出てるんですね。軒桁4.8xからもう整数比じゃないんですか。
屋根の端も微妙に3.7xじゃないみたいだし。
肉眼で0.074を出した。ここが限界です。
>>643ぜんぜん綺麗な比にならないのにこの形で極値をとる。なぞですね。ゴールドバーグさんは論文でこの形になる根拠を示したんですか。
0649132人目の素数さん
垢版 |
2018/06/13(水) 06:25:01.57ID:YkGfLvHx
綺麗な形にこだわるならx軸から見てもy軸から見ても正五角形のシルエットをもつ立体を試してみてはどうか
最適解とは異なりながらも0.0743は越えられるはず
0650132人目の素数さん
垢版 |
2018/06/13(水) 13:21:09.26ID:5ZmF3Enb
>>642
V/S^(3/2) はf個の点の配置に関して滑らかな関数だから
1.が本当にランダムなら、何度も試せば1回ぐらいは最大値に収束するはず。

ゴールドバーグの言う S^3/V^2 = 180.23 なる配置は、ネットで探しても見つからなかった。

>>494 が言うように、
> 今は 180.23 という値だけが何か間違っているという疑いの方が強まってる。
0652132人目の素数さん
垢版 |
2018/06/13(水) 14:55:08.60ID:ygq/w2vW
>>642
すばらしいですね。
多面体の各頂点の座標とか各面の重心の座標とかを求める処理は
1からコーディングすると大変そうだけど、なにかいいツールがあるのでしょうか?
(CAD系のツールでは基本処理なのかな?)

>>650
> V/S^(3/2) はf個の点の配置に関して滑らかな関数だから
> 1.が本当にランダムなら、何度も試せば1回ぐらいは最大値に収束するはず。
そうですね。逆に言うと、>>642の1回の計算だけでは、局所最適解に引っかかる
可能性があるということですね。
8面体の場合も初期値によっては正8面体に収束するかも。
初期値を変えて試行を繰り返して、なるべく多くの局所最適解を探してみたいところです。
(正解以外の局所最適解に収束する確率はかなり低いと予想されますが)

> 今は 180.23 という値だけが何か間違っているという疑いの方が強まってる。
計算機が自由に使える以前の時代の論文の数値計算の間違いというのは、
結構あるのかもしれません…
0653イナ ◆/7jUdUKiSM
垢版 |
2018/06/13(水) 14:59:41.71ID:Oj2yj/8D
>>649水平方向から見た(五角形+等脚台形)の射影を正五角形にすると。前>>648だいぶ平たくなりますね。V(x)が減りそう。五角形が綺麗なわけない。
0654132人目の素数さん
垢版 |
2018/06/13(水) 16:28:17.84ID:8DutWUYy
>>633 を実行してみました。
tha = tan α/2, thb = tan β/2として重心=垂線の足から連立方程式をたててみると

(3*tha^7-12*tha^5+9*tha^3)*thb^4
+(-2*tha^8-10*tha^6+38*tha^4-26*tha^2)*thb^3
+(11*tha^7-11*tha^5-11*tha^3+11*tha)*thb^2
+(-26*tha^6+38*tha^4-10*tha^2-2)*thb
+9*tha^5-12*tha^3+3*tha = 0
(tha^7+4*tha^5-5*tha^3)*thb^7
+(-15*tha^6-72*tha^4+23*tha^2)*thb^6
+(75*tha^5+120*tha^3-3*tha)*thb^5
+(12*tha^6-197*tha^4-72*tha^2+1)*thb^4
+(-tha^7+72*tha^5+197*tha^3-12*tha)*thb^3
+(3*tha^6-120*tha^4-75*tha^2)*thb^2
+(-23*tha^5+72*tha^3+15*tha)*thb
+5*tha^4-4*tha^2-1 = 0

>>621さんの数値データから得られる値

tha = 0.5006040925763866
thb = 0.1338964782891034

を代入して検算するとそれぞれの左辺値は

8.87931586213142e-6
3.632479806903177e-5

となってます。誤差なんだかどうなんだか。tha消去すると

8181*thb^62
-713988*thb^60+17155890*thb^58-164938703*thb^56+506017027*thb^54+1834844826*thb^52
-13744791488*thb^50+3826451839*thb^48+119593971621*thb^46-128477872952*thb^44-571856278634*thb^42
+693554443761*thb^40+1596500744027*thb^38-1841098161058*thb^36-2706178331076*thb^34+2845687450727*thb^32
+2845687450727*thb^30-2706178331076*thb^28-1841098161058*thb^26+1596500744027*thb^24+693554443761*thb^22
-571856278634*thb^20-128477872952*thb^18+119593971621*thb^16+3826451839*thb^14-13744791488*thb^12+1834844826*thb^10
+506017027*thb^8-164938703*thb^6+17155890*thb^4-713988*thb^2+8181 = 0

既約みたいです。
0655132人目の素数さん
垢版 |
2018/06/13(水) 18:24:34.99ID:+VZ1IBn7
八面体の人は別スレ立てて〜な
0657イナ ◆/7jUdUKiSM
垢版 |
2018/06/13(水) 22:00:17.82ID:Oj2yj/8D
>>649
真横から見て影が正五角形になるときですね。
(四面体の高さ)=1.8x√(5+2√5)
(四面体の高さ)=0.9(3-√5)x√(5+2√5)
(四面体の高さ)=1.8(√5-2)x√(5+2√5)

0.074は超えない気がするけど気になってはいます。前>>653めんどくさいなぁ。
0658132人目の素数さん
垢版 |
2018/06/14(木) 02:09:33.11ID:VSzXXZka
>>654
の連立方程式を数値的に解くと
 tha = tan(α/2) = 0.500612548452861
 thb = tan(β/2) = 0.133888590056153
ぐらいになりました。

>>621 さんの数値データから得られた値も(有効数字は)ほぼ一致してますね^^
0659132人目の素数さん
垢版 |
2018/06/14(木) 03:02:03.81ID:VSzXXZka
>>654
の連立方程式を数値的に解いて得られた、   >>658
 α = 53.1862428998954゚
 β = 15.2517985158774゚
はゴールドバーグの文献値に近いです。 >>492

また、cosβ = 0.964779066797437 はメディアル8面体の d_4 = d_5 と一致してます。>>582
0660132人目の素数さん
垢版 |
2018/06/14(木) 04:40:26.09ID:2oXVNEfm
状況をまとめると、

対称性のあるメディアル多面体を>>464のように3つのパラメータで表して
数値計算で最大値を求めた結果(>>471,489,621あたり)も、

Lindelofの条件のうち円に外接するということを先に前提として使った上で
α,βの2つのパラメータで表して、接点が重心という条件で
α,βを求めるというアプローチ(>>633)で得られた結果(>>658)も、

円に外接する際の接点をランダムに設定した上でそれが各面の重心に近づくよう
接点を動かしてLindelofの条件を満たす状態に近づけていくというアプローチで
得られた結果(>>642)も、

すべて(誤差を除き)同様の結果となり、
その結果の各面の対象軸からの角度はGoldbergの論文に記されている値と一致した(>>494,659)

ということですよね。
もうこれは、ここでの結論は
> 180.23という値だけが何か間違っている
ということで打ち止めでいいんじゃないですかね。これ以上やることもあまりないような。
さすがにこれ以上一つの話題を引きずるのも迷惑だし。
0661132人目の素数さん
垢版 |
2018/06/14(木) 12:41:01.69ID:VSzXXZka
>>654
 tha^2 = A,tha*thb = B とおく。

上の式に tha を掛けると
(A-1)・{3(A-3)B^4 + 2(-AA-6A+13)B^3 + 11(AA-1)BB + 2(-13AA+6A+1)B + 3(3A-1)A} = 0,
… Aについて2次方程式になる。

下の式に tha^4 を掛けると
(A-1)(A+5)B^7 + (-15AA-72A+23)B^6 + 3(25AA+40A-1)B^5 + (12A^3 -197AA -72A+1)B^4 + (-A^3+72AA+197A-12)AB^3 + 3(AA-40A-25)AABB + (-23AA+72A+15)AAB + (A-1)(5A+1)AA = 0,
0662イナ ◆/7jUdUKiSM
垢版 |
2018/06/14(木) 20:24:07.21ID:qiPHimn7
棟木を2x、垂木の最短の長さも2xとすると、屋根は等脚台形で、八面体を水平に見て射影が正五角形になるとき、
五角形の水平な対角線は、
(1+√5)x
八面体の真下にある底辺は、2x
八面体を真横から見て、
(八面体の高さ)=x√(5+2√5)
(八面体の高さ)=x/2√(10-2√5
(八面体の高さ)=x√(5+2√5)-2x√(10-2√5)
>>657訂正&再考ですが、極値0.074……を超えないとなると、計算をつづけるに値するほど魅力的な式じゃないです。
0663132人目の素数さん
垢版 |
2018/06/15(金) 09:54:18.30ID:w+/1B0FC
>>654です。まだ次の問題でてきてないので私もその前に最後のレス。
>>654の最後の式既約ではありませんでした。

262144*(thb−1)^3*thb^9*(thb+1)^3*(thb^2+1)^3*(thb^4−10*thb^2+1)*(8181*thb^36−623997*thb^34+10242837*thb^32
−48965288*thb^30−59994180*thb^28+888366516*thb^26−574079300*thb^24−5645292312*thb^22
+4166321790*thb^20+19707900690*thb^18+4166321790*thb^16−5645292312*thb^14−574079300*thb^12
+888366516*thb^10−59994180*thb^8−48965288*thb^6+10242837*thb^4−623997*thb^2+8181)

となっていてthbはQ上36次方程式の解になっています。
この36次方程式は複2次式、かつ相反方程式になっているので指数4の部分体を持ちます。
具体的には (tan^2β + 1/tan^2β)/2 = 4/(1-cos 2β) -1 = C とおくとき C は9次の方程式

8181*x^9 -623997*x^8+10242837*x^7
-48965288*x^6-59994180*x^5+888366516*x^4
-574079300*x^3 -5645292312*x^2+4166321790*x+19707900690 = 0 …… (*)

の解でおそらく [Q(thb) : Q(C) ] = 4、[Q(C) : Q] = 9 です。
おそらくと書いたのは(*)の左辺の既約性がまだ未確認なんですが、 Maxima君もWolfram先生も因数分解できないのでほぼ確定だと思ってます。
兎にも角にもthbの満たす方程式が得られたので必要に応じていくらでも精度の高い解を得ることができます。
問題が残ってるとすれ(*)がホントに代数的に解くのは無理なのかという事、つまりQ(C)/Qの単純性です。
(*)が代数的に解けるのはQ(C)/QがQ上3次の中間体Q(C)/M/Qを持つときですが、それを確認する方法がなかなかうまく思いつきません。
原理的にはQ(C)/QのGalois閉包のGalois群を計算すればいいんですが、そんなの手計算でやるのも無理だし、コード組むのも一手間だし、
何より、多分Q(C)/Qは単純だった、なら労多くして得るもの少ない感じでちょっとやる気が起きません。
0664132人目の素数さん
垢版 |
2018/06/15(金) 09:54:45.34ID:w+/1B0FC
最後に方程式導出したmaximaのコード貼っときます。

ca(tha,thb) := (1-tha^2)/(1+tha^2); sa(tha,thb) := 2*tha/(1+tha^2);
cb(tha,thb) := (1-thb^2)/(1+thb^2); sb(tha,thb) := 2*thb/(1+thb^2);
a(tha,thb) := (ca(tha,thb) + cb(tha,thb))/(1+ca(tha,thb)*cb(tha,thb)-sa(tha,thb)*sb(tha,thb));
b(tha,thb) := (sa(tha,thb) - sb(tha,thb))/(1+ca(tha,thb)*cb(tha,thb)-sa(tha,thb)*sb(tha,thb));
c(tha,thb) := (1-b(tha,thb)*sb(tha,thb))/cb(tha,thb);
d(tha,thb) := (sa(tha,thb) - sb(tha,thb))/(sa(tha,thb)*cb(tha,thb));
e(tha,thb) := 1/sa(tha,thb);
ga(tha,thb) := (2*c(tha,thb)+d(tha,thb))/(3*c(tha,thb)+3*d(tha,thb))*b(tha,thb)
+ (c(tha,thb)+2*d(tha,thb))/(3*c(tha,thb)+3*d(tha,thb))*e(tha,thb);
gb(tha,thb) := ((e(tha,thb)-b(tha,thb))*a(tha,thb)*(e(tha,thb)+2*b(tha,thb))
+2*b(tha,thb)*a(tha,thb)*b(tha,thb)
+2*b(tha,thb)*c(tha,thb)*(-b(tha,thb)))
/(a(tha,thb)*e(tha,thb)+a(tha,thb)*b(tha,thb)+2*b(tha,thb)*c(tha,thb))/3;

factor(ga(tha,thb)-sa(tha,thb));
factor(gb(tha,thb)-sb(tha,thb));
0665132人目の素数さん
垢版 |
2018/06/15(金) 15:45:24.73ID:mm39PC7P
>>663

thb^2 + 1/thb^2 = 2{4/(1-cos 2β) - 1} = C とおくと、上の式の最後の因子は

(thb^18){8181*C^9 -623997*C^8 +10169208*C^7 -43973312*C^6 -131473152*C^5 +1169678304*C^4 -130954112*C^3 -9629462016*C^2 +5516962560*C +32871900928}

となるので{ }内を0とおいて
 C = 55.80233866564161431594753276684087477826
thb = tan(β/2) = 0.1338885900561525235645099960430550447846
β = 15.25179851587733214293978022621725452035゚
ですね^^
0666132人目の素数さん
垢版 |
2018/06/15(金) 18:25:08.44ID:NyOBeIuX
すいません。Cの方程式まちがった。

2094336*C^9−79871616*C^8+650829312*C^7−1407145984*C^6
−2103570432*C^5+9357426432*C^4−523816448*C^3
−19258924032*C^2+5516962560*C+26289900809

です。改訂版。Cの方程式まで一気に作らせます。
よくよく考えたら関数として定義する意味なかった。
基本これで最後です。
もしCが代数的に解けたりしたらまた書くかも。
いまのとこ望み薄ですけどねぇ。

load ("orthopoly");
ca: (1-tha^2)/(1+tha^2); sa: 2*tha/(1+tha^2);
cb: (1-thb^2)/(1+thb^2); sb: 2*thb/(1+thb^2);
a: (ca + cb)/(1+ca *cb - sa *sb);
b: (sa - sb)/(1+ca *cb - sa *sb);
c: (1-b * sb)/cb;
d: (sa - sb)/(sa *cb);
e: 1/sa;
ga: (2*c +d)/(3*c +3*d)*b+ (c +2*d)/(3*c+3*d)*e;
gb: ((e-b)*a*(e+2*b)+ 2*b*a*b+ 2*b*c*(-b))/(a*e + a*b + 2*b*c)/3;

num(factor(ga - sa));
eq1:part(num(factor(ga - sa)),3);
eq2:num(factor(gb - sb));
eqc:part(factor(resultant(eq1,eq2,tha)),7);
sum(coeff(eqc,thb,2*k+18)*expand(chebyshev_t(k,C)),k,0,9),factor;
0667132人目の素数さん
垢版 |
2018/06/15(金) 19:00:41.39ID:NyOBeIuX
コードの最後の行

sum(coeff(eqc,thb,2*k+18)*expand(chebyshev_t(abs(k),C)),k,-9,9),factor;

で結果は

256*
(16362*C^9−623997*C^8+5084604*C^7−10993328*C^6−16434144*C^5+73104894*C^4−4092316*C^3−150460344*C^2+43101270*C+128405863)

でした。スレ汚しスマヌ…orz。
0669132人目の素数さん
垢版 |
2018/06/16(土) 01:03:52.88ID:a+j3J/Zw
まぁ面白かった。数値に関しては原論文超えてる?ひとえに計算機のおかげだけど。
0670132人目の素数さん
垢版 |
2018/06/17(日) 00:07:32.71ID:NrfBnVbQ
気分一新で再開しませんか?

nを自然数、xを実数とするとき
 
 [nx] ≧ Σ[k=1,n] [kx]/k

を示せ。ただし[x]はガウス記号である。
0671132人目の素数さん
垢版 |
2018/06/17(日) 01:13:46.23ID:lI+JiKnS
それにしても よく間違う人だった。(他人のことは言えないが…)
0672132人目の素数さん
垢版 |
2018/06/17(日) 02:56:37.20ID:ratqIZM6
>>669
論文の値は実際には存在しえない間違った「いい値」だったのではないかという話を
ずーーーっとやってたのに、何を見ていたのか…
0674132人目の素数さん
垢版 |
2018/06/17(日) 15:53:47.30ID:ratqIZM6
実際は全く議論に参加できていないのに無意味な発言や計算を大量に垂れ流して
事情がわからない人が見たらそいつが議論の中心にいるかのような錯覚を招きかねない
存在自体が「叙述トリック」のような奴が1人いる。

遡って話をトレースしたい人のために忠告しておくと、
イナ ◆/7jUdUKiSM
とかいうコテハン氏の発言およびそれに対するレスポンスは全部スキップすると、
内容が把握しやすいのでオススメです。
0677132人目の素数さん
垢版 |
2018/06/17(日) 16:38:39.07ID:LEBIHDAI
幾何の難問
https://jmoss.jp/mon/old.php?type=viewproblem&;d=b009

上の問題なのですが凸多面体の定義より
面同士の角は外側から測ると全てπ以上であることと
辺が3本以上あることからは示せますか?
0678132人目の素数さん
垢版 |
2018/06/17(日) 17:36:34.62ID:Mnf6xpK6
江戸末期の田舎の下級武士に経済ユダヤが支援してテロを起こさせ江戸幕府を転覆させたのが明治維新。
江戸末期から日本は経済ユダヤとの繋がりがありお互いの利益の均衡を目指してきたのが今日までの政治
の中心課題だと言えます。複式簿記 資本主義 株式制度 現在の経済の根幹を作ったのは彼等であり、
全ての産業を掌握する彼等(総資産数京円以上)の意向を無視出来ません。旧ソ連 中国共産党 北朝鮮 
ISISを作ったのは彼等であり、日本の技術流出 東芝の半導体事業からの撤退、シャープの倒産全て彼らの
シナリオ通りに動いてます。また、ここ数百年における世界の全ての紛争、戦争は彼等によって引き起こさ
れました。
彼らの目指している世界は自分達を支配階級とした人類の管理であり歯向かう人間の排除です。 
私達が右や左と罵り合う姿は彼らにとって好都合であり、対立は彼らの支配体制の強化になります。そういっ
たことを全ての日本人が理解しないと同じことを繰り返し、十数年後 あの時安部が日本を滅茶苦茶にした。 
今度の保守の誰々さんこそ日本を救うと喚いてるかもしれません。消費税廃止 移民反対と当たり前のことを
各政治家に要求し続けると同時に政治家は全員ユダヤの手先だと疑い続けないと日本の独立は成し得ません。

世界中の人間が知るべきこと

・世界の全てのメディアはユダ金が牛耳っている。

・トランプ プーチン 習近平 安部 麻生 テリーザ・メイ メルケル 文在寅 金正恩はユダ金の手下であり仲間である。
テレビに出てる有名な政治家は国内外問わず全員ユダヤの手先だと考える事。右や左などによる対立は茶番である。

・全てのテロと紛争と戦争は、ユダ金達と軍産複合体によって引き起こされている。
0679132人目の素数さん
垢版 |
2018/06/17(日) 17:36:34.73ID:Mnf6xpK6
江戸末期の田舎の下級武士に経済ユダヤが支援してテロを起こさせ江戸幕府を転覆させたのが明治維新。
江戸末期から日本は経済ユダヤとの繋がりがありお互いの利益の均衡を目指してきたのが今日までの政治
の中心課題だと言えます。複式簿記 資本主義 株式制度 現在の経済の根幹を作ったのは彼等であり、
全ての産業を掌握する彼等(総資産数京円以上)の意向を無視出来ません。旧ソ連 中国共産党 北朝鮮 
ISISを作ったのは彼等であり、日本の技術流出 東芝の半導体事業からの撤退、シャープの倒産全て彼らの
シナリオ通りに動いてます。また、ここ数百年における世界の全ての紛争、戦争は彼等によって引き起こさ
れました。
彼らの目指している世界は自分達を支配階級とした人類の管理であり歯向かう人間の排除です。 
私達が右や左と罵り合う姿は彼らにとって好都合であり、対立は彼らの支配体制の強化になります。そういっ
たことを全ての日本人が理解しないと同じことを繰り返し、十数年後 あの時安部が日本を滅茶苦茶にした。 
今度の保守の誰々さんこそ日本を救うと喚いてるかもしれません。消費税廃止 移民反対と当たり前のことを
各政治家に要求し続けると同時に政治家は全員ユダヤの手先だと疑い続けないと日本の独立は成し得ません。

世界中の人間が知るべきこと

・世界の全てのメディアはユダ金が牛耳っている。

・トランプ プーチン 習近平 安部 麻生 テリーザ・メイ メルケル 文在寅 金正恩はユダ金の手下であり仲間である。
テレビに出てる有名な政治家は国内外問わず全員ユダヤの手先だと考える事。右や左などによる対立は茶番である。

・全てのテロと紛争と戦争は、ユダ金達と軍産複合体によって引き起こされている。
0680132人目の素数さん
垢版 |
2018/06/17(日) 20:13:55.19ID:S9i0Ooes
>>676
私の持ってる解答はこんな感じです。

f(x) = [nx] - Σ[k=1,n] [kx]/k

とおけば周期1で不連続点以外のとこでは定数、不連続点では右連続です。
(0,1]での不連続点は0≦b<a≦n である互いに素な整数a,bを用いてx = b/aとかける点です。
よってそのようなa,bについてf(b/a)≧0を示せばよいことになります。
0681132人目の素数さん
垢版 |
2018/06/17(日) 20:18:47.70ID:S9i0Ooes
>>677
示せない。
例えば正6面体のときは12個ある辺の外角はすべてπ/2でπより大きいということはない。
そもそも通常の幾何学的な本来の意味での角の大きさは0以上π以下です。
いわゆる “一般角” と混同してはいけない。
0682イナ ◆/7jUdUKiSM
垢版 |
2018/06/18(月) 12:33:15.13ID:X9qz/j/u
>>673
一辺xの立方体の体積は
x^3
一辺xの正三角形の面積は
x^2√3/4

四角形どうしがとなりあう辺xのビジュアル八面体の体積もこういう一般的なかたちにならないでしょうか。
0683132人目の素数さん
垢版 |
2018/06/18(月) 14:01:24.30ID:No1r8RIC
相似形なら面積は特定の辺の二乗に比例するし体積は三乗に比例する
そのことと比例係数が代数的に書けるかどうかは別問題

過去レスにあった通り、例えば半径1の球に外接する多面体に限定すれば表面積Sと体積Vは比例するため、SまたはVの最小化問題のみを考えればよい
ただ、この性質を利用して立式しても、五次以上の次数の方程式を解くことになるので結局代数的には解けないんじゃないか、という説が現在有力

「そうじゃない、うまく式を立てれば代数的に解けるはずだ」という可能性があるならトライしてみたらいいんじゃないかな
0684132人目の素数さん
垢版 |
2018/06/18(月) 23:10:58.84ID:Y/8tBeky
>>680 をすこし進めます。

1≦b≦a≦nである互いに素な整数a,bに対しbk÷aのあまりをr(k)とすると

[nb/a] = nb/a - r(n)/a
Σ [1≦k≦n] [kb/a]/k = Σ [1≦k≦n] (kb/a - r(k)/a)/k = nb/a - Σ [1≦k≦n] r(k)/(ak)

なので 示すべきは

r(n)/a ≦ Σ [1≦k≦n] r(k)/(ak)

です。
0685685 ◆3srs.EvKNg
垢版 |
2018/06/19(火) 00:05:11.82ID:pnke3C+M
「面白い問題おしえて〜な」とのことなので、問題を教えるだけです、っていうか解答いただけると嬉しいです(当方解答を持ち合わせておりません)。

[問題]
1桁の自然数をいくつか掛け合わせて表せない最小の自然数は11である。
では1桁の自然数をいくつか掛け合わせて表せる数 a , b を加えて表せない2以上の最小の整数は何か。もし存在しない場合はそれを証明せよ。
0686685 ◆3srs.EvKNg
垢版 |
2018/06/19(火) 00:09:04.08ID:pnke3C+M
>>685
例えば121は1+120=1+3*5*8で表せてしまうので不適になります。
0687132人目の素数さん
垢版 |
2018/06/19(火) 00:38:43.76ID:S2GWbT4K
整数の積と和を組み合わせた問題は、大抵難問。

1000桁前後の自然数に対してこれを応用した暗号が作れるかもね。
0688132人目の素数さん
垢版 |
2018/06/19(火) 01:07:06.92ID:AsZ9maAx
235らしい。By Haskell君

parts = sort [a*b*c*d*e*f*g*h | a<-[1,2],b<-[1,3],c<-[1,4],d<-[1,5],e<-[1,6],f<-[1,7],g<-[1,8],h<-[1,9]]
isNotSum x = (==Nothing) $ find (==x) [a+b|a<-parts,b<-parts]
head [x|x<-[2..],isNotSum x]
0689132人目の素数さん
垢版 |
2018/06/19(火) 01:15:19.25ID:B4wkEBhB
>>685
311っぽいですな。
1桁の自然数をいくつか掛け合わせて表せない ⇔ 2,3,5,7以外の素因数を持つ
ということで、プログラムで検索した結果。
0691132人目の素数さん
垢版 |
2018/06/19(火) 01:30:54.68ID:B4wkEBhB
>>688
同じ1桁の数を複数掛けてもいいのでは?
(出題意図がどちらなのかはわからないけど。)
0692685 ◆3srs.EvKNg
垢版 |
2018/06/19(火) 01:31:37.76ID:pnke3C+M
>>689
ありがとうございます。
意外と小さい数でしたね……
0693685 ◆3srs.EvKNg
垢版 |
2018/06/19(火) 01:34:16.80ID:pnke3C+M
すみません、同じ数は何度掛けてもOKのつもりでした。
235は2*2*5*5+5*7で表せますけど、同じ数がダメだと表せないっぽいですね……
0694132人目の素数さん
垢版 |
2018/06/19(火) 01:37:32.81ID:B4wkEBhB
かぶった。
ちなみに、1000以下では
311,479,551,619,622,671,719,839,851,933,937,958
の12個。
10000以下では1099個。
数が大きくなると、出現頻度は増える。
(nが大きくなると、nの周辺で2,3,5,7のみで表される数なんてほとんどなくなるから)
0695685 ◆3srs.EvKNg
垢版 |
2018/06/19(火) 01:41:29.61ID:pnke3C+M
>>694
なるほど…… 先の解析結果までご丁寧に教えてくださりありがとうございます。
0696132人目の素数さん
垢版 |
2018/06/19(火) 02:24:51.70ID:AsZ9maAx
今更ながらhaskell君にも聞いてみました。

Prelude Data.List> let isgood x = if x == 1 then True else (/= 0) $ head $ [a|a<-[2,3,5,7],mod x a == 0, isgood $ div x a] ++ [0]
Prelude Data.List> let ys = [x|x<-[2..],(==0) $ head $ [a|a<-[1..x-1], isgood a, isgood (x-a)] ++ [0]]
Prelude Data.List> take 10 ys
[311,479,551,619,622,671,719,839,851,933,937,958,1102,1103,1117,1151,1193,1238,1244,1291]
0698132人目の素数さん
垢版 |
2018/06/22(金) 13:19:28.28ID:SuXdtRwP
4人でリーグ戦(総当たり戦)を行います。
勝てば3点、引き分ければ1点、負ければ0点を獲得します。
全試合が終わった後、合計点数の順に順位をつけます。
ただし、同じ点数の人がいれば、その人たちでクジを行い、
最終的には無理矢理1位から4位の順位をつけます。
任意の対戦において、勝つ、負ける、引き分けるは 確率 1/3 で起こるものとします。

問0
「x点しかとれなかったけど、2位になった」
「y点も取ったけど、3位だった」
ということが起こる、最小のxと、最大のyを求めよ。

問1
m位の人の合計点数の平均を求めよ(mは1,2,3,4)

問2
合計点数kを取った人が、上位2名に入っている確率を求めよ(kは8を除く9以下の整数)
0699132人目の素数さん
垢版 |
2018/06/22(金) 16:54:53.78ID:5dKvywCX
〔問題〕
 最高次の係数が1であるn次の整多項式を Pn(x) とし、
 Pn(x) = 0 のn個の解を α1,α2,…,αn とする。

 このとき、α1^3,α2^3,…,αn^3 を解にもつ、
 最高次の係数が1のn次の整多項式 An(x) を求めよ。

http://www.toshin.com/concours/mondai/mondai29.php

 P(x) = p0(x^3) + p1(x^3) x + p2(x^3) xx,
と表わせる。。。
0702132人目の素数さん
垢版 |
2018/06/22(金) 23:21:47.30ID:/GProLmv
>>701
いや、問題が見れないんじゃなくて、>>699は解答になんか自明でない決めつけから始まってるってんでしょ?もう、そういう決めつけから始まる解答になってない。若干おかしいけど大筋治ってる。直す前のやつ見たいなぁと。
0703132人目の素数さん
垢版 |
2018/06/22(金) 23:24:53.34ID:/GProLmv
ただ、大筋なおってるっていってもPの既約性示せてないからアウトなんだけどね
0704132人目の素数さん
垢版 |
2018/06/22(金) 23:28:15.49ID:/GProLmv
間違った。既約性ではなく、重解持たないこと。それはPが既約なのでただしい。それ以外の方法で重解持たないこと示せれば問題ないけど解答にはその旨全くない。
0705132人目の素数さん
垢版 |
2018/06/22(金) 23:32:20.85ID:mDZvFtTn
挫折して予備校講師になった素人の書いた模範解答だから仕方あるまい。
0706132人目の素数さん
垢版 |
2018/06/22(金) 23:41:16.11ID:/GProLmv
でもこれ作った人気づいてないと思えないんだよねぇ?
Pの既約性がいかにもEisensteinの既約判定使ってねって形になってる。
偶然なのかもしれないけど。
必要なのわかってて、あえて簡単に解けるように見せかけてためにはぶいたんだとしたらあまりにも悲しいけど。
ホントに気づいてないなら論外だけど。
そんなことしてたらかえって東進の名にキズがつくような希ガス。
0707132人目の素数さん
垢版 |
2018/06/22(金) 23:57:37.66ID:mDZvFtTn
Y-SAPIX の円順列の問題のときも、模範解答が間違っていて、
「今月は正解者が一人もいませんでした」とか書いていたよな。
そりゃそうだろ、あほか?
あとでこっそり模範解答を差し替えて知らんぷりしていたが、正解者は呆れただろうな
0708132人目の素数さん
垢版 |
2018/06/23(土) 00:38:27.43ID:shdFVkoM
解答が不完全なのに気づいてないなら問題外。
問題の作りからしてそれはないと思うけどそれならそれで大問題。
どうせ不完全なの高校生が気づくわけないとみこして敢えて不完全な解答のせて “うわぁ、こんな簡単に解けたのか!” 感を演出のは道義的にいかん希ガス。
0709132人目の素数さん
垢版 |
2018/06/23(土) 02:19:05.69ID:BnO9HX6O
>>677

凸多面体Pを 任意の向き(↑Ox)に正射影する。
その輪郭は凸m角形となる。(m≧3)
各辺 e_i に対応するPの稜 L_i があって、それらは相異なる。
稜L_iの両側の2面(j,k)は、こちら向き & あちら向きである。
その外向き法線を n_j,n_k とすると、 (↑Ox・↑n_j)(↑Ox・↑n_k) < 0,

L_i (n_j,n_k) に対し、この条件を満たす「接する」向き ↑Ox の存在範囲は、
 平面jの外側で平面kの内側、または、平面jの内側で平面kの外側
であり、立体角4θ_iの範囲となる。(θ_i は稜L_iの両側の2面のなす角)

一方、任意の向き↑Oxに対し、この条件を満たす「接する」稜が3本以上ある。(m≧3)

∴ すべての稜についての立体角の総和 Σ_i (4θ_i) は 3Ω = 12π 以上でなくてはならない。

∴ 両辺を4で割れば示すべき不等式を得る。
0710132人目の素数さん
垢版 |
2018/06/23(土) 02:26:54.92ID:shdFVkoM
>>698
http://codepad.org/pfBCZWvD

import Data.List
import Data.Ratio

gameRes = [(3,0),(1,1),(0,3)]
results = [[a,b,c,d] |
ab <- gameRes, ac <- gameRes, ad <- gameRes,
bc <- gameRes, bd <- gameRes, cd <- gameRes,
let a = sum [fst ab,fst ac,fst ad],
let b = sum [snd ab,fst bc,fst bd],
let c = sum [snd ac,snd bc,fst cd],
let d = sum [snd ad,snd bd,snd cd]
]
posOf0GoFinal result = let
p = head result
fstPt = head $ reverse $ sort $ result
nFsts = length $ filter (==fstPt) result
sndPt = head $ tail $ reverse $ sort $ result
nSnds = length $ filter (==sndPt) result
in
case True of
_| nFsts >= 2 && p >= fstPt -> 2%(fromIntegral nFsts)
| nFsts >= 2 && otherwise -> 0%1
| p == fstPt -> 1%1
| p == sndPt ->1%(fromIntegral nSnds)
| otherwise -> 0%1

question1 = id
$ map ((*(1%( length $ results))).fromInteger)
$ map sum
$ transpose
$ map sort
$ results

question2 = [ (pt,totalPosOf0GoFinal / nCases)|
pt <-[0..9],
let suitCases = filter ((== pt).head) results,
let nCases = fromIntegral $ length suitCases,
let totalPosOf0GoFinal = sum $ map posOf0GoFinal suitCases,
nCases /= 0
]

main = do
print question1
print question2

[1073 % 729,779 % 243,127 % 27,4825 % 729]
[(0,0 % 1),(1,0 % 1),(2,1 % 81),(3,17 % 216),(4,44 % 81),(5,80 % 81),(6,79 % 81),(7,1 % 1),(9,1 % 1)]
0711132人目の素数さん
垢版 |
2018/06/23(土) 02:48:51.81ID:BnO9HX6O
>>709

稜L_i の方向から見ると、条件を満たす向き ↑Ox の存在範囲は、
平面jと平面kに挟まれた中心角θ_i の部分×2 だから
(θ_i/π)Ω = 4θ_i
Ω = 4π
0712132人目の素数さん
垢版 |
2018/06/23(土) 02:50:01.62ID:shdFVkoM
今更ながらよくよく見るとこれあってんの?
勝ち点6取った場合の予選突破確率のほうが
勝ち点5取った場合の予選突破確率より低い?
どっか間違った?
あってるなら意外でおもしろいんだけどなぁ。
0713132人目の素数さん
垢版 |
2018/06/23(土) 11:00:35.69ID:CC9xpxXb
>>710 >>712
私の用意していた数値と一致です。
勝ち/負け/引き分けを同確率という設定が、現実的ではありませんが、とりあえず、
あのオリンピックの時の悲劇(勝ち点6で予選敗退)の様なことは、そう珍しいことでも
無いのかなと思って計算して(させて)みたんですが、予想外に低いのでびっくりしました。
勝ち点2で予選突破できる確率の倍です。

>>勝ち点6取った場合の予選突破確率のほうが
>>勝ち点5取った場合の予選突破確率より低い?
勝ち点の分布が 6660(=1弱3竦)となって予選敗退するのと、
勝ち点の分布が 5550(=1弱3平)となって予選敗退するケースの比較になります。
「三チームの勝ち点が同じ」と言っても、3竦みの場合は、a>b>c>a、と a<b<c<a という
二つのケースがあるけど、引き分けの場合は、a=b=c しかないことに由来します。
勝ち点を、3,1,0 と設定していることにも起因していますね。
0714132人目の素数さん
垢版 |
2018/06/23(土) 11:39:56.96ID:SeCu6IK8
>>712
サッカーで4チームでの総当たり上位2チーム予選突破、の話な。

勝ち点6取っても予選突破できないのは、3チームが2勝1敗,1チームが全敗のケースだけ
勝ち点5取っても予選突破できないのは、3チームが1勝2分,1チームが全敗のケースだけ

どの対戦カードも勝ち負け引き分けがそれぞれ1/3で、
勝ち点が並んだら抽選という単純なモデルで考えると、
勝ち点6を取ったという条件での予選突破できない条件付き確率は2/81
勝ち点5を取ったという条件での予選突破できない条件付き確率は1/81

なので、あながち間違ってはいない。

ただ、もちろんそこまで力が拮抗してるというモデルはあまり現実的ではないし、
引き分けが1/3というのが妥当かも不明だし、
それ以前に、その条件付き確率が意味を持つシチュエーション自体が存在しない。
(全6試合のうち当該チームだけが3試合消化し、残り3試合はまだ実施されていない
なんて状況は通常ありえないし、そもそも当該チームが2試合消化した時点で
勝ち点6と勝ち点5の可能性の両方が残ってることはないわけで…)
0715132人目の素数さん
垢版 |
2018/06/23(土) 12:42:19.36ID:CC9xpxXb
>>714
少し補足すると、リーグ戦全体は、6試合あるので、3^6通り考えることができ、さらに4人いるという
事を考え、分母を4*3^6とする、勝ち点の分布は
0:108     1:324     2:324     3:432
4:648     5:324     6:324     7:324     9:108
となります。
偶然(?)にも、勝ち点5や6となるケース数は一致します。
従って、勝ち点5や6で予選落ちする確率の比較は、パターン数の比較に
置き換えて考えることができ、>>713のような検討が可能となります。

>>714の後半をみると、「自分が勝ち点6or5をとった場合」として計算されているようですが、
この問題の設定やプログラムでは、「3^6通りあるリーグ戦全体の結果」を平等にあつかい、
その中で、勝ち点が5や6になるケースを抽出して比較してるので、ご安心ください。
0716132人目の素数さん
垢版 |
2018/06/23(土) 13:03:45.94ID:SeCu6IK8
リロードしてなかったので、混乱させたならすまない。
>>714は別に誰かに反論するというような意図で書いたわけではないので。
0717132人目の素数さん
垢版 |
2018/06/24(日) 20:21:00.73ID:C9Q8KS7h
>>670 >>680 >>684
もう答えかきますね。>>684の続き。

  Σ [1≦k≦a-1] r(k)/(ak) ≧ (a-1)/a

を示せば十分である。aとbは互いに素であるのでb×はZ/aZ上の全単射をあたえているからr(1),…,r(a-1)は1,…,a-1の並べ替えになっている。
よってΣ [1≦k≦a-1] r(k)/(ak)はr(k)が ”小さいもの順” に並んでいるときの値以上である。よって

 Σ [1≦k≦a-1] r(k)/(ak) ≧ Σ [1≦k≦a-1] k/(ak) =(a-1)/a。    □

参考までに等号成立はx≦[x]+1/nのときです。
0720132人目の素数さん
垢版 |
2018/06/24(日) 22:43:28.29ID:ne7opqz5
> Σ [1≦k≦a-1] r(k)/(ak) ≧ (a-1)/a

>を示せば十分である。

なんでこのケースだけ示せば十分なんですか?
0721132人目の素数さん
垢版 |
2018/06/24(日) 22:57:49.07ID:jLCQQPbm
>>720

>>684

>なので 示すべきは

>r(n)/a ≦ Σ [1≦k≦n] r(k)/(ak)

まできていて


左辺≦(a-1)/a
右辺≧ Σ [1≦k≦a] r(k)/(ak)

なので
0724132人目の素数さん
垢版 |
2018/06/26(火) 01:16:11.00ID:zS+7aIhZ
別スレでプロの数学者でもパズル系は苦手とする人もという話題がでてたのでちなんだ問題を。
Peter Winklerの数学パズルの本に載ってた問題、曰く、"Conway Immobilizer"。

----
1,2,3と表面に書かれたカードが1枚づつ、計3枚のカードとカードの山をおける三ヶ所の場所A、B、Cがある。
三ヶ所それぞれにカードを分けて表面を上にして山を作り配置した状態を考える。
たとえばAに下から順に1,2をおき、Bに3をおき、Cには何も置かないなどである。
あなたの仕事は機械をプログラムしてAの山に上から順に1,2,3という順でカードが置かれている状態(終了状態とよぶ)に移行させることである。
機械にできることは山の一番上に乗っているカードの数字を読み取り、その数字の組み合わせのみに応じていずれかの山の一番上のカードを別の山の一番上に乗せ変えることだけである。
機械には状態を記憶する能力はなく、常にその時の各山の一番上に見えているカードの組み合わせのみに応じてしか次に行う操作を決めることしかできない。
たとえば先の例の状態であれば各山に見えているカードはA:2、B:3、C:空であり、この状態においてあなたは例えば機械にA→CやB→Aのような形で行う操作を指定できる。
無論C→Aなどは指定できない。
山の見えている最上面の状態は23通りあり得るので、あらかじめその23通りそれぞれに対して可能な操作を一つずつ指定しておいて、いかなる状態からスタートしても機械が自動的に最終的に終了状態に到達できるようにしてほしい。
なお、機械は終了状態になれば自動的に終了する装置がついているので、操作を終了させる条件について考慮する必要はないとする。
----

このパズル問題を出題された著名な数学者 JOHN CONWAY が6時間(だったかな?)考え込んで思考の泥沼にはまってしまったという逸話つきの問題です。
ネットで検索すれば解答は出てくるとは思いますがよかったら考えてみて下さい。
0725132人目の素数さん
垢版 |
2018/06/26(火) 03:10:01.84ID:o5dj2kDl
>>724
なぜ23通り?
1つ見えている:3通り
2つ見えている:18通り
3つ見えている:6通り
で、計27通りではないの?
何か問題文読み間違ってる?
0726132人目の素数さん
垢版 |
2018/06/26(火) 03:18:10.71ID:zS+7aIhZ
>>725
失礼しました。
1枚見えてる:3(=どのカードが見えてるか)×3(=どの山に見えてるか)=9
2枚見えてる:3(=どのカードが見えてるか)×6(=どの山に見えてるか)=18
3枚見えてる:1(=どのカードが見えてるか)×6(=どの山に見えてるか)=6
です。
多分正しく解釈されてると思います。
0727132人目の素数さん
垢版 |
2018/06/26(火) 04:59:43.99ID:o5dj2kDl
>>726
では、例えばこんな感じ?

左から順にA,B,Cとする。ただし、Aの左隣はC,Cの右隣はAと解釈する。
この設定で、以下のルールで処理すればよい。
(1) 1枚のみ見えているときは、その1枚を左隣に移動する
(2) 3枚とも見えているときは、2のカードを左隣に移動する
(3) 2枚のみ見えているときは、2空1の場合を除き、
  空いている場所の右隣のカードを空いている場所に移動する
(4) 2空1の場合、C→A

(3)の例外と(4)がなければ、すべてのパターンから同じ無限ループに収束するが、
(4)のルールがループを切って、ゴールへの道が見える。
(4)のルールで分岐するケースは3の場所により2通りあるが、
どちらにせよゴールにたどり着く。
0728132人目の素数さん
垢版 |
2018/06/26(火) 05:20:21.23ID:o5dj2kDl
まあ、(1)のルールは、見えているカードをどこに移動しても構わないのだけど。
0729132人目の素数さん
垢版 |
2018/06/26(火) 12:36:03.50ID:p6aNDz2K
>>727

正解のようです。
http://codepad.org/BGh67AnT
この手の問題は結局コード組んでみないと正解かどうかわからないので組んでみました。
かなり遅いですが実用上問題なしということで。
可読性優先。

では発展でカードの枚数が n ではどうでしょうか?
0731132人目の素数さん
垢版 |
2018/06/26(火) 23:01:40.88ID:myYLliSP
>>717
これもおながいします。

〔問題602〕
正整数nと、1より大きい正の実数xに対し、
 Σ(k=1,n) {kx}/[kx] < Σ(k=1,n) 1/(2k-1)
{x} = x - [x] を表し、[x] はxを超えない最大の整数を表すものとする。

不等式スレ9 - 602
0732132人目の素数さん
垢版 |
2018/06/30(土) 12:49:32.17ID:EwRMB19m
xyz座標空間上の曲面P:z=x^2-y^2について
P上の二点を結ぶP上の曲線で長さが最短となるものはただ一つのみであることを示せ
0733132人目の素数さん
垢版 |
2018/07/01(日) 05:28:33.10ID:PiobKfWu
>>731
でけたかも。
x>0においてf(x) = Σ(k=1,n) {kx+k}/[kx+k] ,g(x) = lim[e→+0] f(x-e) とおく。
このとき
g(x) = (kx+1+[-kx])/(k-1-[-kx])
である。
またf(x)≦g(x)で等号が成立するのはf(x)の連続点のみである。
さらに与式の右辺はg(1)に一致する。
よってg(x)がx=1においてまたその点においてのみ最大値を持つことを示せば良い。
またg(x)は右連続で連続点において単調増大だから1でない不連続点xにおいてg(x) < g(1)を示せば良い。
またg(x+1)<g(x)から0<x<1として良い。
よって0<b<a≦nである自然数a,bをもちいてx=b/aとおける。
r(k) = a[-bk/a] + bkとおくとき
g(b/a) = Σ(k=1,n) (a-r(k))/((a+b)k - (a-r(k)))
となる。
[-bn]≦c≦[-b]であるcを固定し[-bk]=cであるkの全体をl,l+1,…,mとおく。
l≦k≦mにおいてr(k) = px+qとなるp,qがとれる。
d(x) = 1/(2x-1) - (px+q)((a+b)k-(px+q))
とおく。
l,mのとり方からpl+q≦b, pm+q≦aがわかるから特にD((l+m)/2)≧0がわかる。
さらにl≦x≦mにおいてd(x)は単調減少、下に凸より任意のl≦m'≦mに対して
Σ(k=1,m') (a-r(k))/((a+b)k - (a-r(k)))≧0
がわかる。
等号が成立するにはl=m, pl+q = b,pm+q=aのすべてが成立しなければならないがb<aによりそれは不可である。
0734132人目の素数さん
垢版 |
2018/07/03(火) 11:56:14.38ID:F6g7HQZx
y年の大会では
 y≡2 (mod 8) のとき、1次リーグを2位で通過するもベスト16止まり
 y≡-2 (mod 8) のとき、1次リーグで敗退
という経験則がある。これを確率論で説明できるか?
0735132人目の素数さん
垢版 |
2018/07/03(火) 14:23:52.46ID:37f2wROr
>>732
ガウス・ボネの定理を認めるとあっさり解けますね。
https://ja.wikipedia.org/wiki/%E3%82%AC%E3%82%A6%E3%82%B9%E3%83%BB%E3%83%9C%E3%83%8D%E3%81%AE%E5%AE%9A%E7%90%86
Pが測地2角形Mを持つとする。
Mの角∠A,∠Bはいずれも∠A,∠B<π。
z=x^2-y^2上の点(a,b,c)においてx=a,y=bでの切断の曲率が異符号だからガウス曲率Kは負。
∂Mは測地線からなるから∫[∂D]k_g=0。
よって
∫[D]KdA + ∫[∂D]k_g < ∠A + ∠B < 2π。
一方でMは一点とホモトピー同値だから
χ(M) = χ(pt) = 1。
よって
2πχ(M) = 2π。
以上はガウス・ボネの定理
∫[D]KdA + ∫[∂D]k_g = 2πχ(M)
に反する。

………ガウス・ボネの定理勉強せねばww
0736132人目の素数さん
垢版 |
2018/07/04(水) 00:21:08.37ID:QAhoWnUl
>>735
すごい 正解
まさにこの定理を使ってほしかった
0737132人目の素数さん
垢版 |
2018/07/05(木) 02:16:53.46ID:ln/ClMXF
>>714

> 引き分けが1/3というのが妥当かも不明だし、

ワールドカップの892試合では、勝負あり 694、引分け 198 (22.2%) です。

日本代表の関係する21試合では、勝ち 5、負け 11、引分け 5 (23.8%) です。
0739132人目の素数さん
垢版 |
2018/07/05(木) 08:14:29.40ID:ln/ClMXF
xyz座標空間上の曲面Q:z^2 = x^2 - y^2 -1について
 Q上の二点を結ぶQ上の曲線で長さが最短となるものが唯一つでない場合があることを示せ。

 K=0 らしい
0741132人目の素数さん
垢版 |
2018/07/05(木) 15:50:30.03ID:aa26gjJX
>>739
z^2 = x^2 - y^2 -1を整理するとy^2+z^2 = x^2-1
これはx≧1の部分とx≦-1の部分に分かれる二葉双曲面

a>1として
x≧1の部分に2点A(a,√(a^2-1),0),B((a,-√(a^2-1),0)をとる。
Qのz=0による断面に沿った曲線ABの長さをL_1
Qのz=a-xによる断面に沿った曲線ABの長さをL_2とすると、
lim_{a→∞}(L_1/a) = 2√2 = 2.828…
lim_{a→∞}(L_2/a) = √3+(1/√2)log(√2+√3) = 2.5425…

lim_{a→∞}(L_1/a) > lim_{a→∞}(L_2/a)より
あるaが存在してL_1 > L_2となる。
よって、そのようなaについては、L_1はQ上でABを結ぶ曲線の長さの最小値ではなく、
最小となる経路はQのz=0による断面以外の場所を通る。
その最小の経路の1つをCとすると、Cはz=0上にないので、z=0に対してCと対称な曲線をC'とすると
C'はCと異なるもう1つの最小経路となる。


lim_{a→∞}(L_2/a)の計算が少し不安…
0743132人目の素数さん
垢版 |
2018/07/05(木) 16:18:09.19ID:yvviDF5N
>>741
x^2 = y^2+z^2+1のx=aに沿うAB間の距離はπ√(a^2-1)じゃないの?
切り口は半径=√(a^2-1)の円だから。
aで割って極限とってπ>2√2なのでこのルートはすてた。
もっと漸近線のなす角が小さければいけそうなんだけど。
0744132人目の素数さん
垢版 |
2018/07/05(木) 16:21:57.29ID:yvviDF5N
あ、ごめん。x=aでなくてx+z=aか。すこし斜めにとるのね。
なるほど。ならL_2の計算は難しそうww
信じることとしよう!!
0745132人目の素数さん
垢版 |
2018/07/05(木) 16:36:58.34ID:aa26gjJX
lim_{a→∞}(L_1/a)やlim_{a→∞}(L_2/a)の計算は実はそんなに真面目にやらなくても
Qを1/aに縮小した図形はa→∞とすると円錐面y^2+z^2=x^2に近づくので
それで計算しても多分大丈夫。
そうすると、lim_{a→∞}(L_1/a)=2√2は何も計算しなくてもわかるし、
lim_{a→∞}(L_1/a)については
放物線 x=(1+y^2)/2 かつ z=(1-y-2)/2 の -1≦y≦1 の長さ
∫_{-1〜1}√(2y^2+1)dyとして求まる。

そのままやっても出来ない計算ではないがちょっと大変。
0746132人目の素数さん
垢版 |
2018/07/05(木) 16:38:25.37ID:aa26gjJX
>>745
あ、まちがった
  lim_{a→∞}(L_1/a)については
のところは
  lim_{a→∞}(L_2/a)については
に修正
0747132人目の素数さん
垢版 |
2018/07/05(木) 16:41:23.15ID:aa26gjJX
>>744
すこし斜め、というより、断面が放物線になるようにとっているので
そんなに無茶な計算をしているわけではないです。
0748132人目の素数さん
垢版 |
2018/07/05(木) 17:39:19.05ID:izfpNop0
某映画より(全編を観たわけではない)

表裏のある有限枚のカードが横一列に並んでいる。
「表面を向いているカードを選んでひっくり返し、その右隣のカードもひっくり返す」という操作をくり返す。
この操作はいつか終了する(高々有限回しか行えない)ことを示せ。
0749132人目の素数さん
垢版 |
2018/07/05(木) 17:45:54.86ID:sQOol2Jk
一番右のカードに1
右から2番目のカードに2
...
右からn番目のカードに2^(n-1)
というポイントを与える。
表になっているカードの合計を、...以下略
0750132人目の素数さん
垢版 |
2018/07/05(木) 17:59:52.69ID:hOZQWVuc
>>748
カードが1枚のときは自明。
カードがn枚のとき正しいとしてn+1枚のときを考える。
n枚のときに可能な操作の最大回数をNとする。
2N回+1回やっても全裏にならないと仮定する。
一番右カードは2回連続選べないので最初の2N回で一番右のカード以外を選択した回数は少なくともN回。
よってこの時点で裏裏裏…裏裏か裏裏裏…裏裏。
あと一回はできないと駄目だから前者。
しかしその最後の一回で全裏。矛盾。

で桶?
0752132人目の素数さん
垢版 |
2018/07/05(木) 18:42:42.13ID:izfpNop0
映画では>>749の方法を取っていた(2進数で狭義単調減少)
高々2^n-1回の操作(出来るか解らないが全て111…1から1ずつ減少した場合)で成し遂げられる(>>750)

黒板での実演
https://www.youtube.com/watch?v=mYAahN1G8Y8
0753132人目の素数さん
垢版 |
2018/07/05(木) 19:00:01.42ID:hLSyGNAr
右からa枚目にaを与えて表のカードの合計を考えればn枚のカードなら最大n(n+1)/2回で終わる。
最大になるのはすべて表から始めて右が表じゃないカードを選び続けた場合。
0754132人目の素数さん
垢版 |
2018/07/06(金) 01:29:11.26ID:26sRDPd7
長方形のテーブルに同じ大きさのn枚のコインが並べられています。
隙間はありますが重心をテーブル上からはみ出させないようにもう一枚コインを置こうとするといずれかのコインに重なってしまうとします。
さてこのとき、テーブルからすべてのコインを取り除き、改めて4n枚のコインをうまく並べ直せばテーブル全体を覆い尽くせる事を示して下さい。
0755132人目の素数さん
垢版 |
2018/07/06(金) 09:42:24.03ID:KpmZzWMr
>>754
コインの半径をrとする。

もう1枚のコインの重心(中心)をテーブル上のどの点に置こうとしても
他のいずれかのコインと重なるので、テーブル上の任意の地点は、
いずれかのコインの中心から距離2r以内にある。
したがって、今置いてあるコインを全て(中心の位置は変えずに)
半径2rのサイズの円盤に置き換えると、テーブル上の全ての点は
その円盤で被覆される。

テーブルの長方形がn枚の半径2rの円盤で被覆された図を1/2に縮小すると、
テーブルの縦横半分のサイズの長方形がn枚の半径rの円盤で被覆された図となるので、
あらためてテーブルを4分割して、各パーツをその図と同様の配置でn枚ずつのコインで
覆えばよい。
0756132人目の素数さん
垢版 |
2018/07/06(金) 12:37:14.13ID:rNvMJVFD
>>741
これって最小となる経路が少なくとも1つは存在することを証明しないとなんじゃないの?
0759132人目の素数さん
垢版 |
2018/07/06(金) 19:16:43.11ID:jaUkHhY3
あ、計算間違いした。
>>758は逆三角関数使わないと答え出ないですね。
あまり面白くないかも。
0760132人目の素数さん
垢版 |
2018/07/07(土) 00:13:12.37ID:U4/1+k2M
>>758撤回します。どえらい値になる。
参考までに
c:(1+√5)/2
としてA(1,0,c), B(c,-1,0),C(c,1,0)は原点が重心の正20面体のある面の3頂点。
AB,ACを1:2に内分する点をX,YとしてOA,OX,OYと単位球の交点をa,x,yとするとaxyを結ぶ球面三角形△axyは黒い部分の1/100。
∠xay = 2π/5、∠axy = ∠ayx = θとして△axyの面積は2π/5+2θ-π。
あとはθだけど

θ=acos(((sqrt(5)+1)^2/27+(2*((sqrt(5)+1)/2+2))/27)/(sqrt((4*((sqrt(5)+1)/2+2)^2)/81+(4*(sqrt(5)+1)^2)/81)*sqrt((((sqrt(5)+1)*((sqrt(5)+1)/2+2))/6−(sqrt(5)+1)/3)^2+(sqrt(5)+1)^2/36+1/9)))

….orz
0761イナ ◆/7jUdUKiSM
垢版 |
2018/07/07(土) 02:27:38.43ID:0Vd5Kb4Y
>>758
半径1のサッカーボールの表面積は4π・1^2=4π
黒い部分一枚の面積:B
白い部分一枚の面積:W
とおくと、
12B+20W=4π――@
五角形および六角形の一辺をrとすると、
B=r^2・{√(25+10√5)}/4
W=r^2・(3√3)/2
BとWの値を@に代入すると、
3√(25+10√5)r^2+30√3・r^2=4π
r^2=4π/3{√(25+10√5)+10√3}
黒い部分の面積は、
12B=3r^2・{√(25+10√5)}
=4π√(25+10√5)/{√(25+10√5)+10√3}

通分はあるいは必要かと。
0763イナ ◆/7jUdUKiSM
垢版 |
2018/07/07(土) 03:27:26.25ID:0Vd5Kb4Y
>>761
=86.4806266/24.2024177
≒3.57322263

黒い部分の面積には丸みがあって、一辺rの正五角形の面積を求めるやり方はおかしいと感じるが、正六角形にも同様に丸みがあり、表面積4πに対する黒い部分と白い部分の割合は球と三十二面体とでそんな変わらないと思う。
0764132人目の素数さん
垢版 |
2018/07/07(土) 05:58:50.00ID:BXrd5bzu
サッカーボール は多面体か
それとも文字通り球か
題意はどっちよ?
0765132人目の素数さん
垢版 |
2018/07/07(土) 06:51:22.31ID:8oKVVrfK
「そんな変わらない」で数学をやられてもなあ。
いい加減数学には向いてないことに気づいて欲しいものだ。
0766132人目の素数さん
垢版 |
2018/07/07(土) 08:11:54.48ID:ny1i6sPl
多面体の場合の計算ならこのスレのレベルに合わんでしょ?
ただ球面にすると手計算ではリ〜ム〜。
0767132人目の素数さん
垢版 |
2018/07/07(土) 10:32:01.90ID:nRjTFKp9
acos((9-r5)/12).
0768132人目の素数さん
垢版 |
2018/07/07(土) 10:43:39.52ID:VCaMax+U
>>763のトンチンカンぶりを見て、
ずっと昔に某所で球のペーパークラフトを自作しようとしていた
Fラン大学生(本人が紹介ページにそう書いてた)を思い出した。

球を8枚だか16枚だかの同じ形のラグビーボール型のパーツに分解して
それを貼り合わせるという、ごく普通の方式。問題なのは、
そのパーツを自作するときにパーツの算出が全くできてなかったこと。
よく覚えてないが、

「パーツを構成する曲線を厳密に表現しようとしたが、自分の力では難しくて立式できない」

みたいな状況だったはず(この時点で失敗することが確定している)。
0769132人目の素数さん
垢版 |
2018/07/07(土) 10:45:47.54ID:VCaMax+U
結局その人は、曲線の算出にある種の近似を使って、その人なりに
何とか計算しようとしていた。そして、出てきた積分を眺めて

「この積分を計算すると球の大円の周長が出てくるはずなのだが、数値計算すると合わない」

みたいなこと言ってた記憶がある。曲線の算出に近似を使ってる時点で、
大円の周長からはズレるに決まってるのだが、その人は理解していない。
また、そのことを俺が指摘しても本人は全く納得せず、

「ここに厳密な積分があるのに、大円の周長に一致しないのが納得いかない」

みたいな感じだった。
君が出した厳密な積分はデタラメな曲線に対する厳密な積分であって、
もともとの大円の曲線に対する厳密な積分ではないだろっていうね。
0770132人目の素数さん
垢版 |
2018/07/07(土) 10:48:11.23ID:VCaMax+U
で、その近似曲線をもとにしてパーツを自作して貼り合わせたら、
やっぱり球にはならなくて、北極と南極が微妙に尖った、
ラグビーボール型のシロモノになってしまった。本人はそこで

「球になってねーじゃーーーん!」

みたいな愚痴を発して生放送を即座に切っていた。もはやギャグとしか思えない。

学力が低すぎると、自分の意思で選んだ趣味ですら
満足にこなせないんだなって かわいそうになったのを覚えている。
0771132人目の素数さん
垢版 |
2018/07/07(土) 12:58:04.04ID:CT2M6a2y
イナとかいうクソコテも大概だけどグチグチ言ってるやつも相当きめーな

サッカーボールという図形を数学的に厳密に定義してない以上どうとでも解釈出来るだろ
0772132人目の素数さん
垢版 |
2018/07/07(土) 13:01:21.42ID:QlJ5hxgi
>>756
有界閉集合がコンパクトなら完備リーマン多様体
0773132人目の素数さん
垢版 |
2018/07/07(土) 13:56:16.14ID:DOx4W0Fk
f(x)=x^4-2x^2+6とする。
素数pに対し次の条件(※)を考える。

(※) f(x)≡0 (mod p) は整数解を持たない。

p≦xを満たす素数の数をπ(x),その中で(※)を満たすものの数をN(x)とする。
lim[x→∞]N(x)/π(x)を求めよ。
0774132人目の素数さん
垢版 |
2018/07/08(日) 11:54:50.48ID:rpQNxWJy
>>773

f(x) = (xx-1)^2 + 5 だから

(※)  (xx-1)^2 ≡ -5 (mod p) は整数解をもたない。

(1) -5が平方非剰余
または
(2) {1±√(-5)}が平方非剰余

(1) 平方剰余の相互法則(と第1補充法則)から
((-5)/p) = ((-1)/p)・(5/p) = (-1)^((p-1)/2)・(p/5)

 p≡1 (mod 4) かつ p≡±2 (mod 5)
または
 p≡3 (mod 4) かつ p≡±1 (mod 5)
のとき、((-5)/p)=-1 となり、-5 は平方非剰余である。
 p=11,13,17,19,31,37,53,59,71,73,79,97,…

(2) はどうするか

p   x       √(-5)
-----------------------------
p=2  0        1
p=3  0        ±1
p=5  ±1       0
p=7  ±2       ±3     
p=23  ±3,±4    ±8
p=29  なし      ±13
p=41  ±6       ±6
p=43  ±11,±15   ±9
p=47  なし      ±18
p=61  ±9       ±19
p=67  ±11,±22   ±14
p=83  ±5       ±24
p=89  ±44      ±23
p=101  ±37,±42   ±46
p=103  ±24      ±43
0775132人目の素数さん
垢版 |
2018/07/08(日) 20:46:41.23ID:du/lqCAV
考えてくれてる人いるので参考までに実際 mod p での解の個数を数えるプログラム組んでみました。
http://codepad.org/CkKjvyUS
上の方の
#define NPRIMES 3200
#define DEG 4
long coeffs[DEG+1] = {1,0,-2,0,6};
のあたりをいろいろ変えると数値実験できると思います。
この場合の結果は
1998 2 801 0 399

Exited: ExitFailure 10
???Exited: ExitFailure 10???なにこれ?
C言語よく知らないのでよくわかりませんが、一行目の数値はあっています。
計算量多いのでCでないと苦しいのでやってみましたが素人がやるとだめですね。
対処方法ご存知なら教えて下さい。

次数とか係数とか変えてみるとなんか見えてくるかも。
複2次式からなる4次式は大概これに近い比率になるはずです。
それ以外だと解0個の比率はもう少し減ることが多いはずです。
0777132人目の素数さん
垢版 |
2018/07/08(日) 23:31:55.79ID:68ZF08lK
>>774
 7以上の奇素数について

p≡1 (mod 4),p≡±2 (mod 5)  (13,17,37,53,73,97,…)
p≡3 (mod 4),p≡±1 (mod 5)  (11,19,31,59,71,…)
のとき、-5 は平方非剰余

p≡1 (mod 4),p≡±1 (mod 5)  (29,61,89,101,…)
p≡3 (mod 4),p≡±2 (mod 5)  (7,23,67,83,103,…)
のとき、-5 は平方剰余

(1) -5 が平方非剰余となるpの割合は 1/2 に近いかな(?)

(2) {1±√(-5)}が平方非剰余となるpの割合は?

x  π(x)  N(x)
-----------------
2  1   1  
3  2   2
5  3   3
7   4   4
11  5   4
13  6   4
17  7   4
19  8   4
23  9   5
29  10   5
31  11   5
37  12   5
41  13   6
43  14   7
47  15   7
53  16   7
59  17   7
61  18   8
67  19   9
71  20   9
73  21   9
79  22   9
83  23   10
89  24   11
97  25   11   
101  26   12
103  27   13
0778132人目の素数さん
垢版 |
2018/07/09(月) 15:28:48.27ID:9xh3iFPU
2つほど投稿。前者は息抜き程度、後者は自分ではまだ未解決なのでどなたか一緒に考えていただけたら嬉しいです

(1)qを正の奇数とする。この時、nがどんな整数であっても、qと互いに素な整数a,bを適切に定めることで a+b≡n (mod q) を成り立たせることは可能か。

(2)ユークリッド平面R^2の部分集合Aであって、どんな直線との共通部分も二点集合になるようなものは存在するか。
0779132人目の素数さん
垢版 |
2018/07/09(月) 19:07:23.34ID:Al3hwPmB
>>778 の(2)ですが、有限体で同じことはF_2以外不可能であることが以下の通りわかっています:
A⊂(F_q)^2 が条件を満たすとすると |A|=2qでなければならないから、
Aから異なる二点を選ぶ選び方は q(2q-1) 通り。
一方、(F_q)^2 上の直線は q(q+1) 本。
両者には自然な全単射が存在することから q=2 でなければならない。
0780132人目の素数さん
垢版 |
2018/07/10(火) 11:30:55.37ID:8lYR3TJ8
>>763

正20面体の外接球の半径Roは
 Ro = (1/4)√(10+2√5)・(辺長)
   = 0.9510565163・(辺長)

各辺を長さ r:r':r に3分割して、両側を捨てる。(切頂20面体)
正六角形が残るように3等分すると(r=r')
 Ro = 0.9510565163・(2r+r')
   = 2.8531695489 r
このときの外接球の半径Rは
 R = √{(Ro)^2 -r(r+r')}
  = 2.478018659 r

R=1 とおくと r = 0.4035482123

12B = 3√(25+10√5)・rr = 20.6457288 rr ≒ 3.36218088
0781132人目の素数さん
垢版 |
2018/07/10(火) 12:22:30.13ID:8lYR3TJ8
>>780

フラーレン(C_60)分子では
 r = 0.1455 nm(2),0.1458 nm,0.1464 nm
 r' = 0.1384 nm,0.1385 nm,0.1388 nm(1),0.1391 nm(2)
 R = 0.355 nm
らしい。

(1) J.M.Hawkins et al.: Science, 252, p.312-313 (1991)
    "Crystal structure of Osmylated C_60:confirmation of the soccer ball framework"

(2) W.F.David et al.: Nature, 353, p.147-149 (1991)
    "Crystal structure and bonding of ordered C_60"
0782132人目の素数さん
垢版 |
2018/07/10(火) 17:28:53.54ID:9e2HIdsC
昔、何かの記事で読んだんだが、何に載っていたのかが思い出せないし、証明も覚えていない。

「素数の累乗で、n ! + k (n, kは自然数) の形に表わせるものが5つだけだったか存在する」

だれか情報を…
0784132人目の素数さん
垢版 |
2018/07/10(火) 18:33:12.92ID:kWjM72mK
簡単な問題設定の割に難しい問題

長さLの一様な重い棒を、鉛直から角θ傾けて倒す。棒が地面に倒れたときの先端の速さを求めよ。
棒の根本は地面との摩擦によって動かないとする。

地面が滑らかな場合はどうか?
0785132人目の素数さん
垢版 |
2018/07/10(火) 18:41:25.72ID:okqgU0Wa
階乗で検索>階乗 - Wikipedia>ブロカールの問題
0787132人目の素数さん
垢版 |
2018/07/11(水) 00:19:16.07ID:t4/7pAv5
(m / p) を平方剰余記号として奇素数pに対し

(2n+1)^2+3≡0 (mod p)
⇒(-3 / p) = 1
⇒(p / 3) = 1
⇒p ≡ 1 (mod 3)
0789132人目の素数さん
垢版 |
2018/07/11(水) 06:42:33.69ID:P+BTNckt
>>780

球の中心 〜 六角形の中心 の距離(垂線の長さ)
 {(3+√5)/(4√3)}(2r+r') = 0.794654472291766 Ro

球の中心 〜 正五角形の中心 の距離(垂線の長さ)
 Ro - {1/√(φ√5)}r = Ro - 0.5257311121191336 r,

r'/r = (1/2){√(3 + 6/√5) - 1} = 0.6919817084376
のとき、これらは一致し、
内接球の半径 2.03449563343785 r
0790132人目の素数さん
垢版 |
2018/07/11(水) 07:21:29.64ID:P+BTNckt
>>784

・根本が動かないとき
(1/2)Iω^2 = (1/2)MgL(cosθ-cosφ),
I は端点のまわりの慣性モーメントで、I = (1/3)ML^2,
v_S = ωL,
v_S(90゚) = √(3gLcosθ),

・地面が滑らかな場合
(1/2)I’ω^2 + (1/2)M(v_G)^2 = (1/2)MgL(cosθ-cosφ),
I’は中心のまわりの慣性モーメントで、I’= (1/12)ML^2
φ=90゚のとき、v_S = 2v_G = ωL,
v_S(90゚) = √(3gLcosθ),
0791132人目の素数さん
垢版 |
2018/07/11(水) 08:00:21.98ID:P+BTNckt
>>780 >>789

r'/r = (1/2){√(3+6/√5) -1} = 0.6919817084376 のとき

内接球の半径   0.794654472291766 Ro
外接球の半径 R = 0.861318645 Ro
比   1.0838907666
0792132人目の素数さん
垢版 |
2018/07/11(水) 13:27:42.24ID:LEvJsMim
>>773
ヒントです。
というかこれ知らないと多分自力では解けません。
逆にこれ知ってたらあとはチョロチョロ工夫するだけです。
ほんとは以下の定理をさらに発展させた定理もあってそれを使うと一撃で解けるんですが、いい線いってる方針があがっててその方針で進めるなら以下の定理を使うのが筋だと思います。
よかったら挑戦してみて下さい。
-----
Kを代数体、Rをその整数環、vを素イデアル、v∩Z=pZとする。
L/KをGalois拡大、Sをその整数環、wをv=w∩Rを満たすSの素イデアル、F∈Gal((S/w)/(R/v))とする。
このときσ∈Gal(L/K)でσ(v) = vでありσの誘導するGal((S/w)/(R/v))の元がFに一致するものが存在する。
すなわち任意のx∈Sにたいして
σ(x) + w = F(x+w)
を満たすものが存在する。
またL/Kが不分岐のときこのσは唯一存在する。
またw'をv=w'∩Rを満たす他のSの素イデアルとし、同様のσ'を構成するときσとσ'は共役である。
すなわちこの条件をみたすσ∈Gal(L/K)の共役類はvにより一意に定まる。
この共役類をvのFrobenius共役類とよぶ。
-----
Gを有限群とするときGのC値表現ρ:G→GL(n,C)によってtr・ρ:G→Cとかける関数を指標と呼ぶ。
G上の関数fが類関数であるとは同じ共役類に属する元について常に等しい値をとる関数とする。
任意の類関数は指標の線形結合として一意にかける。
----
Kを代数体、Oをその整数環、L/Kを有限次Galois拡大とする。
vをL/Kで分岐しない素イデアル、p(v)をv∩Z=p(v)Zを満たす素数、Fr(v)をvのFrobenius共役類、x0を自明指標とする。
f:Gal(L/K)→Cを任意の類関数として、これを
f = Σ[x]c_x x
と分解するとき次が成立する。

lim[x→∞]Σ[p(v)≦x] f(Fr(v))/π(x) = c_x0

とくにfが自明でない指標のときは左辺は0となる。
0794132人目の素数さん
垢版 |
2018/07/11(水) 13:58:00.78ID:VOQaSRny
イメージしやすいように例を
K = Q、L=K(i)のときR=Z、S=Z[i]、Gal(L/K)={id,σ}
である。(ただしσは複素共役をとる写像でσ(i) = -i。)
――
v=3Rのとき
w=3Sとなる。
このとき
F(i + w) = i^3 + w = -i + w。
故にこのときはFr(v) = σ。

v=5Rのとき
w=(2+i)Sもしくは(2-i)Sのいずれか。
いずれにせよ、このとき
F(i + w) = i^5 + w = i + w。
故にこのときはFr(v) = id。
――
この例では Fr(v) = id ⇔ p(v)≡1 (mod 4) となります。(L/Qがアーベル拡大ならこのようにp(v)のmod ××の類で定まります。)
0795132人目の素数さん
垢版 |
2018/07/11(水) 19:42:36.04ID:zWguNjBa
>>778
(2) は、

ttps://cybozushiki.cybozu.co.jp/articles/m000434.html

の中で全く同じ問題についての記述がある(解答そのものが載っているわけではない)。

記事によると、超限帰納法によって構成的に描けるとあるが、実際には
選択公理&超限帰納法 あるいは 選択公理&超限再帰 のテクニックのことを
指していると思われる。

こちらでやってみたところ、確かに構成できたが、濃度・整列集合に関する
マニアックな知識が必要な上に、きちんと書くと面倒くさい。
しかも、超限再帰の知識が無い人には証明が理解できない。
超限再帰のかわりにツォルンの補題で書き直した証明も出来たが、
結局は濃度・整列集合に関するマニアックな知識が必要で
やることがほとんど同じで面倒くさかった。

選択公理を使わずに構成できるかは知らない。
0796132人目の素数さん
垢版 |
2018/07/11(水) 23:13:11.73ID:t4/7pAv5
なるほど、超限帰納法使うとできるね。
まぁマニアックかな?
全ての直線を連続体濃度の基数でラベルしといてあるラベル番目の直線の番が回ってきたときその直線より前の直線は高々連続体濃度未満しか無い事を利用するのね。
なるほど。
言われたらわかるんだけどなぁ。
0798778
垢版 |
2018/07/12(木) 02:22:48.22ID:/Z2aWdzi
>>795 >>796
ありがとうごさいます。やはり選択公理が必要になりそうなんですね…
まだ自分では示せていないので、>>796をヒントにして考えてみようと思います。

有理数体のような可算無限な体で同様のことができるかどうかも気になっているのですが、同じ手法で示せるのでしょうか?
0802132人目の素数さん
垢版 |
2018/07/12(木) 16:26:03.85ID:sQqagqbK
>>792
森田先生の東大出版の整数論とか
加藤先生の岩波出版の整数論Iとかには載ってると思う。
ただしどっちも証明完全にはのってなかった希ガス。
確実に証明まで含めてのってるのは
Lang の Algebraic number theory。
池原 Winner Landau の定理を使う証明で若干妙な証明だけどのってます。
まぁ森田先生のはLangの訳本に近い。
間違ってるとこもそのままちゃんと間違ってますwww
0805132人目の素数さん
垢版 |
2018/07/12(木) 20:33:04.82ID:sQqagqbK
てか
n!(1/(n-2)! + 1/(n-3)! + ... + 1/1! + 1)
=n!(1/0! + 1/1! + ... + 1/(n-2)!)
こんなんもたまらん希ガス。
0809132人目の素数さん
垢版 |
2018/07/12(木) 22:12:02.43ID:Os9QSTcU
>>803 >>807
分母でやってみた
http://codepad.org/itx6rWET
6 % 7
612 % 325
453800 % 155001
3861634830 % 976314031
481961256261492 % 96969788815873
1054761729394054912664 % 176420776601977522329
9379220392541459116676859552 % 1342977541299460819153297325
6871627232977971685604791983162107670 % 860310167933842952793421619070619213
なんのルールも見えん。
ほんまに解けんのこれ????
0810132人目の素数さん
垢版 |
2018/07/12(木) 22:15:25.17ID:FNY0485u
>>803
ガウス記号使えるなら [e・n!]-n-1 という表示は可能だけど、もしかしてこれが答え?
0813778
垢版 |
2018/07/13(金) 00:58:20.39ID:J3lC7G1w
>>778 (2)ですが自己解決しました。ヒントくださった方ありがとうございました。
0814132人目の素数さん
垢版 |
2018/07/13(金) 12:00:38.73ID:btBhB1qs
>>803
元ネタは、たまたま書庫で見た数学セミナーの連載記事 「算私語録」 で、答えは書いてなかったのだ。
続けたまえ!
0815132人目の素数さん
垢版 |
2018/07/13(金) 14:19:55.91ID:j1khqgOs
ハゲのくせになまいきだぞ
0816132人目の素数さん
垢版 |
2018/07/14(土) 00:32:30.12ID:kfXPO9Dw
半径1のn次元球D^nの体積はπ^[n/2]/(n/2)!
ただし半整数の階乗は1ずつ減らして1/2までの積
これを帰納法使わず証明して欲しい
0817132人目の素数さん
垢版 |
2018/07/14(土) 02:32:04.17ID:5VRLgysv
>>816
半径rのn次元球の体積を Vn*r^nとすると、
n次元球の表面積は、n*Vn*r^(n-1) となる事を利用して、
次の積分は、Vnを使って、下のように書くことができる。

I_n=∫・・・∫exp[-(x1^2+x2^2+...+xn^2)]dx1・・・dxn ; n次元空間全体での積分
=∫[0,∞]exp[-r^2] n*Vn*r^(n-1)dr
=(n/2)*Vn*∫[0,∞]exp[-y] y^((n/2)-1)dy
=(n/2)*Vn*Γ(n/2)

一方、I_n=(I_1)^n={∫[-∞,∞]exp(-x^2)dx}^n=(√π)^n なので 
Vn=π^(n/2)/Γ(n/2+1)
あとはnの偶奇で分け、独自の階乗記号を使って書き下せば完了
0818132人目の素数さん
垢版 |
2018/07/14(土) 03:43:23.92ID:+LT1qx/t
>>817
この証明のdrって面積素、つまりはハウスドルフ測度のことだよね?だとしたらハウスドルフ測度の定義にはn次元球の体積が必要だから循環論法になるんじゃないの?
0819132人目の素数さん
垢版 |
2018/07/14(土) 03:45:07.12ID:+LT1qx/t
>>818
ごめんdrは半径か失礼しました
その前段階で表面積分してるよね?
0820132人目の素数さん
垢版 |
2018/07/14(土) 05:21:45.18ID:kfXPO9Dw
>>817
漸化式も使わないではできない?
0821132人目の素数さん
垢版 |
2018/07/14(土) 05:23:31.56ID:kfXPO9Dw
>>817
使ってないか失礼
0822132人目の素数さん
垢版 |
2018/07/14(土) 09:09:04.02ID:MrcE29He
△ABCはAB=3,BC=4,CA=5を満たすとする。
△ABCの外接円をO、内接円をIとする。
異なる3点P,Q,RがO上をPQ,PRがIと接するようにうごくとき、直線QRが通過しない部分の面積を求めよ。
0823132人目の素数さん
垢版 |
2018/07/14(土) 14:38:08.75ID:2fMgdkQ3
>>773
5/8 と出た。自信は無い。

概略を書くと、
α=√(1+√(-5)), β=√(1-√(-5)) とおく。
f(x) の Q 上の最小分解体は L:=Q(α,β)
Gal(L/Q) は位数 8 の二面体群 D_8 に同型。
σ, τ∈Gal(L/Q) をそれぞれ
 σ(α)=β,σ(β)=-α
 τ(α)=β,τ(β)=α
を満たすものとする。

D_8 の既約表現は 5 つ。それらを ρ_0,...,ρ_4 とする。ただし ρ_0 は自明な表現。他は省略。
それぞれの表現に対応する指標を x_0,...,x_4 とおく。

有理素数 p に対し、f(x) を mod p で既約多項式に分解すると
(1) 4 つの 1 次式
(2) 2 つの 1 次式と 1 つの 2 次式
(3) 2 つの 2 次式
(4) 1 つの 4 次式
の 4 通りが考えられる。(式の形から (1次式)*(3次式) はあり得ない)

それぞれに対応する Frobenius 共役類は
(1) {id} (2) {στ,σ^3τ} (3) {σ^2} または {τ,σ^2τ} (4) {σ,σ^3}
整数解を持つのは (1),(2) のとき。
よって、類関数 f を
 f(ζ)=0 (ζ=id,στ,σ^3τ)
 f(ζ)=1 (otherwise)
で定めれば、求める値は
 lim[x→∞]Σ[p≦x] f(Fr(pZ))/π(x)
に一致する。なお、分岐する pZ は高々有限個なので無視できる。多分。

f を x_0,...,x_4 で表すと
 f=(5x_0+x_1+x_2-3x_3-2x_4)/8
が得られたので、>>792の定理より、求める値は 5/8
0824イナ ◆/7jUdUKiSM
垢版 |
2018/07/14(土) 15:44:20.32ID:+kVDeoWP
>>822
△ABCの外接円Oの半径:5/2
△ABCの内接円Iの半径:1
Pが円Oの周上を一回動くときQRは円Iに接しながら円Oの中かつ円Iの外の領域をくまなく動くので、QRが通らない部分は円Iの中と円Oの外である。
(円Iの面積)=π
(円Oの面積)=25π/4
(QRが通る部分の面積)=(円Oの面積)-(円Iの面積)
21π/4

(QRが通らない部分の面積)=∞+π
→∞
0825132人目の素数さん
垢版 |
2018/07/14(土) 16:30:00.48ID:vTy8qTeq
>>824
ほぼ正解。線分PRではなく直線PRね。
通らない部分は円Iの内部です。
直線QRがIに接して動く事に気付けば2秒で解ける問題でした。
0826132人目の素数さん
垢版 |
2018/07/14(土) 16:47:55.75ID:hjCo+mDv
難問です.
一般に、環A上の写像φ:A→Aが加法群の準同型であり、Leibniz rule(i.e.,φ(xy)=φ(x)y+xφ(y))を満たす時、φをA上の導分(微分)と云う.
今、A=ℝ[x] (実数体上の一変数多項式環)とする. 此の時、A上の導分φで、φ(ℝ)≠{0}なるものは存在するか?
0828132人目の素数さん
垢版 |
2018/07/14(土) 18:46:36.87ID:hjCo+mDv
ヒントとしては、先ずℝ上の微分を考えて其れを応用します.
0829132人目の素数さん
垢版 |
2018/07/14(土) 18:47:17.58ID:hjCo+mDv
ℚの代数閉包までなら自明だから
0830132人目の素数さん
垢版 |
2018/07/14(土) 19:07:11.95ID:hjCo+mDv
>>827
それはそうw
しかしφは加法群の準同型というだけで環準同型ともℝ-加群の準同型とも限らないのでそれだけでは何も言えないw
0831132人目の素数さん
垢版 |
2018/07/14(土) 21:23:25.61ID:SCz7cUJu
Kを微分体とします(標数0)
K(x)をその純超越拡大とします
a∈K(x)を固定します

Kの導分DがK(x)上の導分でD(x)=aを満たすように一意拡張されますよね?
0832132人目の素数さん
垢版 |
2018/07/14(土) 21:46:51.85ID:vPo4n2qv
>>830
ああ、R射であることは要求されてないのね。
一般論はよくしらないけど以下の議論でGrand Field Kは標数0として

― 補題 ―
L/M/Kが拡大体、Mは超越拡大M = K(α)、φ:K→Lが導分のとき任意のx∈Lにたいしてφはφ(α) = xをみたす導分:M→Lに拡張される。

(∵) Σp_iα^i ∈ K[α]に対しては、φ(Σp_iα^i) = Σ( φ(p_i) α^i + i p_i α^(i-1) x ) 定め、p(α),q(α)∈ K[α]に対しては
φ(p(α)/q(α)) = (φ(p(α))q(α) - p(α)φ(q(α)/q(α))) / q(α)^2と定めればよい。
以下煩雑であるが初頭的ゆえ略。

― 補題 ―
L/M/Kが拡大体、Mは単項代数拡大M = K(α)、φ:K→Lが導分のとき任意のx∈Lにたいしてφはφ(α) = xをみたす導分:M→Lに拡張される。

(∵) F(x_0,x_1,…,x_n) ∈ K(x_0,x_1,…,x_n)とβ_1,…,β_nをF(x,β_1,…,β_n)がαの最小多項式となるようにとる。∂F/∂x_i = Fiとして
φ(α) = - ΣF_i(α,β_1,…,β_n)φ(β_i)/F_0(α,β_1,…,β_n)と定めれば良い.
以下煩雑であるが初頭的ゆえ略。

で結局
― 補題 ―
任意の導分 K→L は L→L に拡張される。

からQ[x] → Q[x] ⊂ R ⊂ R(x)の導分φをφ(x)≠0となるように定めておいてからR(x)まで拡張すればよい。

煩雑な計算を回避する方法がありそうでなさそうで………
0833132人目の素数さん
垢版 |
2018/07/14(土) 22:14:40.55ID:vPo4n2qv
>>823
すばらしい!正解!(ホントいうと f の展開のとこチェックしてませんが信じます。)
まさに期待通りの解答です!!
ちなみに>>792のヒントは>>774さんや>>777さんのカキコをみて後付けで思いついて作ったものです。
後日こちらが用意した解答もあげようと思いますけど……いや、すごい!!!
0834132人目の素数さん
垢版 |
2018/07/14(土) 22:28:26.61ID:SCz7cUJu
>>828
わざわざAを設定したからにはそれ使って示すことを想定してるんかな
ちょい気になるから書いて
0836132人目の素数さん
垢版 |
2018/07/15(日) 02:16:45.42ID:8ME/vsb7
今日のことわざ
Ground Field にチャンスは落ちてない。
チャンスは Pitch に落ちている。それを全力で探そう。
0837132人目の素数さん
垢版 |
2018/07/15(日) 09:14:23.66ID:DcTFeZo6
>>833
お、あってたか。良かった。
>>777の結果が 5/8 にあまり近くなかったのが不安で……
こちらも勉強になった。
ちょうど自分の知識の少し先って感じだったんで楽しかった。

答えを出してから気づいたんだけど、直感的に考えて
 -5 が平方非剰余…1/2 の割合
 -5 が平方剰余かつ 1+√(-5), 1-√(-5) が共に平方非剰余…1/8 の割合
で、合わせて 5/8 っていう結果と一致するのね。

f の展開をチェックをしてないってことは、元々の解答では f の展開を使わないってことですかね。
楽しみです。
0838132人目の素数さん
垢版 |
2018/07/15(日) 23:39:30.12ID:0Uh0l9mr
>>837
もうすでに解答が出てるのであれなんですが参考までに用意していた解答を紹介します。
一般にGal(L/K)の部分集合Sとその特性関数(すなわちf(x) = 1 iff x∈S,f(x) = 0 otherwise)についてf(x)が類関数であるものを取ります。

d = lim[x→∞] #{ p ≦x | Fr(p) ∈ S}

を計算したい、もちろんそれは>>792をみとめれば

f = Σ[x]c_x x

と展開するときのc_x0です。
ここで有限群の表現論から一般の類関数 f について得られる結果

c_x = (f,x) = 1/#G Σ[σ∈G] f(σ)x^(σ)  (x^ はxの複素共役による指標)

(http://gauss.ms.u-tokyo.ac.jp/lecture/algebra3/representation-theory.pdf)
を用いれば今のfについては

c_x0 = (f,x0) = 1/#G Σ[σ∈G] f(σ)x0^(σ) = #S/#G

が得られます。結局この設定のもとにおいては

----定理( チェボタレフの密度定理)----
lim[x→∞] #{ p ≦x | Fr(p) ∈ S} = #S/#G
https://en.wikipedia.org/wiki/Chebotarev%27s_density_theorem

が得られます。
本文においてはGal(L/K) = D4、(4つの解への作用は4次2面体の4頂点に対するそれと同じ)
Z/pZで解がない⇔Fr(p) ∈ {(1234),(4321),(13)(24),(12)(34),(14)(23)} (= Sとおく)
なので結局

lim[x→∞] #{ p ≦x | Fr(p) ∈ S} = #S/#D4 = 5/8

となります。
別の例f(x) = x^4 - 2x+6の場合にするとガロア群はS_4で
解0個⇔Fr(p) ∈ {(1234)...} ← 9個
解1個⇔Fr(p) ∈ {(123)...} ← 8個
解2個⇔Fr(p) ∈ {(12)...}  ← 6個
解4個⇔Fr(p) ∈ {e}     ← 1個
なのでmod pで解を0個、1個、2個、4個もつ比率は

9:8:6:1

となります。
http://codepad.org/c9fnjakx
0839132人目の素数さん
垢版 |
2018/07/15(日) 23:43:37.83ID:J7bEvDWH
>>816
半径rのn次元球の体積を Vn*r^nとすると、
n次元球の表面積は、n*Vn*r^(n-1) となる事を利用して、
次の積分は、Vnを使って、下のように書くことができる。

I_n=∫・・・∫exp[-(x1^2+x2^2+...+xn^2)]dx1・・・dxn ; n次元空間全体での積分
=∫[0,∞]exp[-r^2] n*Vn*r^(n-1)dr
=(n/2)*Vn*∫[0,∞]exp[-y] y^((n/2)-1)dy
=(n/2)*Vn*Γ(n/2)

一方、I_n=(I_1)^n={∫[-∞,∞]exp(-x^2)dx}^n=(√π)^n なので 
Vn=π^(n/2)/Γ(n/2+1)
あとはnの偶奇で分け、独自の階乗記号を使って書き下せば完了
0841132人目の素数さん
垢版 |
2018/07/17(火) 07:36:12.46ID:Aegngsr/
>>839
Γ(n/2+1)の値を求めるのに漸化式使うんでなくて?
0844132人目の素数さん
垢版 |
2018/07/17(火) 22:02:28.41ID:qLE42k1Y
Peter Winklerのパズル本より
――
3つの非負整数の組(a,b,c)に対して次の操作を考える。

(※) (a,b,c)から2つの数 (x,y) (x≧y)を選び、その2数を(x-y,2y)に置き換える。

この操作を何回か繰り返すことにより3数のいずれかを0にできることを示せ。
――
0845132人目の素数さん
垢版 |
2018/07/18(水) 15:15:11.43ID:6M2SJbed
>>844
任意の自然数nは、正の奇数aと非負整数bを用いてn=a*2^bの形でただ1通りに表せる。
このとき自然数nについての関数fをf(n)=bと定義する。
また、問題で与えられた操作を以下操作Xと呼ぶ。

非負整数の組(a,b,c)に対し操作Xを施してもa+b+cの値は不変であるので、
その値をSとおく。

証明の基本方針:まず以下の命題1,2を示す

命題1
a,b,cがいずれも0でないとき、
min(f(a),f(b),f(c))<f(S)ならば、
操作Xを有限回数繰り返したものを(A,B,C)として
A,B,Cのいずれかを0とするか、min(f(A),f(B),f(C))=f(S)とすることができる。

命題2
a,b,cがいずれも0でなく、
f(a),f(b),f(c)を小さい方から順に並べたものをm,n,lとしてm=f(S)のとき、
操作Xを有限回数繰り返したものを(A,B,C)として
A,B,Cのいずれかを0とするか、
f(A),f(B),f(C)を小さい方から順に並べたものをM,N,LとしてM=f(S)かつN>nとすることができる。

ここで、S≠0のとき、2^k≦S<2^(k+1)を満たす非負整数kを考えて、
ある(a,b,c)に対し操作Xを繰り返して3数のいずれかを0にすることができないと仮定すると、
命題1,2より,(a,b,c)に対して操作Xを有限回数繰り返したものを(A,B,C)とし、
f(A),f(B),f(C)を小さい方から順に並べたものをM,N,LとしてN>kとすることができる。
このときA,B,Cのうちの1つxが x>Sを満たすこととなり、矛盾。
よって、仮定は誤りであり、
ある(a,b,c)に対し操作Xを繰り返して3数のいずれかを0にすることができる。

以下、命題1,命題2を証明する。
0846132人目の素数さん
垢版 |
2018/07/18(水) 15:56:29.33ID:6M2SJbed
>>844
>>845の続き
命題1の証明

以下、a,b,cはいずれも0でなく、(a,b,c)に操作Xを有限回数繰り返しても0は
出現しないものとする。

f(S)=sとおく。
(x,y,z)は(a,b,c)を並べ替えたもので、f(x)≦f(y)≦f(z)、(m,n,l)=(f(x),f(y),f(z))とすると
p,q,rを奇数として
S=p*2^m + q*2^n + r*2^l = (p+q*2^(n-m)+r*2^(l-m))*2^m
ここで、m<sのとき
m<nとすると、p+q*2^(n-m)+r*2^(l-m)は奇数なのでf(S)=mとなり、矛盾
m=n=lとしても、やはりp+q*2^(n-m)+r*2^(l-m)は奇数なので矛盾。
よって、m<sならば必ずm=n<lとなる。
このとき、x,yを大きい方からx',y'とおくと、
f(x'-y')≧m+1,f(2y')=m+1となるので、
(a,b,c)を1回の操作Xにより(x'-y',2y',z)を並べ替えたものにすることができて、
このときmin(f(x'-y'),f(2y'),f(z))=m+1
このような操作を繰り返すことで、
min(f(a),f(b),f(c))<sのとき、操作Xをs-min(f(a),f(b),f(c))回繰り返すことで、
操作後の組(A,B,C)についてmin(f(A),f(B),f(C))=sとすることができる。

以上より、操作の途中で0が出現するケースも含め、命題1が示された。
0847132人目の素数さん
垢版 |
2018/07/18(水) 17:17:13.64ID:6M2SJbed
>>844
>>845>>846の続き
補題3
非負整数の非順序対{a,b}について、操作YをY:{a,b}→{|a-b|,2*min(a,b)}とする。
いま、a,bがいずれも0でなく、M=min(f(a),f(b)),N=max(f(a),f(b))として
N≧M+2とすると、
{a,b}に操作Yを有限回施したものを{A,B}として、
min(f(A),f(B))=Mかつmax(f(A),f(B))=N-1とすることができる。

証明:Y:{a,b}→{A,B}として、
a,bがともに0でなく、f(a)≠f(b)、min(f(a),f(b))=mのとき、
A,Bもともに0でなく、f(A)≠f(B)、min(f(A),f(B))=mとなることは容易に示される(略)。
よって、a,bがともに0でなく、f(a)≠f(b)、min(f(a),f(b))=mとなるような{a,b}から始めて、
操作Yにより、同じ条件を満たす非負整数の非順序対の列を無限に続けることができる。
一方、操作Yは2つの非負整数の和を変えないので、そのような非負整数の非順序対は
有限個しか存在しない。したがって、その列は必ず循環する。
ここで、a,bがともに0でなく、f(a)≠f(b)であるような非負整数の非順序対について、
操作Zを、以下のように定義する。
  a,bを並べ替えたものをx,yとし、f(x)<f(y)とするとき、Z:{a,b}→{y/2,y/2+x}
このとき、
f(y)≧f(x)+2のとき、f(y/2)=f(y)-1,f(y/2+x)=f(x)より、f(y/2)>f(y/2+x)=f(x)であり、
f(y)=f(x)+1のとき、f(y/2)=f(x),f(y/2+x)≧f(x)+1より、f(y/2+x)>f(y/2)=f(x)となるので、
明らかに操作Zは、a,bがともに0でなく、f(a)≠f(b)であるような非負整数の非順序対
の中で閉じた操作Yの逆操作となる。
逆操作が存在するため、前述の{a,b}から始まる無限列はその途中のどこから見ても
逆順に辿って出発点まで遡ることができ、循環節には必ず{a,b}が含まれる。

ここで、補題の設定の{a,b}について、Z:{a,b}→{A,B}とすると、{A,B}は{a,b}から始まる
操作Yによる無限列の循環節に含まれることになるので、{a,b}から有限回の操作Yで
{A,B}にすることができる。このとき,明らかにmin(f(A),f(B))=Mかつmax(f(A),f(B))=N-1を
満たすので、補題3は成立する。
0848132人目の素数さん
垢版 |
2018/07/18(水) 17:44:07.41ID:6M2SJbed
>>844
>>845>>846>>847の続き
命題2の証明

f(S)=sとおく。
(x,y,z)は(a,b,c)を並べ替えたもので、f(x)≦f(y)≦f(z)、(m,n,l)=(f(x),f(y),f(z))とする。
m=sのとき、s=m<n≦lまたはs=m=n=lのいずれかとなる。

s=m<n=lまたはs=m=n=lのとき、
(a,b,c)から1回の操作Xで(x,|y-z|,2*min(y,z))とすることができ、
このとき、|y-z|=0となるか、
f(x)=m=s,f(|y-z|)≧n+1>n,2*min(y,z)=n+1>nとなる。

s=m<n<lのとき、補題3より
{x,z}に対して操作Yを有限回繰り返すことで
f(x')=f(x)=m,f(z')=f(z)-1=l-1とすることができる、
すなわち、(a,b,c)に対して操作Xを有限回繰り返すことで
f(x')=m,f(y)=n,f(z')=l-1となる(x',y,z')を並べ替えたものにすることができる。
さらに、それを繰り返すことで、
f(x'')=m,f(y)=f(z'')=nとなる(x'',y,z'')を並べ替えたものにすることができる。
そこからさらに1回の操作で(x'',|y-z''|,2*min(y,z''))とすることができ、
このとき、|y-z''|=0となるか、
f(x'')=m=s,f(|y-z''|)≧n+1>n,2*min(y,z'')=n+1>nとなる。

以上より、命題2は示された。
0849132人目の素数さん
垢版 |
2018/07/18(水) 22:16:47.18ID:6M2SJbed
手順としては
(1) まずmin(f(a),f(b),f(c))=sとなるまで、fの値の小さい2つを対象に操作Xを行う
(2) min(f(a),f(b),f(c))=sとなったら、fの値の大きい方から2つが一致してない場合は
  fの値の一番大きいものと一番小さいものを対象に
  fの値の大きい方から2つが一致するまで操作Xを繰り返す。
(3) fの値の大きい方から2つが一致していたら、
  その2つを対象に操作Xを行う。この結果fの値の上から2番目が1増える。
(4) (2)(3)を繰り返す

この流れのどこかで、必ずa,b,cのいずれかが0になる。

たとえば、
(a,b,c)=(13,42,69)からスタートする。このとき
[f(a),f(b),f(c)]=[0,1,0]であり、
S=a+b+c=124、s=f(S)=2

手順(1)
 (13,42,69)[0,1,0]
→(26,42,56)[1,1,3]
→(52,16,56)[2,4,3]

手順(2)
→(36,32,56)[2,5,3]
→(4,64,56)[2,6,3]
→(8,60,56)[3,2,3]

手順(3)
→(16,60,48)[4,2,4]

手順(3)
→(32,60,32)[5,2,5]

手順(3)
→(64,60,0)[6,2,-]
0850132人目の素数さん
垢版 |
2018/07/18(水) 23:06:36.41ID:s/ikFyeC
パズル本だからもっとシンプルな証明があるんじゃないの?
0852132人目の素数さん
垢版 |
2018/07/18(水) 23:58:33.34ID:Tmnw4mMS
>>843やってください お願いします
面白い問題スレなので面白く思えるように問題を言うと、
半径1のリングを空間中に配置する
一辺の長さが十分に長い大立方体のある頂点Xを通る3辺がつねにリングに接するように立方体をゴリゴリ動かす このときXの通過する面とリング面により囲まれる立体の体積を求めよ
0853132人目の素数さん
垢版 |
2018/07/19(木) 01:32:19.16ID:jz3wZoKD
>>844 です。>>845 さん。お見事!正解です。もちろん本に載ってる証明はもっと洗練されてますが解答用意する時間とか全然ちがいますからねぇ。
何より人の作ったエレ解より自力の解答です。
以下は本の解答です。
----
補題 (>>847の補題3に相当)
(a,b,c)がa:奇数、b:偶数のとき何回か操作をして(a+b/2,b/2,c)にできる。
(∵) a,bに対して操作を行い得られる列を(a1,b1),(a2,b2),…とする。
ただし(a1,b1)=(a,b)。
ai+biが不変で正の整数だから(ai,bi)=(aj,bj) (i<j)となるi,jがとれる。
iが最小となるものをとる。
i>1とすると操作によって(a(i-1),b(i-1)) → (ai,bi)、(a(j-1),b(j-1)) → (aj,bj)となったこととai+biが奇数であることと、(ai,bi) = (aj,bj)により(a(i-1),b(i-1)) = (a(j-1),b(j-1))となる。
これはiの最小性にはんするからi=1である。
よって(aj,bj)→(a1,b1)と変化したこおtになるが、このとき(aj,bj) = (a1+b1/2,b1/2)である。□
----
これを用いて示します。
----
(a,b,c)からいかなる操作によっても0が作れないとする。
奇数が2つ以上あればそれに対して操作を行い奇数は1個以下としてよい。
全部偶数ならすべてを2でわってよい。
この作業を繰り返してa:奇数、b,c:偶数としてよい。
操作を繰り返して発生する最大の奇数をMとする。
a=Mとしてよい。
b,cがともに4の倍数でないならb,cに対して操作を行えば4の倍数となるので、bは4の倍数としてよい。
以上の設定ののち補題を適用すれば操作によって(a+b/2,b/2,c)が得られるが、仮定によりa+b/2は奇数。
これはMの最大性に矛盾。□
0856132人目の素数さん
垢版 |
2018/07/19(木) 06:52:48.44ID:uK3KKpwz
円Cに内接する四角形PQRSにおいてP,Q,R,SにおけるCの接線を結んで得られる四角形は円に内接している。
このとき四角形PQRSの4辺の中点は同一円周上にあることを示せ。
0858132人目の素数さん
垢版 |
2018/07/19(木) 10:21:43.92ID:zkAGwq9m
実数解は
z=−sqrt(5)−1/2,z=(sqrt(5)+1)/2,z=2,z=0
みたいだけどこれしかないのかな?
0859132人目の素数さん
垢版 |
2018/07/19(木) 11:28:54.79ID:QaNCXAoL
この4つしかないっぽいんだけどなぁ。
z≠0としてz'を複素共役として
Z^3-3zz'+z’^2/z+2=0
z=r cisθ としてrを固定してθを動かす。
r>>0 では原点周りを、大きく3回周り、r→+0で2に収束していくからその過程で原点を通る回数は高々3回っぽい。
うーん、しかし厳密には示めせてないなぁ?
輪っかが戻ってくる可能性あるしなぁ?
改めて因数定理の偉大さを感じる。
0860132人目の素数さん
垢版 |
2018/07/19(木) 13:11:57.68ID:TH9tCNDf
>>857
zの共役をz'とする。

zが実数のときは、z=z'なので、z^4-3z^3+z^2+2z=0、z(z-2)(z^2-z-1)=0より
z=0,2,(1±√5)/2
>>858はちょっと違う)

zが虚数のときは、与方程式と、それの両辺の共役をとったものを(1)(2)として
(1)+(2)と(1)-(2)の2つの方程式を作る。
(1)-(2)はz-z'でくくれるが、z≠z'なので割って良い。
その結果、2つの左辺が対称式の方程式が得られるので、
z+z'=a,zz'=bとして、a,bについての連立方程式を作り、
それを実数の範囲で解けばよい。
(解けるかどうかはまだやってない)
0861132人目の素数さん
垢版 |
2018/07/19(木) 14:04:13.48ID:TH9tCNDf
>>860
aについての6次方程式が出てきた。
途中計算に全く自信はないが、
4a^6+12a^5-5a^4-48a^3-24a^2+20a+8=0
とかになったので
Wolfram先生に訊いてみたら実数解は4つ。
それぞれの実数解に対応するa,bのペアからは
それぞれzの虚数解(共役ペア)が得られるようなので、
実数解4つと虚数解8つ?
0862イナ ◆/7jUdUKiSM
垢版 |
2018/07/19(木) 14:21:00.04ID:4JvpUq38
半径1の円を底面とする半球の表面積は、
π+4π/2=3π

おもしろいですね。

ちょうど球面が円盤の二倍の面積。
0863イナ ◆/7jUdUKiSM
垢版 |
2018/07/19(木) 15:08:49.11ID:4JvpUq38
三点でしたね。訂正です。前>>862


>>852
半径1の円周上のある点から物体の頂点までの斜辺は、物体が半径1の円の中心の真上に来たとき、
√3/√2
半径1の円の中心からの頂点の高さは、三平方の定理より、
√{(3/2)-1}=1/√2
頂点の通過部分と半径1の円盤とで囲まれる部分の体積は、
(4π/3)(1/√2)(1/2)
=(π√2)/3
0864132人目の素数さん
垢版 |
2018/07/19(木) 16:38:03.49ID:nPcIIs+O
>>857-861

実根(2つ)
z = (1±√5)/2 = φ,-1/φ

虚数根(12個?)
z = (1+√5)(-1±i√3)/4 = φω,φω~,

z = (1-√5)(-1±i√3)/4 = (-1/φ)ω,(-1/φ)ω~

z = -1±i√3 = 2ω,2ω~

z = -0.7660444431189780352 ± 0.6427876096865393263 i

z = -0.1736481776669303489 ± 0.9848077530122080594 i

z = 0.9396926207859083841 ± 0.34202014332566873304 i

ただし
 φ = (1+√5)/2 = 1.618034
 ω = (-1+i√3)/2 = e^(i(2π/3)),
 ω~ = (-1-i√3)/2 = e^(-i(2π/3)),
0867132人目の素数さん
垢版 |
2018/07/19(木) 16:59:29.95ID:QaNCXAoL
なんか日本語変だけど察してチョンマゲ。
zとbar{z}の絡み方から解の個数パッと出せたりするのかな?
0868132人目の素数さん
垢版 |
2018/07/19(木) 17:04:53.01ID:u5A76+YW
正n角形の各頂点に実数が配置されている。
負の頂点を選び、その数を辺でつながった2つの頂点に足し、それ自身の符号を反転する
([…, p, q, r, …] -> […, p+q, -q, r+q, …])という操作を、負の頂点がなくなるまで行う。

頂点上の数の総和が正である初期状態から始めたとき、次が成り立つことを示してほしい。
1.操作は有限回で終わる。
2.操作の回数は、初期状態のみに依り、途中の負の頂点の選び方に依らない。
3.最終状態も、初期状態のみに依り、途中の負の頂点の選び方に依らない。


また、以上のことは一般の有限グラフに自然に一般化されるが、
どのようなグラフが上記のような性質を持つだろうか?

パズルのようだけど、意外に深い数学につながっている問題です。
0869132人目の素数さん
垢版 |
2018/07/19(木) 17:20:48.38ID:QaNCXAoL
あれ?Winkler本の?これ答え知ってるからやめとこ。
ちょっと感動するよね。
0870132人目の素数さん
垢版 |
2018/07/19(木) 17:28:32.54ID:u5A76+YW
Winklerの本は見たことないけど、>>844を見て思い出したから、書いてみた
ほとんど、ある論文の丸写し
0871132人目の素数さん
垢版 |
2018/07/19(木) 17:37:14.65ID:NBL3eCRb
正5角形の各頂点の1つずつ整数を割り当て、それら5つの整数の和が正になるようにする.
連続する3個の頂点に割り当てられた整数をそれぞれ x, y, z とする.
このとき y < 0 ならば次の操作を行う
「3つの数 x, y, z をそれぞれ x + y, −y, z + y で置き換える.」
5つの整数のうち少なくとも1つが負である限り、上述の操作を繰り返し実行する.
有限回の操作の後、この手続きが完了するか否か決定せよ.

数学オリンピックに似た問題あるね
0873132人目の素数さん
垢版 |
2018/07/19(木) 20:53:20.77ID:QaNCXAoL
>>868
一般のグラフのときはどういう操作をするんですか?やっぱり真ん中を-1倍して残りに同じ数加えるんですか?
それとも変化量の総和が0になるようにするんですか?
0874132人目の素数さん
垢版 |
2018/07/19(木) 22:17:53.45ID:u5A76+YW
>>873
>真ん中を-1倍して残りに同じ数加える
です。そのため、総和が変化します。
また、初期状態の条件もグラフによって違ってきます。
例えば、
・―・―・―・―・
     | (下の点は上の中央の点とつながっている)
     ・
の場合は、どんな実数を配置しても(総和が正でなくても)有限回の操作で終わります。
0876132人目の素数さん
垢版 |
2018/07/19(木) 22:35:52.63ID:QaNCXAoL
もしかして長さ2の枝が3またに分かれてると総和正からだと有限で終わりで、どこかの枝がも一つ長いと終わらないとかになったりします?
0878132人目の素数さん
垢版 |
2018/07/20(金) 00:31:50.12ID:dYh8+g4F
>>874
>真ん中を-1倍して残りに同じ数加える
よく読んだら、なんか変?残り?
ちゃんと書くと、一般のグラフの場合も「負の頂点を選び、その数を辺でつながった各頂点に足し、それ自身の符号を反転する」です。

>>876
すごい。挙げた例から分かっちゃいましたか。専門分野によってはよく見かけるグラフですね。
だいたいあってますが、総和が正という条件は違います。

どんなグラフでも初期状態さえ制限すれば性質を満たすようにできるので、
論文は特別にうまくいくグラフのクラスを挙げるものです。
0879132人目の素数さん
垢版 |
2018/07/20(金) 02:01:25.14ID:+Kx7eSAL
やっぱり >>861 の計算は間違ってた。
正しい6次方程式は
a^6+3a^5-2a^4-12a^3-6a^2+5a+2=0
因数分解すると
(a+2)(a^2+a-1)(a^3-3a-1)=0
これは6つの実数解を持ち
a=-2,(-1±√5)/2,2cos(π/9),2cos(5π/9),2cos(7π/9)
(a,b) = (-2,4),((-1±√5)/2,(3∓√5)/2),(2cos(π/9),1),(2cos(5π/9),1),(2cos(7π/9),1)
ここから出てくる12個の虚数解は、>>864と同じだけど、
後半のものは
cos(π/9)±isin(π/9),cos(5π/9)±isin(5π/9),cos(7π/9)±isin(7π/9)
と書ける。
0880132人目の素数さん
垢版 |
2018/07/20(金) 02:17:37.41ID:smLQGUhz
>>857
z≠0 に対して
f(z) = z^3 - 3(z~)z + (1/z)(z~)^2 + 2,
とおくと、
f(zω) = f(zω~) = f(z),

3つ組×5個 と {0} か?
0881132人目の素数さん
垢版 |
2018/07/20(金) 03:36:25.83ID:smLQGUhz
>>864
・実数解(4個)
0,2,φ,-1/φ …… z(z-2)(zz-z-1)

・虚数解(12個)
φω,φω~  …… (zz +φz+φ^2)
(-1/φ)ω,(-1/φ)ω~  …… (zz-(1/φ)z +1/φ^2)
      これらの積:(z^4 +z^3 +2zz -z+1)
2ω,2ω~ …… (zz+2z+4)
残りの虚数解(6個) …… (z^6-z^3+1)
0882132人目の素数さん
垢版 |
2018/07/20(金) 03:59:51.98ID:+Kx7eSAL
>>857
やっと題意が見えた。
以下、zの共役をz'で表す。

z≠0のとき、r=|z|,z=rwとおくと、
w'=1/w,z'=r/w
与式をzで割って
z^3-3z'z+z'^2/z+2=0より
r^3・w^3 + rw'^3 - 3r^2+2 = 0

w^3が実数でないとき,r^3=r ∴ r=1
このとき、w^3 = αとおくと
α + 1/α -1 = 0
α^2 - α + 1 = 0
∴ α = e^(±πi/3)
∴ z = w = e^(±πi/9),e^(±7πi/9),e^(±13πi/9)

一方、w^3が実数のとき,w^3=±1

w^3=1のとき,r^3-3r^2+r+2 = 0
r>0より,r=2,(1+√5)/2
w=1,ω,ω'
z = rw = …

w^3=-1のとき,-r^3-3r^2-r+2 = 0
r>0より,r=(-1+√5)/2
w=-1,-ω,-ω'
z = rw = …
0883132人目の素数さん
垢版 |
2018/07/20(金) 04:29:19.98ID:+Kx7eSAL
z'z^2とzが、ガウス平面上で0からみて同じ向きにあることに気づいて
4項あるようで実は3方向のベクトルの和が0というようなイメージから入れば
わりとすんなりたどり着いたんだろうな。
0884132人目の素数さん
垢版 |
2018/07/20(金) 07:43:49.38ID:J1ODn3P8
>>878
う〜む、特別にうまくいくクラスはAn、Dn、E6、E7、E8?
long とか short とかの議論混じらないだろうし。
いわゆる “こういうグラフを含む→うまくいかない” となる “こういうグラフ” を列挙しといて
“そういうのを含まないのは××…” 的な攻め方するやつかな。
で、その “こういうグラフ” のリストが Dynkin Diagram 導く場合のやつと一致するんかな?
>>868 の前半は答え知ってるから後半考えてみる。
でも論文レベルの話だと流石に無理かな?
0885132人目の素数さん
垢版 |
2018/07/20(金) 10:39:33.47ID:GloVKkCh
グラフ系でもう少しやさしいやつ。

周期n>0の実数列 ‥‥,a[(-1),a(0),a(1),‥‥に対して一斉に
a(i)を[a(I)/2]+[a(i-1)/2]に置き換える。ただし[]はガウス記号。
つまりふたつにわって端数は切り捨て片方を次の人に一斉に渡す。
この操作を有限回行えば定数列になる事を示めせ。

これも例のパズル本の問題です。
0886132人目の素数さん
垢版 |
2018/07/20(金) 12:18:56.91ID:+Kx7eSAL
>>885
グラフで考えるのはよくわからないけど…

k回操作後(初期状態ではk=0)の
1周期の中でのa(i)の最大値をA_k,最小値をB_kとし,C_k=2*[B_k /2]とおく。
さらに、1周期の中でA_kと一致するa(i)の個数をN_kとすると
以下のことが言える。
・A_k≧C_k
・C_{k+1}≧C_k
・A_{k+1}≦A_k(A_kが奇数のときはA_{k+1}<A_k)
・A_{k+1}=A_kとなるとき、{a(i)}が定数列でない限りN_{k+1}<N_k
よって、C_kは減ることはなく、{a(i)}が定数列にならない限り
A_kは着実に減っていく(n回以下の操作で1以上減る)が、
A_kはC_kを下回ることはできないので、いつか必ず{a(i)}は(偶数の)定数列となる。

>・A_{k+1}=A_kとなるとき、{a(i)}が定数列でない限りN_{k+1}<N_k
のあたりの説明でグラフを使うのかな?
0888132人目の素数さん
垢版 |
2018/07/20(金) 17:32:04.98ID:GloVKkCh
>>886
正解です。グラフはあんまり気にしないでください。
原題が周期列でなく円状に並んだ数列として出題されてただけです。
――本の解答――
必要なら最初の列の全てに同じ偶数を足して全て非負の実数としてよい。2番目の列以降は全て整数の列ゆえ、全て整数の列としてよい。
総和は各項を2で割った後、切り捨てを行なった分発生するが、非負の整数の列だから総和が無限に減少することはない。
よって十分大きい操作の後に現れる列は偶数の列であるので、列に奇数は現れないとしてよい。
よって置き換えはa(i)→a(i)+a(i-1)であるとしてよい。
k番目の列の1項からn項を縦に並べた列ベクトルをvk、Aをn次正方行列でA1nとA(i+1)iが1で残りは0の行列、Iをn次の単位行列とするとき v(k+1) = (1/2)(A + I)vkである。
ζを1の原始n乗根とするときA+Iの固有値は(1+ζ^m)/2の形であり、m=0の時を除いてその絶対値は1未満である。
よってvkはk→∞のときA+Iの1に対する固有ベクトルの定数倍に収束するが、それは全ての成分が同じ値からなるベクトルである。vkは整数値からなるベクトルゆえ主張は示された□
0889132人目の素数さん
垢版 |
2018/07/20(金) 20:10:49.35ID:+Kx7eSAL
>>857のすべての答えを解とするn次方程式を後付けで作っても
意味ないと思うんだよな…
元の問題からなんらかの手続きによりその方程式が得られるという話ならわかるのだが。
0890132人目の素数さん
垢版 |
2018/07/20(金) 23:44:33.17ID:RMS2vK39
またまたWinkler本から。囚人と看守のやつです。
----
あなたは他のn人と共にある牢獄に収監されている。
ある日看守からあるゲームに参加することを指示された。
勝てば全員釈放、負ければ全員死刑である。
ゲームの内容は以下のようなものである。
まず看守は秘密裏に囚人に順序付けを行う。
囚人は全員独居房に入れられた後、一人ずつ呼び出され、自分以外のn-1人の並びについて教えられる。
その情報のみに基づいて各囚人は赤か青の帽子を選択できる。
帽子を選択した後、囚人は独居房に戻される。
この作業を全員について行ったのち、囚人全員が集められ、先の決められた順に応じて整列させられる。
その際、囚人のかぶっている帽子の色が赤青交互になっていれば(一人目は赤でも青でもよい)囚人の勝ち、なっていなければ看守の勝ちである。
囚人に許されているのは、このルール説明の後、独居房に移される前に一度だけ全員が集まって選ぶ帽子の色についての取り決めをしておくことだけである。
それ以降には囚人は互いに情報を交換する機会は一切与えられない。
全員が釈放されるための取り決めを考えてほしい。
----
答え見て「へぇ、こんなシンプルな取り決めでうまくいくもんだなぁ」とちょっと感心しました。
取り決め自身はシンプルなんですが、その取り決めでうまくいく部分の証明がややてこずるかもしれません。
一つ解を見つければ正解です。
当方も別解があるのかどうか知りません。
0891132人目の素数さん
垢版 |
2018/07/21(土) 00:02:23.48ID:1aHwZ8/O
>>890
訂正。問題文一行目
×:あなたは他のn人と共にある牢獄に収監されている。
○:あなたは他のn-1人と共にある牢獄に収監されている。
0892132人目の素数さん
垢版 |
2018/07/21(土) 01:14:39.48ID:9U7inpZN
>>890
こんな感じ?

あらかじめ全員の仮の並び順(並び順A)を決めておく。
各囚人は、看守に教えられた自分以外の並び順の最後尾に自分がいるような
並び順を考え(並び順B)、AをBに並べかえる置換が偶置換か奇置換かを調べる。
偶置換なら赤、奇置換なら青をかぶる。

実際にはもう1つ正しい並び順Cが存在するので、AからBへの置換は
A→C→Bとみなすことができ、
A→Cの置換は固定なので、B→Cの置換、すなわち
正しい並び順から自分自身を最後尾に回す並べ替えが偶置換か奇置換かにより
赤か青かが変わるからくりになっている。


パズルに出てくる囚人はみんな数学が得意だな
0893132人目の素数さん
垢版 |
2018/07/21(土) 03:25:53.00ID:4/chbJgW
>>844

ピーター・ウィンクラー「とっておきの数学パズル」日本評論社(2011/July)
 296p.2592円 坂井・岩沢・小副川(訳)
http://www.nippyo.co.jp/shop/book/5638.html
(§4.9の問題が数セミ・エレ解に出題された:2018/Jan/出題1)

ピーター・ウィンクラー「続・とっておきの数学パズル」日本評論社(2012/July)
 256p.2160円
http://www.nippyo.co.jp/shop/book/5955.html
0894132人目の素数さん
垢版 |
2018/07/21(土) 08:16:38.20ID:m9Eu8Sym
>>892
すばらしい!正解です。証明も用意してあった物とほぼ同じです。
看守の順でk番目、(k+1)番目の囚人をとり、Cのk番目の囚人をn番目に回した順列をBk、k+1番めの囚人のそれをB(k+1)とすれば
A→B(k+1)と写す置換hはA→B(k+1)と写す置換gに(k k+1)をつなげたものになるので
sgn(h) = sgn((k k+1) g) = (-1) sgn(g)
となることを利用すればよいというものです。
答え聞くと簡単なんですけどねぇ。

>>893
それです。
なかなか楽しい本です。
おすすめ。
0895132人目の素数さん
垢版 |
2018/07/21(土) 09:21:56.27ID:FMy4cnA/
>>868
とりあえずグラフがAn、Dn、E6、E7、E8の場合有限回で終わることはわかった。
―-
グラフGが上記のいずれかとする。
点vに対して>>868の操作をする変換をgvとする。
隣接する2点v、wについて(gw・gv)^3 = e、gv^2 = eからこれらの操作のなす群はワイル群の商群。
とくに各頂点に現れうる実数は有限個しかない。
端点に対する変換の回数が有限回しかなければ点数に対する帰納法の仮定に矛盾。
端点に対する変換の回数が無限回であるなら総和は無限に増大するが、各頂点に現れうる実数は有限個しかない事に矛盾。

あとは上記でないグラフでないもののうちの極小であるものそれぞれで無限回操作が続きうる事示せばいいはずだけどどんなグラフだったか忘れたorz。
0896132人目の素数さん
垢版 |
2018/07/21(土) 13:06:12.95ID:bUyrGG79
整数k,nは 0≦k≦n を満たすとする。
A⊂(Z/2Z)^n が 2|A|>2^k を満たすならば |2A|≧2^k が成り立つことを示せ。
ただし、2A={a+a': a,a'∈A} とする。
0897132人目の素数さん
垢版 |
2018/07/21(土) 13:14:22.58ID:zqilyAfN
2Aの定義がステキ
0901132人目の素数さん
垢版 |
2018/07/21(土) 14:39:19.69ID:vXT4z2wi
面白い問題といえば、和算の本にはいろいろと
面白そうというか難しい問題が掲載されていますよ。
もっともほとんどすべてが円に関する問題ですが。
0902132人目の素数さん
垢版 |
2018/07/21(土) 14:59:08.89ID:7HCB/VB4
>>896
Fを2元体とする。
Aは0を含むとしてよい。
この時2AはFベクトル空間となる。
|2A|<2^kとする。
この時dim 2A<kであるからF^nのベクトル空間の自己同型φをφ(2A)が第k成分以降が全て0の元からなる部分空間Vに含まれるようにとれる。
この時φ(A)もVに含まれるがVの元数は2^(k-1)であるので仮定に反する。

簡単に見えて案外難しい‥‥まぁもっと楽な方法もあるかもだけど。
0904132人目の素数さん
垢版 |
2018/07/21(土) 17:13:05.39ID:oD6UOuI2
>>899 >>900
言い忘れてた、そうです元の個数です
0905イナ ◆/7jUdUKiSM
垢版 |
2018/07/21(土) 19:06:13.60ID:g6r8bf/f
>>901関孝和?
縦9寸、横12寸の直角三角形に内接する同じ大きさの二個の円の直径を求める問題みつけた。
/_/_/_人_/_/_/_
/_/_(_)/_/_/_
/_/_(__)/_/_/_
/_/_((^。^)/_/_/_
/_/_(_っ-┓_/_/_
/_/_◎゙┻υ◎゙/_/_
/_/_/_/_/_/_/_/_/_/_/_/キコキコ……/_/_/_/_/_/_/_/_/_/_/_/_/_/_/_/_/_/_/_/なかなかやりおる。
0906132人目の素数さん
垢版 |
2018/07/21(土) 23:30:10.14ID:4/chbJgW
>>905

3頂点を O(0,0) A(0,9) B(12,0) とする。
(x,y) と直線ABの距離は (1/5)|36-3x-4y|

P(r,9-2r) を中心とする半径rの円pはOA,ABに接する。
Q(3(4-r),r) を中心とする半径rの円qはOB,BAに接する
 PQ = 5(3-r),
また、2円p,qが外接する。
 PQ = 2r,
∴ r = 15/7
0908132人目の素数さん
垢版 |
2018/07/22(日) 00:11:59.89ID:1DdlAJLc
>>896
一応できたような気がするが、長い。

以下では、{ a+a'|a,a'∈A } のことを A+A と書くことにする(個人的に 2A と書きたくないので)。
Fを2元体とする。次の定理を示せば十分である。

定理:Vは有限次元のFベクトル空間とする。k≧0 と A⊂V は|A|>2^{k−1}を満たすとする。
このとき、|A+A|≧2^k が成り立つ。

証明:n≧0に関する命題P(n)を以下のように定義する。

P(n):Vはn次元のFベクトル空間とする。k≧0 と A⊂V は
|A|>2^{k−1}を満たすとする。このとき、|A+A|≧2^k が成り立つ。

任意のn≧0に対してP(n)が真であることを、nに関する数学的帰納法で示す。
P(0)について:Vは0次元のFベクトル空間とする(自動的にV={o}である)。
k≧0 と A⊂V は|A|>2^{k−1}を満たすとする。示すべきは|A+A|≧2^k である。
まず、1=|V|≧|A|>2^{k−1} より 1>2^{k−1} となるので、自動的に k=0 となる。
次に、|A|>2^{k−1}>0 より A≠φであり、よって A+A≠φであり、よって
|A+A|≧1=2^0=2^k である。よって、P(0)は真である。

次に、n≧1を任意に取る。P(n−1)は真とする。P(n)も真であることを示す。
Vはn次元のFベクトル空間とする。k≧0 と A⊂V は|A|>2^{k−1}を満たすとする。
示すべきは|A+A|≧2^k である。
0909132人目の素数さん
垢版 |
2018/07/22(日) 00:14:30.08ID:1DdlAJLc
簡単のため、B:=A+A と置く。示すべきは|B|≧2^k である。
まず、Fが2元体であることから|V|=2^n となることが分かる。これと
|V|≧|A|>2^{k−1} より 2^n>2^{k−1} となるので、自動的に n≧k である。
次に、|A|>2^{k−1}>0よりA≠φである。よって、a∈A が1つ取れる。
このとき a+a∈A+A=B すなわち o∈B である(Fは2元体なので a+a=o である)。

さて、|B|≧2^k を示したいのだった。もし V−B=φ ならば、
V=B となるので、|B|=|V|=2^n≧2^k である(n≧kに注意)。
よって、この場合は成立。以下では、V−B≠φ としてよい。
そこで、x∈V−B を1つ取る。o∈B だったから、自動的に x≠o である。
W:={λx|λ∈F} (={o,x}) と置けば、W は V の部分空間である。
また、x≠o に注意して dim(W)=1 である。

この W を利用して、s,t∈V に対して s〜t ⇔ s−t∈W と定義すれば、〜 は V 上の同値関係になることが分かる。
s∈V の同値類を [s] と書くことにする。商集合 V/〜:={ [s]|s∈V } は自然な定義でFベクトル空間となる。
また、ベクトル空間の商空間の一般論から dim(V/〜)=dim(V)−dim(W) = n−1 となることが分かる。
0911132人目の素数さん
垢版 |
2018/07/22(日) 00:17:45.51ID:1DdlAJLc
さて、

A':={ [a]|a∈A } ⊂ V/〜

と置くと、|A'|=|A|である。実際、f:A→A' を f(a):=[a] で定義すれば、
これは明らかに well-defined かつ全射である。また、f は単射である。実際、
f(a_1)=f(a_2), a_i∈A とすると、[a_1]=[a_2] であるから a_1〜a_2 となる。
よって、a_1−a_2∈W={o,x} すなわち a_1−a_2=o,x である。a_1−a_2=x のときは、
x=a_1−a_2=a_1+a_2∈A+A=B となる(Fは2元体なので −a_2=a_2 である)。
しかし、x∈V−B だったから矛盾する。よって、a_1−a_2=o となるしかない。
よって、a_1=a_2 となるので、f は単射である。よって、fは全単射となったので、
|A|=|A'|である。|A|>2^{k−1} だったから、|A'|>2^{k−1} となる。

今の段階で、次が成り立っている。

・ V/〜 は(n−1)次元のFベクトル空間, k≧0, A' ⊂ V/〜, |A'|>2^{k−1}.

よって、P(n−1)が真であることから、|A'+A'|≧2^k である。
0912132人目の素数さん
垢版 |
2018/07/22(日) 00:21:20.07ID:1DdlAJLc
次に、|B|≧|A'+A'|が成り立つことを示す。g:B → A'+A' を g(b):=[b] で定義すると、
これは well-defined である。実際、b∈B を任意に取ると、b=a_1+a_2, a_i∈A と表せるので、
このような表示を何でもいいから1つ取れば

g(b)=[b]=[a_1+a_2]=[a_1]+[a_2]∈A'+A'

であり、よって well-defined である。また、g は全射である。実際、c∈A'+A' を任意に取ると、
A' の定義から、c=[a_1]+[a_2] なる a_i∈A が取れる。このような a_1,a_2 を何でもいいから
1つずつ取って b:= a_1+a_2∈B と置けば、g(b)が定義できて

g(b)=[b]=[a_1+a_2]=[a_1]+[a_2]=c

となるので、確かに g は全射である。g:B → A'+A' だったから、以上より、|B|≧|A'+A'|である。
|A'+A'|≧2^k だったから、以上より、|B|≧2^k である。

よって、いずれの場合も|B|≧2^kとなったので、P(n)は真である。
数学的に帰納法により、任意のn≧0に対してP(n)は真である。よって、題意が成り立つ。
0913 【ぴょん吉】
垢版 |
2018/07/22(日) 00:56:01.57ID:SEmuhAob
>>905
三平方の定理より、
直角三角形の斜辺は、
√(9^2+12^2)=15(寸)
円の直径を2rとすると、
二つの円の中心間の距離も2rなので、直角三角形の辺の比は、
3:4:5=3(2r/5):4(2r/5):2r
=(6r/5):(8r/5):2r
直角三角形の斜辺は、
{9寸-(6r/5)-r}+2r+{12寸-(8r/5)-r}=15寸
6寸=14r/5
2r=30/7(寸)
0914132人目の素数さん
垢版 |
2018/07/22(日) 01:38:28.55ID:41rzkBcX
>>912
正解です素晴らしい!実はより一般に
X,Y⊂F^n が空でない時、 |X|+|Y|>2^k ならば |X+Y|≧2^k
を示してからその系として導く解法を想定していたのですが、本質的にかなり簡単になっていて驚きました。V\(A+A) から元をとる発想はなかったです。。
0915132人目の素数さん
垢版 |
2018/07/22(日) 01:53:47.71ID:1DdlAJLc
>>914
>X,Y⊂F^n が空でない時、 |X|+|Y|>2^k ならば |X+Y|≧2^k

なんと、そんな定理も成り立つのか (^o^)

実は、>>896 の F_p バージョンである次の定理が、>>908 と同じやり方で証明できる。
―――――――――――――――――――――――――――――――
定理:p は素数とする。V は有限次元の F_p ベクトル空間とする。
k≧0 と A⊂V は|A|> p^{k−1} を満たすとする。このとき、

|Σ[i=1〜p] A|≧ p^k

が成り立つ。ただし、Σ[i=1〜p] A := { Σ[i=1〜p] a_i|a_1,…,a_p∈A }
と定義する。
―――――――――――――――――――――――――――――――

この定理を

>X,Y⊂F^n が空でない時、 |X|+|Y|>2^k ならば |X+Y|≧2^k

と見比べると、たぶん次の定理も成り立つのかな (^o^)
―――――――――――――――――――――――――――――――
定理(?): p は素数とする。V は有限次元の F_p ベクトル空間とする。
k≧0 と空でない X_1,X_2,…,X_p⊂V は Σ[i=1〜p]|X_i|> p^k を
満たすとする。このとき、|Σ[i=1〜p] X_i|≧ p^k が成り立つ。
―――――――――――――――――――――――――――――――
0917132人目の素数さん
垢版 |
2018/07/22(日) 12:19:45.04ID:U4aZuyBV
>>915
情報ありがとうございます。色々考えてみました。
上の定理(?)には、残念ながら反例があるようです。W⊂VをVのk次元部分空間として、
X_1=W∪{a} (ただしa∈VはWに属さない元)
X_i=W (i=2,…,p)

代わりに次の定理が成り立つようです。
(定理)pを素数、整数、 1≦m<p とし、VをF_pベクトル空間とする。
整数 k≧0 と空でない X,Y⊂V が |X|+|Y|>mp^k を満たすならば |X+Y|≧mp^k が成り立つ。□
折角なので証明の概略だけ置いときますね
〜〜〜〜〜〜〜〜
a∈V, S⊂V に対して a+S={a+s: s∈S} と定める。
適当な x∈X, y∈Y をとれば |-x+X|=|X|, |(-x+X)+(-y+Y)|=|X+Y| 等が成り立つので、 0∈X,Y の場合のみを考えればよい。
X∩(-y+Y)≠X が成り立つような y∈Y が存在する時、次の操作を考える。
(操作)「X'=X∩(-y+Y), Y'=X∪(-y+Y) とし、 XをX'に、YをY'に置き換える」
この操作により、|X|は減少し、|X|+|Y| は保たれる。
また、X'+Y'⊂X+(-y+Y) より、|X+Y| は非増加となる。新しいX,Yはどちらも0を元に持つ。
この操作は、|X|の狭義単調減少性により、有限回でできなくなる。
このような最終状態のX,YをそれぞれP,Qとおくと、操作ができないことから、任意のq∈Qについて P∩(-q+Q)=P が成り立つ。
ゆえに、q+P⊂Q より、 Q+P⊂Q.
したがって、Q+<P>=Q. (ただし、<P>はPが張るベクトル空間。)
これより、元を足すことによる<P>のQへの作用を考えることができるが、この作用による任意の軌道は|<P>|個の元を持つので、|Q| は |<P>| の倍数。…[1]
また、|Q|+|P| = |X|+|Y| > mp^k より
|Q| > mp^k - |<P>| となる。 …[2]
|<P>| ≦ p^k の場合、[1]と[2]より |Q| ≧ ([2]の右辺)+|<P>| = mp^k.
|<P>| ≧ p^(k+1) の場合、 <P>⊂Q より |Q|>mp^k.
したがって、いずれの場合も |P+Q| ≧ |Q| ≧ mp^k.
操作により|X+Y|は非増加であったから、 |X+Y| ≧ |P+Q| ≧ mp^k. □
0918132人目の素数さん
垢版 |
2018/07/22(日) 16:50:21.14ID:X11xpoqn
和算の問題です。
一つの円があります。
その円の中に、大、中、小の円を内接させます。
條件は、大、中、小の円は、一番外側の円に内接します。

大円は、中円と小円に外接します。
中円は大円と小円に外接します。
小円は、大円と中円に外接します

この場合、この4つの円の関係を求めてください。

出典」三上義夫「日本数学史」(この本は、「科学図書館」という
サイトに全文がPDFファイルとしてアップされています)
0919イナ ◆/7jUdUKiSM
垢版 |
2018/07/22(日) 20:03:51.82ID:SEmuhAob
>>918
外側の円の半径:Я
大円の半径:R
中円の半径:R
小円の半径:r
とすると、
Я>R+R>R>R>r
0920132人目の素数さん
垢版 |
2018/07/22(日) 20:53:32.75ID:XHMrpicM
大円の半径が3
中円の半径が2
小円の半径が1
のときの内側の円の半径は?
とかにしないと問題としては答えにくくね?
まぁこのケースはそんなに難しくないかもしれないけど。
この3円に外接するの方が難しいのかな?
数値もへぇって値になった記憶が
0922132人目の素数さん
垢版 |
2018/07/22(日) 23:51:55.24ID:8X1Zeg9C
反転法使えばそんな難しくなさそうだけど、暗算できるほど簡単ではないな。
0925イナ ◆/7jUdUKiSM
垢版 |
2018/07/23(月) 08:16:34.35ID:7/0/1MEy
よって四つの円の包含関係は、
外側の円⊃大円
外側の円⊃中円
外側の円⊃小円
但し、大円、中円、小円はたがいに外接する。
>>919
0926132人目の素数さん
垢版 |
2018/07/23(月) 10:11:04.09ID:+uNFdt3Z
デカルトの円定理
0928132人目の素数さん
垢版 |
2018/07/23(月) 12:19:19.88ID:BhRl/p7g
複素係数一変数多項式 f, g であって {f(x)}^2 + {g(x)}^2 = x を満たすものは存在するか。
0930132人目の素数さん
垢版 |
2018/07/23(月) 17:17:16.75ID:KhSOAKcF
ごめん間違えた、
f^2 + g^3 = x を満たす多項式は存在するか
でした
0931132人目の素数さん
垢版 |
2018/07/23(月) 21:21:51.05ID:7FS8HckQ
>>930
できたか?
存在すると仮定する。
f(x)がxを因子に持てばg(x)もxを因子にもちv_x(左辺) ≧ 2、v_x(右辺) = 1により矛盾。
よってf(x)はxを因子に持たない。よってf(t^2)もtを因子に持たない。
与式より
g(t^2)^3 = (t - f(t^2)) (t + f(t^2))
であるが、(t-f(t^2),t+f(t^2)) = (2t,t+f(t^2)) = 1によりt-f(t^2)とt+f(t^2)は互いに素である。
よってg(t^2)の因子のうちt-f(t^2)の因子になっているものの積をh(t)、t+f(t^2)の因子になっているものの積をk(t)とおけば
h(t)^3 = c(t - f(t^2))、k(t)^3 = d(t + f(t^2))、cd = 1
となる定数c,dがとれる。
h,kをc,dの3乗根で割ったものに取り替えれば
h(t)^3 = t - f(t^2)、k(t)^3 = t + f(t^2)
となるとしてよい。
一方t-αがh(t)の因子なら-t-αはk(t)の因子であるからk(t) = e h(-t)となる定数eがとれる。
このとき
h(t)^3 - t = - f(t^2) = t - k(t)^3 = t - e^3 h(-t)^3
により
h(t)^3 + e^3 h(-t)^3 = 2t
となる。
ここで容易にg(x)の次数は奇数であり、h(t)とk(t)の次数は等しいからh(t)の次数も偶数である。
よって上の式の最高次からe^3 = -1がわかる。
よって
h(t) + h(-t) = 2t
を得るが左辺は偶関数により矛盾。
0932132人目の素数さん
垢版 |
2018/07/23(月) 23:46:59.29ID:jsKLvMqB
>>931
まちごうた。最後から2行目
h(t)^3 - h(-t)^3 = 2t
で左辺の次数は3(deg h)-1で5以上より矛盾。
0933132人目の素数さん
垢版 |
2018/07/24(火) 01:11:19.24ID:v83j+alb
>>931-932
改めて清書するとミスや余計な議論のオンパレードだけど、もう修正のせるとスレ汚しになるのでやめときます。
ホントはC(x)上の楕円曲線

 Y^2 = - X^3 + x

の有理点についての議論でかっこよくやるのが通なんだろうけどオラには無理。
なんか数オリの解答みたいになってヤだけどこれしか思いつかん。
0934132人目の素数さん
垢版 |
2018/07/24(火) 01:17:52.25ID:rRTBzOQ4
>>930
メーソン・ストーサーズの定理(ABC予想の多項式版)を使ったら凄く簡単に出たw
もちろん、「存在しない」が答え。
0937132人目の素数さん
垢版 |
2018/07/24(火) 04:25:23.69ID:lehQeRGl
>>931
レス遅くなりました。23時頃に一度投稿しようとして投稿規制くらった文章をそのまま載せときます。
最後の定理は既出でしたね
〜〜〜〜〜〜〜〜

>ここで容易にg(x)の次数は奇数であり、
以降がちょっと難しいですがその直前の式でほぼ矛盾が示せているので正解とします。
(直前の式の左辺を因数分解して、両辺の次数を比べ、左辺の3つの因数のうち少なくとも二つが次数0でなければならないことからも矛盾が示せます)

実はABC予想の多項式版の類似であるメーソン・ストーサーズの定理を使えば比較的簡単に解くことができるので、よければ調べてみてください
0938132人目の素数さん
垢版 |
2018/07/24(火) 06:21:54.98ID:bmjGlIcJ
>>937
ありがとうございます。
比較的どころかメーソン・ストーサーズの定理使えば瞬殺ですね。勉強しときます。
0939132人目の素数さん
垢版 |
2018/07/24(火) 17:51:22.07ID:xg2jMb4Q
これもしかして、このメーソン・ストーサーズの定理が元でその数論版がABC予想とかではないんですかね?
やっぱりABCの方が先?
0940132人目の素数さん
垢版 |
2018/07/26(木) 04:23:17.15ID:gCZSgyqq
群Gの正規部分群NはZ(整数)と同型
G/NはZ/nZと同型
nは1より大きい整数
Gの構造を決定しろ
0942132人目の素数さん
垢版 |
2018/07/26(木) 09:42:15.39ID:r9ee9dZW
>>940
以下x’:=x^(-1)とする。
N=<a>とし、b∈G\NをbNがG/Nの生成元となるようにとる。
準同型x→bxb’は同型N→Nを引き起こすがこのときaの像はa,a’のいずれかである。
(i) bab’ = aのとき。
このときG≌Z⊕Z/nZである。
(ii) bab’ = a’ のとき。
nが奇数とするとn = 2q + 1とおくとき
a = b^(2q+1)ab^(-(2q+1)) = bab’ = -a
となって矛盾するからnは偶数である。
逆にnが偶数のとき
<a,b|bab’ = a’,b^n=e>
はZのsemi trivial extensionであり、条件を満たす。
0943132人目の素数さん
垢版 |
2018/07/26(木) 11:06:27.02ID:g7KCpv5X
>>832
返信遅くなって申し訳ない (>>826です)
超越基底を用いれば簡単になるであろうということ。
――――――――
何か鳩ノ巣原理を使う難問が欲しい
此れだけでは申し訳ないので幾つか投下:
(難問ではなく何も既知の筈)

・長さNの正整数から為る数列が2つ存在し, 数列を構成する数は1以上N以下である.
A, Bから其々空でない部分数列を適当に選ぶとき, 其々の総和を等しく出来るか.

・αが無理数の時, 次の不等式を満たす整数の組(p,q)が無限に存在することを示せ:
|α-p/q|<1/q²

序でに右辺をk/q²として不等式を満たす組が無限に存在する様な最小のkを求めると, k=1/√5になることが知られてる

母関数を使うらしく, 序でにk=1でも分母の自乗はx(>2)乗に変えた瞬間成り立たないそうだ(知ってるのは此処まで).
0944132人目の素数さん
垢版 |
2018/07/26(木) 12:19:01.89ID:B8nOkJxx
【何故シヌの、JK″】 島津論文「安倍とオウムに接点」 露国防相「気づかれてないと思うな晋三」
http://rosie.5ch.net/test/read.cgi/liveplus/1532569537/l50



地震多すぎ! 日本は地震大国だから、は大ウソだった! ほら吹きの安倍が、地下核実験をやっている!
0945132人目の素数さん
垢版 |
2018/07/27(金) 06:49:09.69ID:0WjqahXc
>>940
Zの自己同型は±1のみ
Zn→Z2はnが偶数なら0と全射nが奇数なら0のみ
よって
Z+Znか偶数ならZとZnの半直積
094682
垢版 |
2018/07/27(金) 07:59:20.49ID:sps923Uv
>>82の正解発表
>>87 (A)正解
>>85 (B)正解
094782
垢版 |
2018/07/27(金) 08:00:12.73ID:sps923Uv
>>82(A)の模範解答】
以下、図形の内部には周も含める。

[補a]
幅1のルーローの三角形は、内部の任意の2点間の距離が1以下である。
半径1、中心角60°以下の扇形は、幅1のルーローの三角形の一部である。よって、内部の任意の2点間の距離が1以下である。 ■

[A]
5点ならば、円に内接する正五角形の各頂点に配置すれば、どの2点間の距離も1より大きくなる。
どの2点間の距離も1より大きくなるような6点の配置を考える。
円の中心に1点Aを配置すると、円の中心と内部の任意の点との距離は1以下だから、Aと他の5点との距離は全て1以下になる。よって、円の中心以外にAを配置する。
円を扇形で6等分すると、Aが2つの扇形X,Yの境界に乗るようにすることができる。別の点をX,Yに配置すると、[補a]より、Aとその点との距離は1以下になる。よって、X,Y以外の4つの扇形に残りの5点を配置する。
鳩の巣原理より、少なくとも2点は同じ扇形の内部になる。[補a]より、その2点間の距離は1以下になる。
したがって、どのように6点を配置しても、ある2点間の距離が1以下になる。
最小のmは6である。 ■
094882
垢版 |
2018/07/27(金) 08:02:00.38ID:sps923Uv
>>82(B)の模範解答】
以下、図形の内部には周も含める。

[補b]
半径1/2の円の内部の2点間の距離は、直径の両端のときは1、それ以外のときは1未満である。
一辺1/2の正六角形は、半径1/2の円に内接する。よって、内部の2点間の距離は、最も遠い頂点どうし(3組ある)のときは1、それ以外のときは1未満である。
カップケーキ形(図の黄色部分)は、一辺1/2の正六角形の一部である。よって、内部の2点間の距離は、弧の両端のときは1、それ以外のときは1未満である。
弧の片側の端点を欠いたカップケーキ形(以下、単に「図形」)は、元のカップケーキ形の一部である。よって、内部の2点間の距離は1未満である。 ■

[B]
7点ならば、円の中心と、円に内接する正六角形の各頂点に配置すれば、どの2点間の距離も1以上になる。
どのような8点の配置も、ある2点間の距離が1未満であることを背理法で示す。
どの2点間の距離も1以上であるような8点の配置が存在すると仮定する。
http://imgur.com/kTTm64O.jpg
半径1の円は、7つのパーツ
一辺1/2の正六角形ABCDEF、図形HBA(G)、図形ICB(H)、図形JDC(I)、図形KED(J)、図形LFE(K)、図形GAF(L)
で覆うことができる。
鳩の巣原理より、少なくとも2点は同じパーツの内部にある。[補b]より、その2点は正六角形のパーツの内部にある。AとDにあるとして一般性を失わない。
[補b]より、A,Dを含む図形4つには別の点はない。また、[補b]より、図形LFE(K)とICB(H)にはそれぞれ最大で1点しかない。
このとき、合計で最大でも4点しか配置されていないため矛盾。仮定は誤りであった。
したがって、どのように8点を配置しても、ある2点間の距離が1未満になる。
最小のnは8である。 ■
094982
垢版 |
2018/07/27(金) 08:03:17.37ID:sps923Uv
出典
(A)
https://www.cut-the-knot.org/pigeonhole/six_points.shtml
このサイトに証明が2つ載っている。
1つ目は上記。
2つ目は>>87と同じ方針で最後に[補a]を使わない方法である(三角形の内角の大小関係を使っている)。
(B)
https://math.stackexchange.com/questions/1228119/
1点を欠いた図形を考えるのがミソである。この一工夫で証明はかなり楽になる。クレバーな方法。

(B)の画像はGeoGebraで自作した。

この手の問題は
「定幅図形(またはそれらに内包される図形)で元の図形を被覆/分割して、鳩の巣原理に持ち込む」
のが定石だが、効率の良い被覆や分割は発見に試行錯誤を要することが多い。
「図形Aに、どのようにk個以上の点を配置しても、ある2点間の距離がd以下/未満になる」
みたいな一般化は厳しいだろう。
0950188
垢版 |
2018/07/27(金) 08:05:38.26ID:sps923Uv
>>188の正解
正n角形について、辺AB,BC,CD…の順に鏡映を取っていく操作を
nが偶数ならばn-1回
nが奇数ならば2n-1回
それぞれ繰り返せば、l(n)を「平行な2辺間を結ぶ直線か折れ線の長さ」に帰着することができる。折れ線のときは直線のときより長いことを利用すれば、幾何的にl(n)の最小が求まる。
もちろん>>189みたいに式で解けるならそれに越したことはないが…
0951132人目の素数さん
垢版 |
2018/07/27(金) 09:50:50.05ID:NlkV/5Nh
>>856
解答です。
PQRSにおける接線の交点を結んで得られる四角形をXYZWとする。
XからCに引いた2接線の接点の交点をx、y,z,wに対するそれをy,z,wとする。
仮定よりXYZWは円D上にあるしてよい。
このときDのCに関する反転をdとするとxyzwはd上である。
よって主張は成立する。□
0952132人目の素数さん
垢版 |
2018/07/27(金) 22:17:47.94ID:uXdC9xjt
一辺の長さ1の正五角形の頂点を全て結ぶ分岐あり曲線の長さの最小値を求めよ
0953132人目の素数さん
垢版 |
2018/07/27(金) 22:19:49.89ID:uXdC9xjt
正n角形の頂点を全て結ぶ分岐あり曲線の長さが最小となるとき、分岐点の角度は必ず120°となることを証明せよ
0954132人目の素数さん
垢版 |
2018/07/27(金) 22:22:17.06ID:uXdC9xjt
>>953
ごめんこれ嘘
なんでもない
0955 【豚】
垢版 |
2018/07/28(土) 00:32:41.08ID:6VVd4WCT
>>952


∵五角形の中に桜の花びらを描くように半径1の弧を各頂点から描くと、
弧の最小単位
2π×(36°/360°)
が十個、頂点と分岐点を交互に通るかたちになる。

>>925開運!!
0957535
垢版 |
2018/07/28(土) 07:52:44.92ID:o+vDTN8W
>>535の正解発表

【Step 1 与式の分割】

例えば最初の分数式について
(a-b)(a-c)/(a+b+c)
=(1/2)(a-c)(a-c)/(a+b+c)
+(1/2)(a-c)(a-2b+c)/(a+b+c)
は容易に確認できる。

そこで
s=a+b+c+d
A'=(a-c)(a-c)/(s-d)
A''=(a-c)(a-2b+c)/(s-d)
B'=(b-d)(b-d)/(s-a)
B''=(b-d)(b-2c+d)/(s-a)
C'=(c-a)(c-a)/(s-b)
C''=(c-a)(c-2d+a)/(s-b)
D'=(d-b)(d-b)/(s-c)
D''=(d-b)(d-2a+b)/(s-c)
と置くと
(与式の左辺)
=(1/2)[A'+A''+B'+B''+C'+C''+D'+D'']
=(1/2)[(A'+B'+C'+D')+(A''+B''+C''+D'')]
と分割できる。
0958535
垢版 |
2018/07/28(土) 07:55:37.99ID:o+vDTN8W
【Step 2 A'+B'+C'+D'の評価】

√A',√B',√C',√D',√(s-a),√(s-b),√(s-c),√(s-d)にコーシー・シュワルツの不等式を適用すると
[A'+B'+C'+D'][(s-a)+(s-b)+(s-c)+(s-d)]
≧[{√A'}*{√(s-a)}+{√B'}*{√(s-b)}+{√C'}*{√(s-c)}+{√D'}*{√(s-d)}]^2 …△
⇔3s(A'+B'+C'+D')≧(|a-c|+|b-d|+|c-a|+|d-b|)^2
⇔3s(A'+B'+C'+D')≧(2|a-c|+2|b-d|)^2
相加相乗平均の不等式より
2|a-c|+2|b-d|≧2√(2|a-c|*2|b-d|)>0
だから
(2|a-c|+2|b-d|)^2≧16|a-c||b-d| …▲
よって
3s(A'+B'+C'+D')≧16|a-c||b-d|
⇔A'+B'+C'+D'≧16|a-c||b-d|/(3s) …@
0959535
垢版 |
2018/07/28(土) 08:02:39.32ID:o+vDTN8W
【Step 3 A''+B''+C''+D''の評価】

(a-c)(a-2b+c)(s-b)+(c-a)(c-2d+a)(s-d)
=(a-c)[(a+c-2b)(a+c+d)-(a+c-2d)(a+c+b)]
=(a-c)[{(a+c)^2+(d-2b)(a+c)-2bd}-{(a+c)^2+(b-2d)(a+c)-2db}]
=(a-c)[(d-2b-b+2d)(a+c)]
=3(a-c)(d-b)(a+c)
またM=(s-d)(s-b)とおくと
M=s(s-b-d)+db=s(a+c)+bd
A''+C''
=[(a-c)(a-2b+c)(s-b)+(c-a)(c-2d+a)(s-d)]/[(s-d)(s-b)]
=3(a-c)(d-b)(a+c)/M

同様にN=s(b+d)+acとおくと
B''+D''=3(b-d)(a-c)(b+d)/N

よってW=(b+d)M-(a+c)Nとおくと
W=(b+d){s(a+c)+bd}-(a+c){s(b+d)+ac}=(b+d)s(a+c)+(b+d)bd-(a+c)s(b+d)-(a+c)ac=(b+d)bd-(a+c)ac
A''+C''+B''+D''
=3(a-c)(b-d)[(b+d)/N-(a+c)/M]
=3(a-c)(b-d)[(b+d)M-(a+c)N]/(MN)
=3(a-c)(b-d)W/(MN)

ここで
MN={(a+c)s+bd}{(b+d)s+ac}=(a+c)(b+d)s^2+{(a+c)ac+(b+d)bd}s+bdac
>{(a+c)ac+(b+d)bd}s
x>0,y>0のときx+y>|x-y|より
{(a+c)ac+(b+d)bd}s>|(a+c)ac-(b+d)bd|s=|W|s
よって
MN>|W|s⇔(1/s)>|W|/(MN)

ゆえに
|A''+C''+B''+D''|=3|a-c||b-d||W|/(MN)≦3|a-c||b-d|/s …▼
したがって
A''+C''+B''+D''≧-3|a-c||b-d|/s …A
0960535
垢版 |
2018/07/28(土) 08:06:13.82ID:o+vDTN8W
【与式の証明と等号成立条件】
@とAより
(与式の左辺)
=(1/2)[(A'+B'+C'+D')+(A''+B''+C''+D'')]
≧16|a-c||b-d|/(3s)-3|a-c||b-d|/s
=16|a-c||b-d|/(3s)-3|a-c||b-d|/s
=7|a-c||b-d|/(3s) …☆
明らかに
7|a-c||b-d|/(3s)≧0=(与式の右辺) …★
よって
(与式の左辺)≧(与式の右辺)

(与式の等号が成立する)
⇔(☆、★の等号が成立する)
⇔(@、A、★の等号が成立する)
⇔(△、▲、▼、★の等号が成立する)

▲の等号成立条件は
2|a-c|=2|b-d|⇔|a-c|=|b-d|
▼と★の等号成立条件は
|a-c|=0∨|b-d|=0⇔a=c∨b=d
よって
a=c∧b=d
このとき△でも等号が成立している。

したがって、与式の等号成立条件はa=c∧b=d ■
0961535
垢版 |
2018/07/28(土) 08:08:38.70ID:o+vDTN8W
他の2つの模範解答もどう発想するのか判らない解答であるうえ、ただ煩雑で汚いので省略。
リンク先で見てください。
出典:IMO2008SL-A7
https://www.imo-official.org/problems/IMO2008SL.pdf
0962132人目の素数さん
垢版 |
2018/07/28(土) 08:13:53.49ID:Sc9m8D2O
|a b c d|
|b bx d cx|
|c d ay by|
|d cx by axy|
を因数分解せよ
0963132人目の素数さん
垢版 |
2018/07/28(土) 09:35:34.58ID:pdtqHzrG
>>418
解答です。
R=F[T]/(T^2+1)、X1={(x,y,z)∈X | z≠0}、X2={(x,y,z)∈X | z=0}
とおいてR^をRの可逆元のなす群としN:R→Fをノルム写像とする。
またTの類T+(T^2+1)をtとする。
x,y∈FにたいしてN(x+yt) = x^2+y^2である。
x∈Fでx^2+1≠0かつx^2+1がGに属さないものがとれる。
(∵1〜q-2のうちa∈G,a+1はGに属さないaをとってx^2=aとなるxをとればよい。)
このときN(x+t)=x^2+1はGに属さず0でもないのでx+tはR^\N^(-1)(G)に入る。
よってR^/N^(-1)(G)はR^の真部分群であり準同型定理によりZ/2Zであるとわかる。
とくに#N^(-1)(-G) = (1/2)#R^である。
以上により
#X = 2・(1/2)#R^ + (R\R^) = #R^ = q^2
である。
以下(a/q)を平方剰余記号とする。
(-1/q)=-1のときX2={(0,0,0)}であり#X2=1である。
(-1/q)=1のときu^2=-1となるu∈FをとればX2={(x,±ux,0)}であるから#X2=2q-1である。
以上により
#Y=#X-3#X2+3-1
=q^2-1 (q≡3 mod 4)、=q^2-6q+5 (q ≡ 1 mod 4)
である。
つぎに
Y2={(x,y,z) | x=y,xyz≠0}
とおくとき(-2/q)=-1ならばばY2=∅であり、
(-2/q)=1ならばv^2=-2となるv∈FをとればY2={(x,x,±vx)|x∈F^}であるから#Y2=2q-2である。
また
#Z=#Y-3#Y2
であるから以上と第2補充法則により
#Z=q^2-12q+11 (q ≡ 1 (mod 8))
#Z=q^2-6q+5 (q ≡ 3,5 (mod 8))
#Z=q^2-1 (q ≡ 7 (mod 8))
を得る。
0964132人目の素数さん
垢版 |
2018/07/28(土) 09:38:12.21ID:z2BC7zek
log2=0.3010, log3=0.4771が与えられている.
ここから, log11の小数第2位の値を求めよ.
0965132人目の素数さん
垢版 |
2018/07/28(土) 09:41:05.15ID:pdtqHzrG
>>962
>|a b c d|
>|b bx d cx|
>|c d ay by|
>|d cx by axy|
determinant(matrix([a,b,c,d],[b,b*x,d,c*x],[c,d,a*y,b*y],[d,c*x,b*y,a*x*y])),factor;
a^3*b*x^2*y^2−a*b^3*x*y^2−a^2*b^2*x*y^2+b^4*y^2−a*b*c^2*x^2*y−a^2*c^2*x^2*y−a*b*d^2*x*y−a^2*d^2*x*y+2*b^2*c*d*x*y+6*a*b*c*d*x*y−2*b^2*c^2*
x*y−2*b^2*d^2*y+c^4*x^2−2*c^2*d^2*x+d^4
????
0967132人目の素数さん
垢版 |
2018/07/28(土) 10:43:17.16ID:eDTZE8Ag
>>962
こう?
determinant(matrix([a,b,c,d],[b,a*x^2,d,c*x^2],[c,d,a*y^2,b*y^2],[d,c*x^2,b*y^2,a*x^2*y^2])),factor;
(a*x*y−b*y−c*x+d)*(a*x*y−b*y+c*x−d)*(a*x*y+b*y−c*x−d)*(a*x*y+b*y+c*x+d)
0968132人目の素数さん
垢版 |
2018/07/28(土) 13:37:27.98ID:25At2aHe
>>955
今更だけど不正解
少なくとも4つの辺を結べば5点を結べるから4より小さくないとおかしい
0969132人目の素数さん
垢版 |
2018/07/28(土) 14:57:21.13ID:z4N8++BV
>>953
分岐点に隣接する3点の作る三角形の外心と、分岐点が一致する
ということでいいんでない?
0970132人目の素数さん
垢版 |
2018/07/28(土) 15:11:48.34ID:z4N8++BV
いや、>>969はたぶん違うな……
むしろ分岐点の角がすべて120°のほうが正しい気がしてきた
0971イナ ◆/7jUdUKiSM
垢版 |
2018/07/28(土) 15:20:45.41ID:6VVd4WCT
>>968長さ4だと正五角形の周長より短いじゃないか。曲線じゃない直線だし。

一つの頂点を一回通ればいいってことか。
>>955
0973イナ ◆/7jUdUKiSM
垢版 |
2018/07/28(土) 15:37:26.36ID:6VVd4WCT
>>972わかった。十個の弧のうち四個は省けるね。
2π×(36゚/360゚)人人
/_/×6=4π/5(_^_)
/_/_/_/_/(__)
/_/_/_/_/(^。^))
/_人人_/_/_(_っ┓
/_(_)_)_/_/◎┻υ◎
/_( __)_/_/_/_/_
/_(_(`)_/_/_/_/_
/_(υ_)┓_/_/__/_/
/◎υ┻-◎_/_/_/_/_/_/_/_/_/_/_/_/_/_/_/_/屁でもねえや。前>>971それよかいいワープロねえか。
0974132人目の素数さん
垢版 |
2018/07/28(土) 16:14:37.96ID:Sc9m8D2O
>>966
axの間違い
0975132人目の素数さん
垢版 |
2018/07/28(土) 16:15:36.89ID:Sc9m8D2O
>>967
も1つ
abcdの多項式でxyについては2次拡大まで使うと1次4つの積に
0976132人目の素数さん
垢版 |
2018/07/28(土) 16:21:20.32ID:RosE4Rin
>>952
角度120度の前提で 3.891156823 って数値が出たけど、これより良い結果ある?
http://i.imgur.com/VhC8cug.png
0977132人目の素数さん
垢版 |
2018/07/28(土) 16:24:52.44ID:EnyRsA6W
ax = f, cx = g とおくと


|a, b, c, d|
|b, f, d, g|
|c, d, ayy, byy|
|d, g, byy, fyy|

= {(af-bb)yy + (cg-dd)}^2 - (ag+cf-2bd)^2 yy

= {(af-bb)yy +(ag+cf-2bd)y +(cg-dd)}{(af-bb)y^2 -(ag+cf-2bd)y +(cg-dd)},
0978132人目の素数さん
垢版 |
2018/07/28(土) 16:40:58.48ID:Sc9m8D2O
>>977
もひとつ
0979132人目の素数さん
垢版 |
2018/07/28(土) 16:56:48.40ID:zqnKg1oN
>>971
そう、一つの頂点を一回通ればいいってこと
あと直線も曲線の一部

>>969,970
正7角形の場合、周をなぞるのが一番短いから角度は120°じゃないって言おうとしたけどジャンクションではないね
ジャンクションに限定するなら120°は成り立ちます
「プラトーの法則」

>>976
不正解です

実は左右非対称になる
0980イナ ◆/7jUdUKiSM
垢版 |
2018/07/28(土) 17:37:50.18ID:6VVd4WCT
直線も曲線のうち!?
;;;;;;;;;;;人人;;;;;;
;;;;;;;;;;(_;^_);;;;;
;;;;;;;;;;(_^;_);;;;;
;;;人人;;;(^。^;);;;;;
;;(_)_);;(_っ┓;;;;
;;(_(_);◎゙┻υ◎゙;;
;;(_(`);;;;;キコキコ……
;;(υ_)┓;;;;;;;;;;;
◎゙υ┻-◎゙_/_/__/_/
/_/キコキコ……/_/_/_/_/_/_/_/_/きっといい地境がみつかる。前>>973
0981イナ ◆/7jUdUKiSM
垢版 |
2018/07/28(土) 18:05:00.89ID:6VVd4WCT
>>980
対角線2つ=1+√5
対角線の一つに残りの頂点から引いた垂線={√(5+2√5)}/2-{√(10-2√5)}/4
分割線=1+√5+{√(5+2√5)}/2-{√(10-2√5)}/4
0982132人目の素数さん
垢版 |
2018/07/28(土) 18:38:01.92ID:Sc9m8D2O
>>967
正解だったどもスマン
0983132人目の素数さん
垢版 |
2018/07/28(土) 18:40:27.16ID:Sc9m8D2O
ちなみに8次でも同じような問題できる
2^n次でできるのかも
0984132人目の素数さん
垢版 |
2018/07/28(土) 19:32:48.35ID:57kIc8+e
>>2
7+8-5=10
俺の勝ち

ちなみに
ID:AT99r3l3(>>24,29) → 9+9/(3*3)=10
0985132人目の素数さん
垢版 |
2018/07/28(土) 19:33:18.55ID:57kIc8+e
そろそろ次スレを
0986132人目の素数さん
垢版 |
2018/07/28(土) 20:56:04.78ID:zqnKg1oN
>>981
不正解
>>979でも言ったけど左右対称じゃない
0987132人目の素数さん
垢版 |
2018/07/28(土) 20:58:42.11ID:zqnKg1oN
>>981
しかもそれ4より大きいじゃん
0988132人目の素数さん
垢版 |
2018/07/28(土) 21:04:35.57ID:5RD8Md9I
数列{a_n}を以下のように定める。
a_1 = 3
a_(n+1) = (a_n)^2 - 2
この時、 a_n が合成数になるような n は存在するか。
0989132人目の素数さん
垢版 |
2018/07/28(土) 21:36:52.34ID:Nf1txf93
>>988
mod 1087で考えると
a_1≡3
a_2≡7
a_3≡47
a_4≡33
a_5≡0
明らかにa_5>1087なのでa_5は合成数
0990132人目の素数さん
垢版 |
2018/07/28(土) 21:46:55.05ID:boOQAkuB
ちなみにmod 127でも
a_1≡3
a_2≡7
a_3≡47
a_4≡48
a_5≡16
a_6≡0
a_6>127よりa_6は合成数

1087も127も勘で見つけた
0991132人目の素数さん
垢版 |
2018/07/28(土) 22:05:52.09ID:ttDOnSiN
>>990
正解、1087は見つけられんかったわ すごい
pがメルセンヌ素数の時にフィボナッチ数列がmodpでp+1を周期に持つ条件やら何やらを考えてて127を偶然見つけたけど、
メルセンヌ素数かどうかの判定法でリュカテストというのがあって、殆ど同じことやってたのを問題出してから知った…
0992イナ ◆/7jUdUKiSM
垢版 |
2018/07/28(土) 22:32:30.32ID:6VVd4WCT
>>981対角線2つのほかに、あえて対称じゃない分割線を一本引いたのに、対称と言われた。
――――――――――
@対角線1つ=(1+√5)/2
A対角線から最寄りの頂点への垂線=(1/4)√(10-2√5)
B中心角72°の扇形の弧=2π/5
C扇形の弧から残りの頂点への垂線={(1+√5)/2}-1
――――――――――
@+A+B+C=√5+2π/5+(1/4)√(10-2√5)
=4.08049029……ぉしい!!
0993132人目の素数さん
垢版 |
2018/07/28(土) 22:40:01.22ID:boOQAkuB
まあa_5, a_6をwolframに因数分解してもらって、modで書き直しただけなんだけど


余談だが、素数を無限に生成する関数
強い順に
f(n)=p_n
{f(n)}=Pかつf(m)≠f(n)
{f(n)}=P
{f(n)>0}=P
は存在するが、いずれも人為的なものであり実用性は乏しい(下の論文では"engineered"と表現している)

漸化式で定義された数列では
a_1=7
a_n=a_(n-1)+gcd(n, a_(n-1))
の階差数列b_nは1か奇素数になる
しかも全ての奇素数が現れるという
{a_n}=7,8,9,10,15,18,19,20,21,22,33,36,37,…
{b_n}=1,1,1,5,3,1,1,1,1,11,3,1,…

https://cs.uwaterloo.ca/journals/JIS/VOL11/Rowland/rowland21.html
0994132人目の素数さん
垢版 |
2018/07/28(土) 22:45:27.23ID:XEewS8qw
>>983
そりゃできるんじゃね?
|a b c d|
|b ax d cx|
|c d ay by|
|d cx by axy|
なら行列は0行0列から数えるとして
1の位が0の行、つまり0行目と2行目に√xをかけ、1の位が1の列、つまり1列目と3列目を√xで割る。
同じことを2の位について√yで行えば√x=u、√y=vとして
|auv bv cu d|
|bv auv d cu|
|cu d auv bv|
|d cu bv auy|
となって結局
|A B C D|
|B A D C|
|C D A B|
|D C B A|
を考えることになる。
2行目+3行目+4行目を1行目にたせば1行目は全部A+B+C+Dだからdetは(A+B+C+D)で割り切れる。
ー2行目+3行目ー4行目を1行目にたせば1行目は全部A-B+C-Dだからdetは(A-B+C-D)で割り切れる。
2行目ー3行目ー4行目を1行目にたせば1行目は全部A+B-C-Dだからdetは(A+B-C-D)で割り切れる。
ー2行目ー3行目+4行目を1行目にたせば1行目は全部A-B-C+Dだからdetは(A-B-C+D)で割り切れる。
A^の係数は1だからdet = (A+B+C+D)(A-B+C-D)(A+B-C-D)(A-B-C+D)。
これ2^2でやったけど2^nでもできると思う。
0995132人目の素数さん
垢版 |
2018/07/28(土) 22:58:36.93ID:Sc9m8D2O
>>994
なるほど
0996132人目の素数さん
垢版 |
2018/07/28(土) 23:00:35.26ID:Sc9m8D2O
2^nだとどう並べたら良いかな
0997132人目の素数さん
垢版 |
2018/07/28(土) 23:17:01.50ID:XEewS8qw
とりあえず2^2のパターンを2つつかって2^3は
A B C D E F G H
B A D C F E H G
C D A B G H E F
D C B A H G F E
E F G H A B C D
F E H G B A D C
G H E F C D A B
H G F E D C B A
でこのパターンをまた文字変えて並べて…でいけると。
1,-1のパターンは
n=1のとき1,1と1,-1
n=2のとき1,1,1,1と1,-1,1-1と1,1,-1,-1と1,-1,-1,1 (2つコピペして並べたものとそのままと-1倍したものを並べたもの)
n=3のとき1,1,1,1,1,1,1,1,と1,1,-1-1,1,1,-1-1と1,-1,1,-1,1,-1,1,-1と1,-1,-1,1,1,-1,-1,1と…
でいけると思う。このパターンで各行を足したり引いたりしたら全成分同じ値が並ぶ行が出てくると思う。
0998132人目の素数さん
垢版 |
2018/07/28(土) 23:46:15.45ID:Sc9m8D2O
A=[[a,b],[b,a]]という形式の行列でテンソル積を取っていけばよいのかな>>997
A*A=[[aA,bA],[bA,aA]]
A*A*A=[[aA*A,bA*A],[bA*A,aA*A]]
みたいな
ただし
aA=[[aa,ab],[ba,bb]]
の成分は非可換でA*^nの成分はaaa…aからbbb…bまでの2^n通りで
0999132人目の素数さん
垢版 |
2018/07/28(土) 23:54:37.19ID:Sc9m8D2O
そしたら
|A*^(n+1)|=|[(a+b)A*^n,(a+b)A*^n],[bA*^n,aA*^n]|=|[a+b)A*^n,O],[bA*^n,(a-b)A*^n]|=|(a+b)A*^n||(a-b)A*^n|
から上手く因数分解した形で求められそう
1000132人目の素数さん
垢版 |
2018/07/28(土) 23:58:12.29ID:XEewS8qw
>>998
テンソル積でうまく表現できるかもですね。
いま思いついたんだけどGを可換有限群としてGの元gに対応する不定元Agを用意しておいてg行h列がAghである行列にすればよさそう。
GがZ/2Zをn個直積した場合が今回の例でG=Z/nZの場合が巡回行列の行列式の理論になる。
その行列式はGの既約指標x(g)にたいしてΣ[g] x(g)Agの形の一次式をn個の指標全体でかけ合わせたものになると思う。
それで今回の話も巡回行列の行列式の理論も同様に説明できるみたい。
10011001
垢版 |
Over 1000Thread
このスレッドは1000を超えました。
新しいスレッドを立ててください。
life time: 159日 23時間 37分 2秒
10021002
垢版 |
Over 1000Thread
5ちゃんねるの運営はプレミアム会員の皆さまに支えられています。
運営にご協力お願いいたします。


───────────────────
《プレミアム会員の主な特典》
★ 5ちゃんねる専用ブラウザからの広告除去
★ 5ちゃんねるの過去ログを取得
★ 書き込み規制の緩和
───────────────────

会員登録には個人情報は一切必要ありません。
月300円から匿名でご購入いただけます。

▼ プレミアム会員登録はこちら ▼
https://premium.5ch.net/

▼ 浪人ログインはこちら ▼
https://login.5ch.net/login.php
レス数が1000を超えています。これ以上書き込みはできません。